AW5.5大牛的新GRE argument高频提纲(50之后的)

时间:2024.5.8

113,假设+-

1,课程有效果。学生一可能本来速度就很快,也没有数据说明他阅读的效率,也许读得快但理解记住很少。学生二的成就可能使他因造成。两个学生样本太少。

2,对我们公司有用。公司不同,工作内容,员工都不同,我们的员工可能阅读已经不错了。

3,所有的员工都需要这个培训。工种不同。培训值这个钱,没有相关信息。

77假设

1,municipal golf course 和resort hotel导致效益而不是其他。 Actually contribute to the economy of OV

本市没有高尔夫球场和假日酒店,或者说有但不吸引人。(值不值得借鉴?)我们原来就比隔壁好。even,隔壁的情况与我们相同,农村与城市,海边与高原。

3,这是最好的办法。没有证据表明这种比其他好,也没有说预期的增长

152问题

1,是否盈利了?没有任何信息反映利润情况,可能卖的多但由于价格低还是亏了。 2, 顾客喜欢bb是因为低价格?是不是因为口味包装或者单纯的想尝试新产品?为何不需要提价?可能提价了顾客不变但利润高了

3,一个产品的成功就能推广到所有食品?可能别的做不好。拓展业务还要考虑经济环境,顾客情况,竞争对手。

78,证据

1,没证据表明F不好。一个月的样本过小,可能天气原因,食品自身原因。虫害。可能刚开始不习惯。

2,没证据表明B比F好。两地不可比。公司规模,存储东西不同等等,虫害程度。可能差价太高。

3,其他原因可能导致选F更好。补偿服务,其他服务。

132假设

1,是青少年造成了事故,驾驶责任。可能是其他原因,青少年可能只是受害者。可能是环境天气因素等。开课程有必要?

2,父母无力教驾驶,抱怨的人数没说明。没时间教不代表不能保证孩子安全驾驶。没给出相关比例。Percentage of

3,结论,开课程是唯一有效的解决问题途径。强制?有没有经费,学生是否会接受?

69/70 question

1,Z比A好?首先,地方不同,受气候,地形等影响,维护相关费用可能有不同,A、地潮湿。第二,楼的用处直接决定了维修费用和能耗。三,可能Z的造价过高,大大超过了今后节省的维修 费用

公司,可能未来变差了。

74问题

1,入学学生是否就不会增多?调查的是以入学的学生,没说真话。没统计潜在学生。K可能改了人家还是愿意来。

2,校友支持是否会降低?校友的意愿不应是主要考虑因素,该跟着眼于学生,教育本身。而且改革以后可能产生正面效果让校友改变看法。Even,可能校友资助的减少相比改变后的收入增加微不足道。

3,即使学生不喜欢校友不支持,是否就不该改变?学校该更多考虑教育本身目的,而不是一味顺从学生意愿。社会是男女共存了,今后更易适应社会。

64问题

攻击第一段: 手工比人工有价值?

即便承认作者的关于“大雕象是模具做的”“小雕象是雕刻出的”的观点,作者的结论依然基于一个未经证实的前提:“手工雕刻做的比模具做的古物雕象更有价值”。在古物雕象里面完全可能并不是依靠这个标准,或者说至少不是唯一依靠这个标准来判断价值的。他们必然还有考虑其他很多因素,比如这类雕象的存世数量,他们本身的艺术水平,以及他们的完整程度,等等这样的因素。另外即便说,大雕象可能因为是模具制作的,所以价格下降。但是小雕象并不是从考古发现后才被认为手工制造的,为什么之后他会涨价呢?作者应该提供其他的证据来支持这一说法。

攻击第二段:攻击第二层次中的大雕象,即——大雕象是模具做的?。

作者关于“大雕象是模具做的”这个观点是值得怀疑的。

1. 没有任何证据表明,发掘出来的模具,能match存世的大雕象。所以完全有可能,这个是K岛另个文明时期的模具,与现在的那些大雕象没有任何关系。

2. 即便发现的模具真是用于大雕象,也没有证据表明,大雕象只是靠模具制作的。因为只找到了手和头的模具,所以我们完全有理由相信,其他的一些身体部分,是用手工雕刻完成的。并且限于当时的技术水平,手和头的模具,很有可能只能用于初步的成型,而进一步细化很有可能也是靠人工雕刻完成的。

攻击第三段:攻击第二层次中的小雕象,即——小雕象是人工雕刻的?

作者关于“小雕象是人工雕刻的”这个观点也是值得怀疑的。

即便如今找到的模型不能用来做小雕象,但是这并不能说一定不存在能做小雕象的模型啊。更何况这样的模型相对于大模型来说更容易制作。很有可能的确存在过这样的模型,只是因为某些原因已经损毁了,或者至今仍然没有被找到。

攻击第四段:指出作者搬起石头砸自己的脚!!!

作者关于“没有雕刻工具”的证据,削弱了文章的说服力,应该避免。虽然,作者想用这个

证据来证明,大雕象不是用雕刻工具做的,但是我们同样可以因为这个原因来怀疑小雕象也不是用雕刻工具做的。

结尾段:

所以,为了巩固作者的论述,他应该提供一些更详细,并且有说服力的证据。

方。没有具体人数无法与C比较

园人流量的决定性因素。他因:环境美观,搞活动演出,娱乐设施。

更愿意做草地。其他使S受欢迎的办法。

修路。相比给交通带来的便利,价钱和一时的扰动是可以接受的。

通问题解决。污染主要附近工厂的,汽车很多都是电动的。

交通。可能是修新的路了。

拼车的坏处,事故发生。And,其他解决办法。

实环境中物理化学因素的加入。

少很多。其次可能是他因。

力宣传。样本太小,可能其他店不是这样。杂志调查不可信。订约人数多少,有没有代表性,特殊人群。.

可能影响很大的客户群。

导致总产量显著增加。可能单个农庄产量下降从而总量只是略微上涨甚至持平。

胀。调查具体盈利和其他产品主比较。

没有任何具体资料表明养宠物有什么好处。

可能有细菌,感染,增加其他医药费。

过度关心造成坏效果。

AW55大牛的新GREargument高频提纲50之后的

AW55大牛的新GREargument高频提纲50之后的

AW55大牛的新GREargument高频提纲50之后的

AW55大牛的新GREargument高频提纲50之后的

肥的人,年轻女性。

标准。口味,外观影响买不买。

其次可能还是有很大一部分人会买。再次,可能公司已经开始开发新产品了。

AW55大牛的新GREargument高频提纲50之后的

AW55大牛的新GREargument高频提纲50之后的

AW55大牛的新GREargument高频提纲50之后的

AW55大牛的新GREargument高频提纲50之后的

AW55大牛的新GREargument高频提纲50之后的

AW55大牛的新GREargument高频提纲50之后的


第二篇:GRE_Argument+issue终极提纲


GRE Argument终极提纲

1

使用说明:

这里是Argument全部242题的完整提纲。题号后面括号里的三项分别是:题目讨论的题材/结论的性质/主干论证过程。题目中带下划线的部分为题目背景信息;斜体部分为题目假设;无任何标记部分为一般论据;黑体部分为题目结论。题目中括号里的数字表明这句话体现的是提纲中第几号缺陷。题目后面的星星数量表示题目论证难度,星数越多题目越难。题目后面的数字为论证结构与之类似的其他题目题号,可以参考准备。

提纲中具体缺陷后面括号里斜体缩写表示该缺陷所属的类型,缺陷后面的星号表示该缺陷在本题的严重程度,星数越多缺陷越严重。

题目顺序以新东方写作教程第一章题库顺序为准。

符号索引:

V.D: vague data I.I: incomplete information U.C: unrelated concepts C.S: changing scopes

F.A: false analogy I.C: incomplete comparison F.D: false dilemma I.T: incomplete thought

NCR: non causal relationship I.E: insufficient evidence U.A: unwarranted assumption

Adv:disadv: failing to weigh the advantage and disadvantage thoroughly

P→F/ P→C/ C→F/C→P: inferring future (current/past) condition from past (current) condition

2

1, (商业/决策/并列)The following appeared in a memorandum written by the vice president of Nature's Way, a chain of stores selling health food and other health-related products. We should therefore build our next new store in Plainsville(5), which has many such residents(8). Plainsville merchants report that sales of

running shoes and exercise clothing are at all-time highs(1). The local health club, which nearly closed five years ago due to lack of business, has more members than ever, and the weight training and aerobics classes are always full(2,7). We can even anticipate a new generation of

customers: Plainsville's schoolchildren are required to participate in a 'fitness for life' program, which emphasizes the benefits of regular exercise at an early age(3,4)." ★★14,40

1. Strong sales of exercise apparel do not necessarily indicate that Plainesville residents would be interested in NW's products, or

that these residents are interested in exercising. (U.C) ★★★

2. The popularity of the local health club is little indication that the residents in Plainsville live much healthier lives now, and that NW will

earn a profit from a store in Plainesville. (NCR)

3. The memo contains no evidence to support the assumption that people who exercise regularly are also interested in buying health food

and health-related products. (U.C) ★★★

4. The fact that a certain fitness program is mandatory for Plainesville's schoolchildren accomplishes nothing toward bolstering the

recommendation. (NCR) ★★★

5. The arguer does not provide any information regarding the cost of building such a new store thus we can not be convinced that

the new store will be profitable.(adv:disadv) ★★

6. The arguer fails to consider other possible factors besides residents? interest in healthy lives that may contribute to the success of

their existing stores.(confusing concurrence with causality) ★★

7. The recommendation fails to substantiate that this interest will continue in the foreseeable future. (P→F) ★★★★

8. The arguer unfairly assumes that Plainesville residents will prefer NW over other merchants that sell similar products. (U.A/I.T) ★

——————————————————————————————

2, (社区,地产/提议/类比)The following appeared in a letter sent by a committee of homeowners from the Deerhaven Acres to all homeowners in Deerhaven Acres.

"Seven years ago(3), homeowners in nearby Brookville community adopted a set of restrictions on how the community's yards should be

landscaped and what colors the exteriors of homes should be painted. Since then, average property values have tripled in Brookville(1). In order to raise property values in Deerhaven Acres, we should adopt our own set of restrictions on landscaping and housepainting(2,4,5). " ★25,237

1. The committee fails to substantiate the assumption that this course of action was responsible for the increase in Brookville

property values. (post hoc, ergo propter hoc) ★★★★★

2. The committee fails to consider possible differences between Brookville and Deerhaven that might help to bring about a

different result for Deerhaven. (F.A) ★★★★★

3. Many factors that could influence property value may change during these years, and the committee fails to rule out these

factors.(P→F) ★★★

4. The proposed action may result in some undesired consequences.(adv:disadv) ★★

5. The proposed action may be opposed by some homeowners, or may not be effectively executed by all the homeowners in

Deerhaven Acres.(feasibility of the conclusion) ★★★★

——————————————————————————————

3, (就业/论断/对比+并列)The following appeared in a newspaper article about law firms in the city of Megalopolis.

"In Megalopolis, the number of law school graduates who went to work for large, corporate firms declined by 15 percent over the last three smaller firms. In a survey of first-year students at a leading law school, most agreed with the statement that earning a high salary was less important to them than job satisfaction(2,3). This finding suggests that the large, corporate firms of Megalopolis will need to offer graduates more benefits and incentives and reduce the number of hours they must work.(4,5)" ★★★

1. The 15% decline that the author cites is not necessarily due to the vocational preferences of new law-school graduates. Perhaps 3

during the last three years Megalopolis' large firms have had fewer and fewer job openings for these graduates. (NCR) ★★★

2. The vocational goals of first-year law students cited in the survey do not necessarily reflect those of graduating students. (U.C)

★★★

3. The goals of students at one law school do not necessarily reflect those of the overall pool of graduates that might seek

employment with Megalopolis law firms.(C.S) ★★★

4. The author falsely equates the proposed tangible incentives with job satisfaction.(U.C) ★★★★

5. Small firms have many other advantages that large ones do not have, thus the large, corporate firms need to adopt other policies

to attract more graduates.(sufficiency of the solution) ★★★

——————————————————————————————

4, (商业,地产/建议/对比+并列)The following was posted on an Internet real estate discussion site.

"Of the two leading real estate firms in our town—Adams Realty and Fitch Realty—Adams is clearly superior. Adams has 40 real estate agents. In contrast, Fitch has 25, many of whom work only part-time(1,2,5). Moreover, Adams' revenue last year was twice as high as that of Fitch, and included home sales that averaged $168,000, compared to Fitch's $144,000(3,4,6). Homes listed with Adams sell faster as well: ten years ago, I listed my home with Fitch and it took more than four months to sell; last year, when I sold another home, I listed it with Adams, and it took only one month(7). Thus, if you want to sell your home quickly and at a good price, you should use Adams." ★★

1. The author provides no evidence that the quality of a real estate firm is directly proportional to the number of its agents or the number of

hours per week that its agents work. (U.C) ★★★

2. The author does not provide any information about how many Adams agents work part-time.( ex parte information) ★★

3. Last year?s sales result may not be representative. (selective sample) ★★★

4. The disparity in sales volume can readily be explained by factors other than the comparative quality of the two firms, such as serving area

and location.(NCR) ★★★★

5. For lack of data concerning the average number of homes sold per agent, we cannot evaluate the actual performance of agents in the two

firms.(V.D) ★★

6. The fact that the average sales price of a home sold by Adams is higher than the average price of a home sold by Fitch does not

indicate that Adams is more effective in selling homes than Fitch. (NCR) ★★★★

7. The disparity in the speed of a sale is explainable by other plausible factors that would change through the time, or some

essential differences of the two properties. (F.A/I.C/P→F) ★★★★★

——————————————————————————————

5, (医药/论断,预言/递进)The following appeared in the business section of a newspaper. of the drug(2). Thus, it is more likely that in ten years the most profitable pharmaceutical company will be Perkins Pharmaceuticals, maker of a new drug called Xylan, which clinical studies show is preferred over Xenon by seven out of ten patients suffering from the most extreme cases of arthritis(3,4)." ★★★★

1. The arguer falsely assumes that a rise in the number of arthritis sufferers implies a rise in profitability for arthritis drug

manufacturers, future competition may diminish the profit for each individual company. (I.E) ★★★

2. The expiration of the patent on Xenon does not necessarily lead to the decline in the profit of Becton, because Becton may have

the advantage of market seniority. (I.E) ★★★★

3. The preference of particular patients does not indicate that Xylan is a more profitable drug, we do not know the manufacturing

cost and sales price of Xylan. (adv:disadv) ★★★

4. We do not know what fraction of patients suffer from the most extreme cases of arthritis. (V.D) ★★

5. There may be new companies and new technology emerging in the future. (P→F/F.D) ★★★

——————————————————————————————

6, (文化娱乐/论断/并列)The following was written as a part of an application for a small business loan by a group of developers in the city of Monroe.

4

"A jazz music club in Monroe would be a tremendously profitable enterprise. Currently, the nearest jazz club is 65 miles away(1); thus, our proposed club, the C Note, would have the local market all to itself(3,7). Plus, jazz is extremely popular in Monroe: over 100,000 people attended Monroe's jazz festival last summer, several well-known jazz musicians live in Monroe(4), and the highest-rated radio program in

Monroe is 'Jazz Nightly,' which airs every weeknight(2). Finally, a nationwide study indicates that the typical jazz fan spends close to $1,000 per year on jazz entertainment(5,6). It is clear that the C Note cannot help but make money."

★★★235

1. If the demand for a live jazz club in Monroe were as great as the applicant claims, it seems that Monroe would already have one or more

such clubs. (negative evidence) ★★★

2. The popularity of Monroe's annual jazz festival and of its nightly jazz radio show does not necessarily indicate that jazz is extremely

popular in Monroe. (NCR) ★★★★

3. Granted that jazz is extremely popular in Monroe, we cannot ensure that those jazz fans will attend the jazz club. (U.A) ★★★

4. The mere fact that several well-known jazz musicians live in Monroe lend no significant support to the applicant's claim. (I.E) ★★★

5. The result of the nationwide study could not ensure that Monroe residents are willing to spend much money on jazz entertainment. (C.S)

★★★★

6. The arguer also does not inform us how much of the $1000 goes to club admission. (V.D) ★★★

7. The scale of Monroe?s jazz market, and the profitability of the proposed club are open to doubt. (I.I) ★★★★

——————————————-——————————————-

7, (政治经济,社会/提议/并列)The following appeared in a letter to the editor of the Clearview newspaper.

"In the next mayoral election, residents of Clearview should vote for Ann Green, who is a member of the Good Earth Coalition, rather than for Frank Braun, a member of the Clearview town council(3), because the current members are not protecting our environment. For example, during the past year the number of factories in Clearview has doubled(1), air pollution levels have increased, and the local hospital has treated 25 percent more patients with respiratory illnesses(2). If we elect Ann Green, the environmental problems in Clearview will certainly be solved(4,5)." ★★★148

1. The argument unfairly assumes that last year's increase in the number of factories was due to the city council's decisions—rather

than to some other phenomenon—and that this increase poses environmental problems for Clearview. (NCR) ★★★★

2. The argument also assumes unfairly that last year's increase in the number of patients reporting respiratory problems indicates

worsening environmental problems in Clearview. (NCR) ★★★★

3. The editorial provides no evidence to substantiate the assumption that Braun was a factor in the city council?s decisions. (I.I) ★★

4. The mere fact that Green is a member of the Good Earth Coalition hardly suffices to prove her willingness and ability to help solve

Clearview's environmental problems. (I.E) ★★★★★

5. The author provides no firm evidence that electing Green is necessary to solve those problems, or that electing Green would suffice.

(necessity and sufficiency of the solution) ★★★

——————————————————————————————

8, (食品,商业/提议/并列)The following appeared in a memorandum issued by the strategic planning department at Omni Inc. growth potential. Mesa enjoyed a 20 percent increase in profits last year(3,4), and its best-selling product, Diabolique Salsa, has had increased ★★★

1. The assumption that Mesa's snack foods appeal to 14-to-25-year olds is open to doubt. (U.A) ★★★★

2. The increased sales of Diabolique Salsa in the region where it is sold does not indicate that it will also gain success nationwide. (C.S)

★★★★★

3. We do not know how large is the snack food market in the region where Mesa?s products are sold, thus the 20% increase may be

insignificant. (V.D) ★★

4. Last year's 20% increase in Mesa's profits may be an aberration, no evidence could indicate that this trend will continue in the

future. (P→F) ★★★

5. We do not know the profitability of other food products of Mesa Food, thus could not evaluate if Omni should buy the entire

company. (I.I/necessity of the solution) ★★★★★

——————————————————————————————

5

9, (教育/提议/递进)The following appeared in a memorandum from a dean at Omega University. Omega have risen by thirty percent.(2) Potential employers apparently believe the grades at Omega are inflated; this would explain why Omega graduates have not been as successful at getting jobs as have graduates from nearby Alpha University(3,7). To enable its graduates to secure better jobs, Omega University should now terminate student evaluation of professors.(4,5,6)"

★★★211, 238

1. The dean provides no evidence about the number or percentage of Omega students who participate in the procedure. (V.D) ★★

2. The argument also assumes unfairly that the grade-average increase is the result of the evaluation procedure—rather than some other

phenomenon. (post hoc, ergo propter hoc) ★★★★

3. The dean's claim that grade inflation explains why Omega graduates are less successful than Alpha graduates in getting jobs is

unjustified. (NCR) ★★★★

4. The dean ignores other possible ways by which Omega can increase its job-placement record. (necessity of the solution) ★★★

5. Merely terminating the evaluation system might not suffice to enable Omega?s graduates to secure better jobs. (sufficiency of the

solution) ★★★★

6. The dean fails to mention the positive effects of the evaluation system. (adv:disadv) ★★★★★

7. The graduates from the two universities may not be comparable. (F.A) ★★

——————————————————————————————

10, (政治经济/对策/先并列后递进)The following appeared in a letter to the editor of a Batavia newspaper.

"The department of agriculture in Batavia reports that the number of dairy farms (2)throughout the country is now 25 percent greater than it was 10 years ago. During this same time period, however, the price of milk at the local Excello Food Market(1,3) has increased from $1.50 to over $3.00 per gallon(4,5,6). To prevent farmers from continuing to receive excessive profits on an apparently increased supply of milk, the Batavia government should begin to regulate retail milk prices(8). Such regulation is necessary to ensure both lower prices and an adequate supply of milk for consumers(7)." ★★

1. The author unfairly assumes that Excello's milk prices reflect those throughout Batavia. (C.S/quantity of the sample)

★★★★

2. The number of dairy farms does not necessarily indicate the supply of milk. (U.C) ★★★★

3. Even the production of milk in the country increased in general, no information is offered to show whether the production in Excello also

increased. (C.S) ★★★

4. We are not informed about how much did the price of milk actually increase after adjustment for inflation. (I.T) ★★★

5. The author fails to consider the variation in the demand of milk. (I.T/I.I) ★★★

6. The author does not provide a cost benefit analysis. (I.I) ★★★

7. The author fails to consider other possible methods that may guarantee lower prices and an adequate supply. (necessity of the solution) ★

★★

8. The proposed regulation may lead to undesirable consequences. (I.T/unexpected consequences) ★★★★★

——————————————————————————————

11, (社会/预言/并列)The following appeared in a memo from the mayor of the town of West Egg. years(2,3). Next month the amount of material recycled should further increase, since charges for garbage pickup will double(5). Furthermore, over ninety percent of the respondents to a recent survey(6) said that they would do more recycling in the future. Because of our residents' strong commitment to recycling, the available space in our landfill should last for considerably longer than predicted.(4,7)" ★★★

1. Aluminum and paper may account for only some of the materials West Egg's residents can recycle. (V.D))

★★★

2. We do not know the actual amount of recycled garbage in previous years, thus the doubling may be insignificant. (V.D) ★★★

3. An increase in the amount of recycled materials does not necessarily indicate a decrease in the total amount of trash deposited in the

city's landfill. (U.C) ★★★★

4. The recycling habits of West Egg residents are not the only factor affecting how quickly the landfill will reach capacity. (I.T) ★★★★ 6

5. The assumption that increased charges for trash pickup will serve to slow the rate at which the landfill is reaching capacity is

unwarranted. (U.A) ★★★

6. The mayor provides no evidence that the survey's respondents are representative of the overall group of people whose trash goes to the

city's landfill. (Are the respondents representative?) ★★★★

7. The mayor fails to consider many other factors that may change in the future. (P→F) ★★★★★

8. The consultants may have already taken the recycling factors into consideration when they made the prediction, thus the

assumption that the available space in our landfill will last for longer than predicted is unfounded. (U.A) ★★

——————————————————————————————

12, (生产,劳动安全/对策/先并列后递进)The following appeared in a memo from a vice president of Alta Manufacturing.

"During the past year, Alta Manufacturing had thirty percent more on-the-job accidents(2,4) than nearby Panoply Industries, where the work shorten each of our three work shifts by one hour so that our employees will get adequate amounts of sleep(3)." ★★181

1. The president provides no evidence that overall worker productivity is attributable in part to the number of on-the-job accidents. (NCR)

★★★

2. The president assumes that some accidents at Alta are caused by fatigue or sleep deprivation. (U.A) ★★★★

3. The assumption that Alta's workers would use the additional hour of free time to sleep or rest is open to doubt. (U.A) ★★★

4. The president fails to consider that the per-worker accident rate might reveal that Alta is actually safer than Panoply, depending on the

total number of workers at each company. (V.D) ★★★

5. The president fails to consider possible differences between Alta and Panoply which render them incomparable. (F.A) ★★★★

6. Merely shortening the work shifts might not suffice to reduce the number of on-the-job accidents. (sufficiency of the solution)

★★★

——————————————————————————————

13, (文化娱乐,商业/预言/并列)The following appeared in a memo from the chief executive of a video game company. short-term, there are two reasons that the purchase of these rights will undoubtedly be a wise investment. First, Wood's last three books have been best sellers(1) and the movie based on his first book was highly profitable(2). Second, the popular characters and story line in "Squirrel Power" (4)will save us time in developing our video game, thereby reducing our costs(3,5)." ★★★

1. The arguer fails to convince us that Squirrel Power will also be a best seller solely because its writer Peter Wood achieved

tremendous success in his last three books. (P→F) ★★★★★

2. The fact that the movie based on Wood?s first book was profitable does not ensure our game will also be profitable. (F.A) ★★

★★

3. The arguer fails to demonstrate that the revenue generated by the game will greater than the cost. (I.T/adv:disadv) ★★★★

4. The arguer unfairly assumes that the characters and story line in “Squirrel Power” will surely be popular. (U.A) ★★

5. The arguer fails to consider other costs in producing the game. (adv:disadv) ★★★★★

——————————————————————————————

14, (商业/决策/先并列后递进)The following appeared in a memo from the owner of Green Thumb Gardening Center, a small business serving a suburban town.

"There is evidence that consumers are becoming more and more interested in growing their own vegetables. A national survey(1) conducted last month indicated that many consumers(3) were dissatisfied with the quality of fresh vegetables available in supermarkets. And locally, the gardening magazine Great Gardens has sold out at the Village News stand(6) three months in a row(4,5). Thus, we at Green Thumb Gardening Center can increase our profits by greatly expanding the variety of vegetable seeds we stock for gardeners(2) this coming spring(7)." ★★1,40, 117, 219

1. The argument depends on a dubious assumption that the national levels of satisfaction with store-bought groceries could reflects the

level of a certain area. (C.S) ★★★★

2. The argument falsely assumes that consumers who are dissatisfied with store-bought groceries are likely to grow their own vegetables

instead. (U.A/I.E) ★★★★★

7

3. The arguer fails to provide any information about what fraction of consumers buy their vegetables from supermarkets. (V.D/selective

sample) ★★

4. The arguer fails to convince us that the gardening magazine Great Gardens is indeed popular among residents. (I.I) ★★

5. We do not know the content and circulation of the magazine, thus the fact that the magazine has sold out three months in a row

does not necessarily indicate that people in this area are interested in planting vegetables themselves. (I.I/I.E) ★★★

6. The situation at one certain news stand does not prove that people in this area are interested in gardening. (quantity of the sample)

★★★

7. The arguer fails to consider the possible cost of the proposed action. (I.T/adv:disadv) ★★★★

——————————————————————————————

15, (经济,投资/建议/大并列小递进)The following appeared in a newsletter offering advice to investors.

"Over 80 percent of the respondents to a recent survey indicated a desire to reduce their intake of foods containing fats and cholesterol(1,5), and today low-fat products abound in many food stores(2). Since many of the food products currently marketed by Old Dairy Industries are high in fat and cholesterol(7), the company's sales are likely to diminish greatly and their profits will no doubt decrease(3,4). We therefore advise Old Dairy stockholders to sell their shares and other investors not to purchase stock in this company(6)." ★★★★66

1. The author fails to assure us that the survey results accurately reflect the desires of most consumers, or that the results accurately predict

consumer behavior. (Are the respondents representative?) ★★

2. The fact that low-fat foods are in abundant supply in food stores does not necessarily indicate an increasing demand for low-fat dairy

products or a diminishing demand for high-fat dairy products. (NCR) ★★★★★

3. The newsletter concludes too hastily that Old Dairy profits will decline. (U.A) ★★★

4. The argument fails to provide any information about other products marketed by Old Dairy which may be low in fat and

cholesterol. (I.I) ★★★

5. The author fails to consider other factors that may influence the consumers? decision in choosing food. (I.T) ★★★

6. The mere fact that many Old Dairy?s food products are high in fat and cholesterol does not necessarily prove that their stock is

not worth investing. (I.T) ★★★★

7. The author ignores the possibility that Old Dairy may change its policy and main product in the future. (P→F) ★★★

——————————————————————————————

16, (社会/提议/大并列小递进)The following appeared in a letter to the editor of a local newspaper from a citizen of the state of Impecunia.

"Two years ago our neighboring state, Lucria, began a state lottery to supplement tax revenues for education and public health. Today, Lucria spends more per pupil than we do, and Lucria's public health program treats far more people(3,4) than our state's program does(1). If we were to establish a state lottery like the one in Lucria(2), the profits could be used to improve our educational system and public health program. The new lottery would doubtless be successful, because a survey conducted in our capital city (6)concludes that citizens of Impecunia already spend an average of $50 per person per year on gambling(5)." ★★

1. The arguer unfairly assumes that the current condition of education and health program in Lucria is due to the lottery. (post hoc, ergo

propter hoc) ★★★★

2. The two states may be different in many aspects, thus a similar lottery system may not prove helpful for Impecunia. (F.A) ★★★★

3. We do not know the base amount of citizens in each state, hence the fact that Lucria's health programs treat more people than

Impecunia's programs lends no support to the argument. (V.D) ★★★★

4. The number of people treated by a health program does not necessarily indicate the quality of health care of the program. (U.C)

★★★★

5. The fact that Impecunia's residents spend $ 50 per capita on gambling each year does not suffice to prove that residents will spend much

money on lottery. (U.C) ★★★

6. The situation in our capital city could not illustrate that people in other areas of the state are also interested in gambling.

(selective sample/C.S) ★★★

——————————————————————————————

17, (社会/论断/并列)The following appeared in a letter to the editor of the Walnut Grove town newspaper. 8

once(5). Moreover, EZ—which, like ABC, currently has a fleet of 20 trucks—has ordered additional trucks(2,3). Finally, EZ provides

exceptional service: 80 percent of respondents(6) to last year's town survey agreed that they were 'satisfied' (8)with EZ's performance(7)." ★★★41,93,233

1. The author fails to substantiate the assumption that the town would benefit from an additional collection each week. (U.A) ★★

2. The author does not inform us the usage of the newly ordered trucks, and that of the current fleet, of EZ. (I.I) ★★★

3. We could not rule out the possibility that ABC has also ordered additional trucks to its fleet. (ex parte information) ★★★

4. The author fails to consider other factors such as the credit, the procedure and technology of trash-collecting of each company. (I.T) ★

★★★★

5. The author fails to demonstrate that the residents care more about the frequency of trash collection than the charge. (U.A) ★

6. The author fails to provide assurances that the respondents of the survey are representative of the overall population of people

whose trash were collected by EZ. (are the respondents representative?) ★★

7. Those respondents, or citizens of our town, may be even more satisfied with ABC's services. (ex parte information) ★★

8. We do not know what question was asked in the survey, thus we could not evaluate whether the answer “satisfied” is significant.

(what question was asked in the survey?) ★★★★

——————————————————————————————

18, (道路交通/提议/大并列小类比小递进)The following appeared in an editorial in a Prunty County newspaper. accident rate throughout Prunty County (2)has decreased only slightly(3). If we want to improve the safety of our roads, we should instead undertake the same kind of road improvement project that Butler County completed five years ago(7): increasing lane widths and resurfacing rough roads(8). Today, major Butler County roads still have a 55 mph speed limit, yet there were 25 percent fewer reported(6) accidents in Butler County this past year than there were five years ago(5)." ★★★★55,119

1. The author fails to rule out the possibility that not enough time has passed to determine the effectiveness of this change in reducing the

accident rate. (I.E) ★★★

2. The author ignores the possibility that the accident rate on the county's major roads has decreased while on minor roads not subject to

the speed-limit reduction it has increased. (I.I) ★★★★

3. The argument assumes that all other factors affecting highway accident rates have remained unchanged since the county lowered its

speed limit. (P→F) ★★★

4. The fact that most drivers are exceeding the new speed limit does not indicate that the policy is ineffective. (NCR) ★★

5. The author unfairly implies that the higher speed limit in Butler County has not served to increase the incidence of road

accidents in that county. (lack of controlled experiment) ★★★

6. The cited statistic involves only "reported" accidents in Butler County, we do not know the percentage of accidents which are

going unreported in that county. (I.I) ★★★

7. The situation at Prunty County and Butler County may not be comparable. (F.A) ★★★★

8. The author fails to demonstrate that road improvement will be both sufficient and necessary to guarantee fewer accidents in

Prunty County. (sufficiency/necessity of the solution) ★★★★

——————————————————————————————

19, (商业/决策/递进)The following appeared in a letter from the manager of a rock band named Double Rice. Megalopolis sold out in 12 minutes(4). Clearly the ticket sales in Megalopolis are a result both of the band's increased popularity and of the advertising campaign run in Megalopolis by the Ad Lib advertising agency(1). Thus, in order to ensure that the band's success in Megalopolis is repeated across the country, the band should hire Ad Lib to duplicate the Megalopolis ad campaign on a nationwide scale(2,3)." ★★★★

1. The author fails to establish the causal relationship between Ad Lib?s campaign and the Megalopolis success. (NCR) ★★★★★

2. The manager's claim that this success can be repeated elsewhere through duplicating the Megalopolis ad campaign might be

unwarranted. (C.S) ★★★★

3. The manager assumes that Ad Lib's services are both necessary and sufficient for this purpose. (sufficiency/necessity of the 9

solution) ★★★★

4. The success in Megalopolis may have other causes, and is not a good indication of increased popularity of the band. (NCR) ★

★★

——————————————————————————————

20,(交通安全/提议/类比)The following appeared in a letter to the editor of the Balmer Island Gazette. pedestrians(4), the town council of Balmer Island should limit the number of mopeds rented by each of the island's six moped and bicycle rental companies(5) from 50 per day to 30 per day(6) during the summer season(7). By limiting the number of rentals, the town council is sure to attain the 50 percent reduction in moped accidents(1,4) that was achieved last year in the neighboring island of Torseau(3), when Torseau's town council enforced similar limits on moped rentals(2)." ★★★★

1. The author assumes that all other conditions in Balmer that might affect the rate of moped-pedestrian accidents will remain unchanged

after the restrictions are enacted. (P→F) ★★★

2. The author fails to consider other possible explanations for the 50% decline in Torseau's moped accident rate last year. (NCR) ★★★

3. Balmer Island and Torseau may not be comparable. (F.A) ★★★★

4. The author provides no evidence that the same restrictions that served to reduce the incidence of all "moped accidents" by 50%

would also serve to reduce the incidence of accidents involving "mopeds and pedestrians" by 50%. (U.C) ★★★

5. We do not know what percentage of mopeds in Balmer Island is rented by the six companies. (V.D) ★★★

6. The author fails to provide accurate data about the current number of mopeds rented by the six companies per day. (I.I) ★★

7. The proposed regulation may not be necessary for the author?s purpose. (necessity of the solution) ★★★

——————————————————————————————

21, (生产/决策/大并列小递进)The following appeared in a memo from the new vice president of Sartorian, a company that manufactures men's clothing. have not offered an alpaca overcoat for five years and since our major competitor no longer makes an alpaca overcoat(2), there will be pent-up customer demand. Also, since the price of most types of clothing has risen in each of the past five years(3,6), customers should be willing to pay significantly higher prices for alpaca overcoats than they did five years ago(4), and our company profits will increase(7)." ★★★

1. The author fails to substantiate the assumption that the new fabric supplier will be a reliable supplier of alpaca, and fails to provide any

information about the quality of the fabric they supply. (U.A/I.I) ★★★★

2. The fact that competitor stopped making alpaca coats may just indicate diminishing consumer demand for them. (negative evidence) ★

★★

3. Other factors may change during the past five years. (P→F) ★★★

4. The assumption that consumers will be willing to pay significant higher prices for alpaca overcoats is unwarranted. (U.A) ★★★

5. The author fails to consider the possible cost of resuming production. (adv:disadv) ★★★★

6. The fact that clothing prices have been steadily increasing for five years suggests that consumers might have less disposable income for

purchasing items such as alpaca coats. (negative evidence) ★★

7. The memo's claim that the company's overall profits would increase thereby is unwarranted. (U.A) ★★★

——————————————————————————————

22, (地产,商业/决策/并列)The following appeared in a memo from the president of a company that builds and sells new homes in Steel City.

"Over the past five years, the population of Steel City has increased by more than 20 percent, and family incomes in Steel City have risen much faster than the national average(4). Nationwide(7), sales of houses priced above $150,000 have increased more than have sales of lower-priced houses(5). Such data indicate that we should make changes in our business to increase company profits(6). First, we should build fewer

low-priced houses than we did last year and focus instead on building houses designed to sell at above $150,000(2,3). Second, we should hire additional workers so that we can build a larger total number of houses than we did last year(1)." ★★★★

1. The argument relies on an unsubstantiated assumption that area demand for new housing will support additional home construction in

the foreseeable future. (P→F) ★★★

10

2. Granted that more residents will purchase new houses, we cannot guarantee that they will choose expensive houses. (NCR) ★★

3. The speaker relies on other tenuous assumptions that area residents interested in buying new homes can afford homes priced over

$150,000. (U.A) ★★

4. We do not know the base amount of the population and average family income in Steel City, thus the increase may not support the

proposed home construction. (V.D) ★★

5. The speaker does not provide accurate data regarding the sales of expensive houses and lower-priced houses respectively and we cannot

evaluate the actual market of each kind of houses. (V.D) ★★★

6. The speaker does not take into account some negative effects of the proposal such as the cost, difficulty in selling expensive homes, etc.

(adv:disadv) ★★★★

7. Nationwide statistics can say little about the demand and preference of the local residents. (C.S) ★★★

——————————————————————————————

23, (商业,餐饮/预言/并列)A recent sales study indicated that consumption of seafood dishes in Bay City restaurants has increased by 30 percent over the past five years(1,2). Yet there are no currently operating city restaurants that specialize in seafood. Moreover, the majority of families in Bay City are two-income families, and a nationwide study(4) has shown that such families eat significantly fewer home-cooked meals than they did a decade ago but at the same time express more concern about eating healthily(5,6,8). Therefore, a new Bay City restaurant specializing in seafood will be quite popular and profitable(7). ★★

1. The 30% increase might be insignificant because we do not know the base amount of seafood consumption in Bay City. (V.D) ★★

2. The past current does not necessarily indicate that seafood consumption will continue to increase in the future. (P→F) ★★★

3. The increase in consumption of seafood dishes in normal restaurant does not indicate that a restaurant specializing in seafood will be

profitable. (I.E) ★★

4. The nationwide study showing trends among two-income families toward dining out and eating healthily does not necessarily apply to

Bay City. (C.S) ★★★

5. The fact that two-income families express more concern about eating healthily does not indicate that they will necessarily patronize a

new seafood restaurant. (U.C) ★★★★

6. Without any detailed data concerning the number of home-cooked meals these two-income families eat currently and formerly,

and without knowing the actual level of their concern about eating healthily, we cannot evaluate if these families are a key factor in determining the profitability of a restaurant specializing in seafood. Other families may eat even fewer home-cooked meals and are concerned more about eating healthily. (confusing comparison and variation/lack of comparison) ★★★★

7. The argument fails to provide any information about the expense of establishing such a restaurant. (adv:disadv) ★★★

8. The fact that two-income families eat fewer home-cooked meals may lead to the popularity of catering services, delivered meals,

but not necessarily that of a sit-down restaurant. (U.A) ★★★

——————————————————————————————

24, (生产/决策/大并列小递进)The following appeared in a memo from the president of Viva-Tech, a manufacturer of high-tech medical equipment.

"In order to reduce costs, we should close some of our existing small assembly plants and build a large central plant(1). Grandview would be an ideal location for this new plant(6). First, of the locations that we have considered, Grandview has the largest adult population, so that we will be able to staff our plant quickly and easily(2,3). Second, since the average wage earned by workers in Grandview is less than that in the other locations, we should be able to keep production costs low(4). Last, as an inducement for us to build there, Grandview's town council has offered to allow us to operate for the first three years without paying city taxes(5)." ★★★100,226,240

1. Building a large central plant may actually increase the cost. (adv:disadv) ★★★★

2. The large population in Grandview may not suitable for staffing the new plant. (I.E) ★★★

3. The president fails to consider if the residents at Grandview are currently unemployed, or are willing to quit their current job to

work at Viva-Tech. (feasibility of the conclusion) ★★★★

4. Although Grandview?s workers average wage is less than in other locations, other expense might be higher. (I.T) ★★★

5. We are not informed about taxation policy and the actual amount of city taxes in Grandview after the first three years. (I.I) ★★★

6. Other suitable locations might be available while ignored by the president. (necessity of the solution/ex parte information) ★★★ ——————————————————————————————

25, (城市/决策/类比)The following appeared in a memo from the mayor of the town of Hopewell.

11

"Two years ago(1,5), the town of Ocean View built a new municipal golf course and resort hotel. During the past two years, tourism in Ocean View has increased, new businesses have opened there, and Ocean View's tax revenues have risen by 30 percent(6). The best way(4) to improve Hopewell's economy, and generate additional tax revenues, is to build a golf course and resort hotel similar to those in Ocean View(2,3)." ★2,237

1. The arguer fails to point out how the golf course and resort hotel actually contributed to the economy of Ocean View. (post hoc,

ergo propter hoc) ★★★★

2. Ocean View and Hopewell may not be comparable. (F.A) ★★★★★

3. The argument relies on an unwarranted assumption that Hopewell lacks golf course and resort hotel, or that the existing ones are

not attractive enough. (U.A) ★

4. The arguer ignores other possible methods that may improve Hopewell?s economy more efficiently. (necessity of the solution)

★★★

5. The mayor assumes too hastily that the golf course and hotel will continue to benefit Hopewell's overall economy. (P→F) ★★★

6. The 30 percent increase in tax revenues of Ocean View does not indicate that its economic condition is better than Hopewell?s,

because we do not know the base amount of tax revenues to begin with. (V.D) ★★

——————————————————————————————

26, (人事/提议/并列)The following appeared in a memo from the chairperson of the school board in the town of Saluda.

"For the past five years, Mr. Charles Schade has been the music director at Steel City High School, and during that time the school band from Steel City High has won three regional band competitions(1). In addition, the quality of the music rehearsal facilities and musical instruments at Steel City High has improved markedly over the past five years(3). Because of such successes at Steel City High, the Saluda school board should hire Mr. Schade to plan and direct the general music education programs for the entire Saluda school system(2,4)." ★★85,140,209,225

1. The three band awards might not be attributable to Schade?s abilities and efforts. (NCR) ★★★★★

2. The skills that Schade possesses and that resulted in the band's winning these awards might not be the same skills required for the

district position. (F.A) ★★★★

3. The argument unfairly assumes that the improvements in the quality of music facilities and instruments in the school can be

attributed to Schade?s abilities and efforts. (NCR) ★★★★★

4. Other candidates might be even more suitable for the position. (necessity of the solution) ★★★

——————————————————————————————

27, (管理运营/决策/递进)The following appeared as part of a memo from the president of Automate, a company that manufactures automobiles. Sparks(2). In order to keep our best staff, we must pay them salaries equal to those Sparks pays its employees(6). Otherwise we will continue to lose employees in the future(3), because Sparks must staff the additional new plants that it plans to build in the state(7)." ★★★

1. We do not know what kind of jobs are being advertised by Spark. (I.I) ★★★

2. Other reasons may well explain why some of our employees left to work for Sparks. (NCR) ★★

3. The president fails to prove that there are still significant number of our employees would leave Automate if the company does

not raise its employees? salaries. (I.I) ★★★

4. We need to know what fraction of our employees left for Sparks, (V.D) ★★★

5. We need to know if the employees who left for Sparks are valuable for Automate. (I.I) ★★★★

6. The president does not consider other measures to keep its valuable employees rather than raising the salaries. (I.T) ★★★★

7. The fact that Sparks plans to build additional new plants does not indicate that Automate will continue losing valuable

employees. (I.E) ★★

——————————————————————————————

28,(教育/决策/大并列小递进)The following is a memo from the superintendent of the Mylar school district.

"A recent six-month study, in which breakfast was made available at school for 100 schoolchildren(1) ages five to twelve, found that children on the breakfast plan were less likely than other children to be absent from or late for school(2). Clearly, eating breakfast before school plays a role in reducing student absenteeism and tardiness. It is also well known that children who regularly eat a healthful breakfast tend to perform 12

better in school(4). Therefore, in order to reduce absenteeism and tardiness and to improve academic performance in all of Mylar's elementary and secondary schools(3), we should provide breakfasts(5) for all students(6,7) before each school day(8)." ★★★★

1. The participants of the study may not be representative of all schoolchildren. (are the respondents representative?) ★★★

2. The reduced absenteeism might have been due to other explanation. (NCR) ★★★

3. Secondary-school students might not behave the same as elementary schoolchildren under similar conditions. (F.A) ★★★

4. The better performance of the children cited by the superintendent may results from other reasons rather than eating breakfast

regularly. (NCR) ★★★★

5. The superintendent does not prove that the district?s breakfast would be healthy enough, or that our students will eat the

breakfast regularly. (U.A) ★★★

6. The feasibility of providing breakfast for each student is open to doubt. (feasibility of the conclusion) ★★★★

7. The conclusion that all students need to eat breakfast provided by the district is unfounded. (C.S) ★★★★

8. Other possible measures could also be applied to achieve the superintendents? purpose. (sufficiency/necessity of the solution) ★

★★

——————————————————————————————

29, (选举,环境/论断,建议/类比+her intention to close all(3) Lake City parks to automobile traffic(1,2), thus following the example of the former mayor of Plainsville, Alecia Yu . On the recommendation of a small group of concerned citizens, Mayor Yu prohibited automobile traffic in all Plainsville parks, and therefore was credited with solving the pollution problem and improving the quality of life in Plainsville(4). This action would have great appeal to the citizens of Lake City, most of whom participate in the community's recycling program(5), and would guarantee McAllister's success in her current quest to be Lake City's new mayor(6,7). ★★★

1. No evidence is provided to indicate that citizens of Lake City prefer prohibiting automobile traffic in all Lake City parks. (I.E)

★★★★★

2. The author fails to demonstrate that the environmental problems at Lake City are mainly caused by automobile traffic. (I.I) ★★★

★★

3. The author fails to consider the feasibility and necessity of closing all Lake City parks to automobile traffic. (feasibility of the

conclusion/C.S) ★★★★

4. The situations of Lake City and Plainsville might not be quite different. (F.A) ★★★

5. The fact that most of the citizens of Lake City participated in the community?s recycling program does not indicate that they are

also highly concerned about environmental problems. (U.C) ★★★★

6. The citizens of Lake City might be concerned about other problems rather than environment. (I.E) ★★★

7. The author ignores many other candidates who might be more competent than McAllister. (I.T) ★★★

——————————————————————————————

30,(社会/提议/大递进小并列)According to information recently reported in the Eliottown Gazette, the number of people who travel to Eliottown has increased significantly over the past several years. So far this year over 100,000 people have arrived on flights to Eliottown's airport, compared with only 80,000 last year and 40,000 the year before. Eliottown's train station has received more than 50,000 passengers this year, compared with less than 40,000 last year and 20,000 the year before(5). Clearly tourism(2) in Eliottown has been increasing, thanks to the new Central Park and Museum of Modern Art that opened last year(1). Therefore, the funding for the park and museum should be increased significantly(3,4). ★★★42

1. The author fails to point out how the Central Park and the Museum actually contributed to the tourism in Eliottown. (post hoc,

ergo propter hoc)★★★★★

2. The passengers arrived Eliottown may came here for purposes other than tourism. (I.I) ★★★★★

3. No evidence is provided to show that the park and museum need more funding. (necessity of the solution) ★★★

4. The author ignores other possible methods that may also improve Eliottown?s tourism. (necessity of the solution) ★★★

5. The situation may vary in the following years. (P→F) ★★

——————————————————————————————

31,(生活,安全/论断/并列)The following appeared in the editorial section of a newsmagazine.

13

folly. Although some(1) people with cell phones undoubtedly cause problems on the road, including serious accidents, the majority do not(2). Besides, problems are also caused by drivers who are distracted by any number of other activities, from listening to the radio to disciplining children. Since there is no need to pass legislation restricting these and other such activities(3,4), it follows that there is no need to restrict people's freedom to use a device that they find convenient—or helpful in emergencies(5)." ★★★★★

1. The author does not inform us what fraction of drivers who use cell phones involved in accidents. (V.D) ★★★

2. The fact that most people with cell phones do not cause problems does not indicate that there is no need to prevent the possibility

of accidents caused by using cell phones. (I.E) ★★★★

3. Other dangerous activities, as mentioned by the author, should also be restricted if necessary. (I.T) ★★★

4. Accidents caused by using cell phones might occur more frequently, or be much more serious, than accidents caused by other

activities. (F.A) ★★★

5. Restricting the use of cell phones during driving situation does not interfere with people?s freedom to use a convenient device in

emergencies. (U.C) ★★★★★

——————————————————————————————

32, (生活/预言/a resulting reduction in the consumption of vehicle fuel in Hiparia. ★★★★

1. The fact that citizens are beginning to do more shopping by mail or delivery does not indicate that there are no need for them to

drive to shopping malls. (I.I) ★★★★★

2. The author unfairly assumes that the current trend of increasing remote shopping will continue in the future. (P→F) ★★★

3. Fuel consumption of mail or delivery services may increase, thus the total consumption of vehicle fuel in Hiparia would not fall.

(I.T) ★★★★

4. The usage of vehicles for purposes other than shopping may increase. (I.T) ★★★

5. The author does not inform us how many people in Hiparia actually begin to do shopping through mail or the net, and what

fraction of their whole shopping can be completed by these methods. (V.D) ★★★★

——————————————————————————————

33, (考古/推测/并列)The following report appeared in an archaeology journal. levels of a certain metallic element contained in various foods are strongly associated with(3) people who migrated to a new place after

childhood. Many of the bones(2) found near the pots at a few sites(1) showed high levels of the metallic element. Therefore, it must be that the pots were spread by migration, not trade(4,5)."

★★★★★37

1. The discovery mentioned above was only found in a few sites, it may not be representative of the whole sites where the pots

were discovered. (quantity of the sample) ★★★★

2. The author unfairly assumes that the bones containing high levels of the metallic element belong to the pot makers. (U.A) ★★

★★

3. The author ignores other factors that may also result in high levels of the metallic element in bones. (NCR) ★★★★★

4. Activities other than migration and trade, such as hunting, traveling, etc., may also contribute to the spread of the pots. (F.D) ★★

★★

5. The author fails to provide sufficient evidence to rule out the possibility that the pots were spread by trade. (I.E) ★★★

——————————————————————————————

34, (保健/论断/并列)who have consistently consumed dairy products throughout the years of the study have a higher rate of bone fractures than any other 14

participants in the study(2,3,4). Since bone fractures are a symptom of osteoporosis(1), this study result shows that a diet rich in dairy products may actually increase, rather than decrease, the risk of osteoporosis. ★★★★49

1. Although bone fractures are a symptom of osteoporosis, they may well result from other causes. (U.C) ★★★★★

2. People who consistently consume dairy products in the study may do so just because they are more likely to suffer from bone

fractures. (confusing the cause and the effect) ★★★★★

3. The author does not provide detailed information about the studied people who consistently consume dairy products and those

who do not. (are the respondents representative?/I.C) ★★★★

4. Other reasons, such as aging, genetic factors, lifestyle, etc. may also account for the higher rate of bone fractures of the people

mentioned in the study. (I.T) ★★★

——————————————————————————————

35, (食品,保健/预言/并列)The following appeared in the summary of a study on headaches suffered by the residents of Mentia. commercial use of salicylates has been found to correlate with a steady decline in the average number of headaches reported by participants in With this new use for salicylates, we can expect a continued steady decline in the number of headaches suffered by the average citizen of Mentia.(4,5)" ★★★★

1. Salicylates may not have the same effect as aspirin in treating headaches although they are of the same chemical family. (F.A)

★★★★

2. Other factors may also lead to the decline in the number of headaches in the study mentioned above. (NCR) ★★★★★

3. The author does not inform us what fraction of foods consumed by citizens of Mentia contains salicylates, and how many of the

citizens prefer such kind of foods. (I.I) ★★★

4. The author fails to consider factors other than food, such as environment, lifestyle, etc. that may result in the increase in the

number of headaches suffered by citizens of Mentia would increase (I.T) ★★★

5. Granted that salicylates are effective in treating headaches, we cannot hastily infer that they would also be effective in preventing

headaches. (U.C) ★★★

——————————————————————————————

36, (人类学/论断、预言/——)The following appeared in an article written by Dr. Karp, an anthropologist. islands that includes Tertia(2) show that these children spend much more time talking about their biological parents(1) than about other adults in the village. This research proves that Dr. Field's conclusion about Tertian village culture is false, and thus that the observation-centered approach to studying cultures is invalid(3). Because they are using the interview-centered method, my team of graduate students working in Tertia will establish a much more accurate understanding of child-rearing traditions there and in other island cultures.(4)" ★★★★

1. Talking more about their biological parents does not indicate that those children were reared by their parents. (U.C) ★★★★★

2. We do not know how many of the children interviewed by Dr. Karp were actually from island of Tertia. (I.I ) ★★★

3. The facts cited by Dr. Karp do not necessarily prove that the observation-centered method used by Dr. Field is invalid. (I.E) ★

★★★

4. No evidence could guarantee that Dr. Karp?s team will establish a more accurate understanding of island cultures just because they

are using the interview-centered method. (I.E) ★★★★★

5. There may have been dramatic changes in nurturing patterns of Teria children during the past 20 years. (P→F) ★★★

——————————————————————————————

37, (考古/论断/大并列小递进)and broad(3), and so the ancient Paleans could only have crossed it by boat(1), but there is no evidence that the Paleans had boats. And boats capable of carrying groups of people and cargo were not developed until thousands of years after the Palean people disappeared. Moreover, 15

Paleans would have had no need to cross the river(5)—the woods around Palea are full of nuts, berries, and small game(4). It follows that the so-called Palean baskets were not unique to Palea(2). ★★★★★33

1. The author fails to consider the possibility that the Paleans may arrived in Lithos through other means. (I.T) ★★★★

2. The Palean basket could arrive other place through many possible methods such as trade, or boats possessed by other culture.

(I.T) ★★★★

3. In Palean times, the Brim River may not be as deep and broad as it is currently. (C→P) ★★★

4. The fact that there are abundant resources currently does not indicate that those resources also existed in Palean times. (C→P)

★★★

5. Other reasons other than food shortages may well result in the migration of a culture. (I.E) ★★★

——————————————————————————————

38, (保健/建议/递进)The following memo appeared in the newsletter of the West Meria Public Health Council.

"An innovative treatment has come to our attention that promises to significantly reduce absenteeism in our schools and workplaces. A study reports that in nearby East Meria(3), where fish consumption is very high, people visit the doctor only once or twice per year for the treatment of colds(1). Clearly, eating a substantial amount of fish can prevent colds(2). Since colds are the reason most frequently given for absences from school and work(4), we recommend the daily use of Ichthaid(5,6), a nutritional supplement derived from fish oil, as a good way to prevent colds and lower absenteeism." ★★★

1. The fact that people in East Meria rarely visit doctor for the treatment of colds does not indicate a low incidence of colds. (U.C) ★

★★★

2. The author fails to establish a causal relationship between high level of fish consumption and the low incidence of cold in East

Meria. (NCR) ★★★★★

3. The condition in East and West Meria may not be comparable. (F.A) ★★★

4. We need to know how many absences from school and work were actually caused by colds. (I I) ★★

5. Ichthaid may not have the same function in preventing colds as has eating fish. (H.G) ★★★★

6. The author does not inform us the cost, and any possible side effects of Ichtahaid. (adv:disadv) ★★★★

——————————————————————————————

39, (教育/决策/并列)The following appeared in a memorandum from the president of Humana College.

"Last year the number of students who enrolled in long-distance degree programs at Omni University increased by 50%(1,2). During the same year, Omni showed a significant decrease from prior years in expenditures for dormitory and classroom space, most likely because instruction in the long-distance programs takes place via interactive video computer connections(3). In contrast, over the past three years, enrollment at Humana College has diminished and costs of maintaining buildings have risen. Thus, to increase enrollment and solve the problem of budget deficits(6) at Humana College, we should initiate and actively promote long-distance degree programs(5) like those at Omni(4,7)." ★★★

1. We do not know the actual number of students who enrolled in long-distance program at Omni University last year, thus the 50%

increase might be insignificant. (V.D) ★★★

2. We do not know the total number of enrollment, and actual tuition income at Omni last year. (I.I) ★★★

3. Other factors may well explain the decrease in expenditures for dormitory and classroom space at Omni. (NCR) ★★★

4. Possibly Omni and Humana are not comparable at many aspects. (F.A) ★★★★

5. The president does not provide any information about the cost and any possible requirements of installing such a program.

(adv:disadv/feasibility of the conclusion) ★★★★

6. The president does not consider other possible causes that may result in budget deficits. (NCR) ★★★

7. The president overlooks other methods that could be applied to solve the problem. (necessity of the solution) ★★★

——————————————————————————————

40, (商业/决策/并列)The following appeared in a memorandum from the president of Excello Food Markets.

growing concern about foods grown using pesticides or preserved with chemicals(1). Recently our market in Sun City(3) participated in a local food tasting fair, and 75 percent of the fair goers who visited the Excello booth requested free samples of organic fruit(4). Such evidence indicates that to increase our profits, we should begin to stock a full line of organic food products in all (7)our markets(6)." ★★★1,14

1. The results of cited surveys do not necessarily indicate our customers will be interested in organic food. (U.C) ★★★★

16

2. The reliability of cited surveys is open to doubt. (are the respondents representative?) ★★

3. The situation in Sun City may not be representative of consumers all over the country. (quantity of the sample) ★★★

4. The fact that most fairgoers asked for samples of organic fruit does not indicate that most people are really interested in organic

foods. (are the respondents representative?/I.E) ★★★

5. The president does not provide any information concerning the profit of natural-food stores mentioned in the argument.

(adv:disadv) ★★★

6. The president does not consider the cost of stocking a full line of organic food products in every market. (adv:disadv) ★★★

7. It might be unnecessary to stock organic food in all markets. (C.S) ★★★★

——————————————————————————————

41, (生产,管理/决策/对比)The following appeared in a memo from the vice president of a food-distribution company with food-storage warehouses in several cities.

"Recently we signed a contract with The Fly-Away Pest-Control Company to provide pest-control services at our fast-food warehouse in Palm City, but last month we discovered that over $20,000 worth of food there had been destroyed by pest damage. Meanwhile, the Buzzoff

Pest-Control Company, which we have used for many years, continued to service our warehouse in Wintervale, and last month only $10,000 worth of the food stored there had been destroyed by pest damage(1). Even though the price charged by Fly-Away is considerably lower(2), our best(5) means of saving money is to return to Buzzoff Company(4) for all(3) our pest-control services." ★★17,93,233

1. The situation in Palm City and Wintervale might be quite different. (F.A) ★★★★★

2. The vice president does not inform us about the actual price charged by each company, thus we can not evaluate if we can save

money by adopting the president?s advice. (lack of comparison) ★★★★

3. There may be some difference between the specialty of the two company on pest-control, thus it is too hasty to use Buzzoff for all

our pest-control services. (C.S) ★★★★

4. There may be many other pest-control companies which can do even better than the two companies for us to choose from (F.D )

★★★

5. Other methods can also be applied to achieve the president?s purpose of saving money. (Necessity of the solution) ★★

——————————————————————————————

42, (社会,经济/提议/大类比小并列)The following appeared in a proposal from the economic minister of the country of Paraterra. per capita income in Bellegea increased by ten percent(4). To provide more income for the population of Paraterra and also preserve the natural environment of our tiny country(5), we too should begin to promote ecotourism(1,8). To ensure that our advertising campaign is successful, we should hire the current director of Bellegea's National Tourism Office(6) as a consultant for the campaign.(7)" ★★30

1. Bellegea and Paraterra might not be comparable in tourist resources, environment, etc. (F.A) ★★★★

2. No evidence could indicate that it is the promotion of ecotourism that lead to the increased number of visitors at Bellegea. (NCR)

★★★★

3. We do not know what fraction of visitors arriving at Bellegea came here for tourism. (V.D) ★★★

4. Other factors could also account for the increase in per capita income in Bellegea. (NCR) ★★★★

5. The minister does not provide any information about details of ecotourism, thus the assumption that the natural environment of

Paraterra will be preserved is not ensured. (I.I) ★★★

6. The success of ecotourism in Bellegea, if any, may have no causal relationship with the director?s effort. (NCR) ★★★★

7. Hiring the current director of Bellegea?s National Tourism Office as a consultant may not be the best solution to ensure our

success. (Necessity of the solution) ★★

8. The minister fails to consider the possible cost of carrying out ecotourism. (feasibility of the conclusion) ★★★

——————————————————————————————

43, (经营管理/决策/并列+类比)The following appeared as part of a business plan developed by the manager of the Rialto Theater. "Despite its downtown location, the Rialto Movie Theater, a local institution for five decades, must make big changes or close its doors forever. It should follow the example of the new Apex Theater in the mall outside of town(1). When the Apex opened last year, it featured a video arcade, plush carpeting and seats, and a state-of-the-art sound system(2,3). Furthermore, in a recent survey, over 85 percent of respondents 17

(4)reported that the high price of newly released movies prevents them from going to the movies more than five times per year. Thus, if the Rialto intends to hold on to its share of a decreasing pool of moviegoers, it must(6) offer the same features as Apex(5)." ★★★63

1. The manager offers no evidence that the two theaters are indeed comparable. (F.A) ★★★

2. We are not informed about the actual profit and the number of moviegoers of Apex since its opening. (I.I) ★★★★

3. The manager unfairly assumes that the success of Apex, if there is any, was result from those new features mentioned above.

(NCR) ★★★★★

4. The reliability of the survey on which the argument relies is not guaranteed. (are the respondents representative) ★

5. The manager fails to consider the possible cost of these fashionable features. (adv:disadv/feasibility of the conclusion) ★★★

6. Other solutions can also be used to achieve the manager?s goal. (Necessity of the solution) ★★★

——————————————————————————————

44, (经营/决策/大并列小递进)The following is a recommendation from the business manager of Monarch Books.

"Monarch Books should open a café in its store(5) to attract more customers(3) and better compete with Regal Books, which recently opened a café would clearly attract more customers. The café would require relatively little space. Space could be made for the café by discontinuing the children's book section(4), which will likely become less popular(8) given that the last national census(6) indicated a significant decline in the percent of the population(7) who are under age ten." ★★★

1. The manager offers no evidence that the two bookstores are comparable at every aspect. (F.A) ★★

2. We are not informed about the possible change in the profit and number of customers of Regal Books after Regal has opened the

new café. (I.I) ★★★★

3. No evidence could guarantee that opening a café at Monarch will attract substantially more customers.(I.E) ★★★

4. The manager fails to consider the negative effects of cutting out children?s book section and opening the café. (adv:disadv) ★★

★★★

5. The manager fails to consider the possible cost of opening the café. (adv:disadv) ★★★

6. The result of a national census may not be properly applied to a certain region. (C.S) ★★★★

7. Although the percent of children in the general population has declined, the total number of children may increase. (V.D) ★★

8. Granted that the number of children who are under age ten has decreased, we can not ensure that the number of children who go

to our bookstore, or the need for children?s book will also decline. (NCR) ★★★

——————————————————————————————

45, (动物/推测/并列)The following appeared as an editorial in a wildlife journal. populations are declining. Since these reports coincide with recent global warming trends(2) that have caused the sea ice to melt, we can conclude that the decline in arctic deer populations is the result of deer being unable to follow their age-old migration patterns(4) across the frozen sea(3)." ★★★★★

1. The accuracy of the reports from local hunters can be cast doubt on. (selective sample) ★★★★

2. The global warming trends may have no effects on the specific region mentioned by the editorial. (C.S) ★★★★

3. Alternative explanations can also account for the decline in arctic deer population.(NCR) ★★★★

4. The change in climate patterns does not necessarily lead to the decline in deer population, deer may adapt themselves to the new

weather by certain means. (NCR) ★★★★

——————————————————————————————

46, (动物/论断,推测/——)thousands of years, reveals that their language has words for two different kinds of bears, and their ancient legends attribute different 18

characteristics to the two kinds of bears(2,3). Therefore, there probably were grizzly bears in Labrador, and the explorer's account probably accurately identified the bear. ★★★★★

1. The reliability of the research on local language and legends is open to doubt. ( credibility of the evidence) ★★★★

2. Although the Innu has words for two kinds of bears, we still cannot ensure that one of them means grizzly bears. (I.E) ★★★★

3. The author does not provide any information about the habitat and living patterns of Innu thousand years ago, thus it is possible

that they use their word for grizzly bears to describe bears living in other regions. (I.I) ★★★

4. The 19th-century explorer may mistake black bear for grizzly bear under that certain circumstance. ( credibility of the evidence)

★★★

——————————————————————————————

47, (自然,考古/推测/递进)created a large dust cloud throughout Earth's atmosphere that would have been capable of blocking enough sunlight to lower global

temperatures significantly(2). A large meteorite collision, however, would probably create a sudden bright flash of light, and no extant historical records of the time(3) mention such a flash. Some surviving Asian historical records of the time, however, mention a loud boom(4) that would be consistent with a volcanic eruption. Therefore, the cooling was probably caused by a volcanic eruption(5). ★★★★★

1. The reliability of historical records is open to doubt. (credibility of the evidence) ★★★

2. Other factors can also contribute to the cold temperatures. (F.D) ★★★★

3. The fact that no records about sudden bright flash have been discovered does not indicate that there was no any meteorite

collision at that time. (I.E) ★★★★

4. The loud boom mentioned by Asian historical records could be created by reasons other than volcanic eruption. (I.T) ★★★

5. Granted that the loud boom mentioned by Asian historical records did result from volcanic eruption, the assumption that the

eruption would created enough amount of dust, and further lower global temperatures significantly is still open to doubt. (NCR) ★★★

——————————————————————————————

48, (健康,社会/论断,预言/大并列小递进)The following appeared in a newspaper article published in the country of Corpora. ownership are also highest(3), it is clear that using computers has not made citizens less physically fit. Instead, as shown by this year's unusually low expenditures on fitness-related products and services(4), the recent decline in the economy is most likely the cause(5), and fitness levels will improve when the economy does(6)." ★★★201

1. The standard for fitness may vary during past 20 years. ( P→F) ★★★★

2. Although the regions mentioned by the author have relatively highest fitness levels, it is still possible that their fitness levels are

declining compared with themselves. (confusing variation and comparison) ★★★★

3. High levels of computer ownership do not indicate that citizens of these regions will spend more time on using computers. (U.C)

★★★

4. The low expenditures on fitness-related products and services may not directly result in low level of fitness. (NCR) ★★★★

5. The low expenditures on fitness-related products do not necessarily resulted from the decline in the economy. (NCR) ★★★

6. The author fails to convince us that once the economy improves, people will spend more money on fitness-related products and

services, and their fitness levels will therefore be improved. (sufficiency of the conclusion) ★★★★

——————————————————————————————

49, (保健/论断,建议/先并列后递进)The following appeared in a letter to the editor of a newsletter on health issues. recent study, teenagers who avoided greasy foods for a month reported approximately as many outbreaks of acne and related skin conditions as 19

did those who ate an average of two servings of greasy food per day(4,5). Such data indicate that eating greasy foods is unlikely to be a cause of acne and related skin conditions. Therefore, health experts should no longer recommend that people(7) avoid such foods(6)." ★★★★34

1. The editor does not inform us what fraction of teenagers actually followed experts? suggestions during the past decade. (V.D/I.I) ★

★★★

2. Other reasons may also lead to the increase in the number of teenagers who suffer from skin problems. (NCR) ★★★★

3. The number of teenagers who sought medical help is not a good indication of actual incidence of acne. (U.C) ★★★

4. The editor does not provide any information about the number of teenagers in the two groups of the recent study. (V.D) ★★★

5. There might be some other substantial differences between the two groups of teenagers studied. (I.C) ★★★★

6. Eating greasy foods may have negative effects other than causing skin problems . (I.T) ★★★★

7. Avoiding greasy foods may be necessary for people of other ages. (C.S) ★★★★

——————————————————————————————

50, (自然/科学假说/递进)From a draft textbook manuscript submitted to a publisher.

"As Earth was being formed out of the collision of space rocks(1), the heat from those collisions and from the increasing gravitational energy of the planet made the entire planet molten, even the surface. Any water present would have evaporated(2) and gone off into space. As the planet have been retained in the atmosphere(4), eventually falling as rain on the cooled and solidified surface of Earth(5). Therefore, the water in Earth's oceans(3) must have originated from comets(6)." ★★★★★

1. The assumption that the Earth was formed out of the collision of space rocks is open to doubt. (U.A) ★★★

2. The assumption that all of the original water evaporated is unwarranted. (U.A) ★★★

3. The author falsely assumes that there were sufficient amount of comets striking the earth at the critical stage of ocean formation.

(U.A) ★★★★

4. The author?s assumptions that the Earth's surface had already cooled and solidified when strikes by comets occurred, and that the

heat generated during the striking was enough for all the comets to vaporize are unwarranted. (U.A) ★★★

5. No evidence could ensure that ice contained in comets could have formed the water in Earth?s oceans, it may also evaporated, or

diffused in the space. (U.A) ★★★★

6. The author fails to take into account other possible explanation for the origination of ocean water. (I.T) ★★★★

——————————————————————————————

51,(医疗/建议/对比)The following appeared in a medical newsletter. hypothesis has now been proved by preliminary results of a study of two groups of patients. The first group of patients, all being treated for muscle injuries by Dr. Newland, a doctor who specializes in sports medicine, took antibiotics regularly throughout their treatment. Their recuperation time was, on average, 40 percent quicker than typically expected. Patients in the second group, all being treated by Dr. Alton, a general physician, were given sugar pills(3), although the patients believed they were taking antibiotics. Their average recuperation time was not significantly reduced(1,2). Therefore, all patients(4) who are diagnosed with muscle strain would be well advised to take antibiotics(5) as part of their treatment." ★★★★

1. The author does not inform us about the severity of injuries, physical conditions of the two groups of patients. (I.C) ★★★★

2. There might be other differences between treatments offered by the two doctors.(I.C) ★★★★

3. Sugar pills may cause negative effects on the healing of the second group of patients. (I.T) ★★

4. Not all patients who are diagnosed with muscle strain will suffer from secondary infections. (C.S) ★★★★

5. Not all patients with muscle strain are suitable to taking antibiotics. (feasibility of the conclusion) ★★★★

——————————————————————————————

52, (人事,就业/提议/大并列小递进)The following appeared in a memo to the human resources manager at Baobob Inc., a large architectural firm.

"Several well-known, retired architects(1,2) were interviewed in Architecture Today about changes in the field. Only one had earned a college degree in architecture. All others had come into the field at an early age by serving apprenticeships that required them to work under the direct supervision of an experienced architect. Several of the colleges that we recruit from report that many(3) promising architecture students leave school(4) early in their undergraduate career(6). Therefore, because finding talented architecture graduates is becoming more difficult(5), 20

Baobob Inc. should start an aggressive apprenticeship program(8) and hire students who express an interest in architecture directly out of high school(7) rather than wait for them to get out of college(9)." ★★

1. The architects interviewed might not be representative of all architects. (are the respondents representative?) ★★★★

2. The conditions of those retired architects may not be properly applied to current situation. (P→F) ★★★★★

3. We need to know what fraction of promising architecture students actually leave school. (V.D) ★★★

4. We do not know the actual reasons of those students for leaving school, and what kind of jobs did they take after leaving. (I.I)

★★★

5. The firm can still recruit from colleges where their promising architecture students do not leave. (I.T) ★★★★

6. Students who are thought to be promising at the early stage of undergraduate career may become average students when they

graduate, or we can still find enough talented graduates from students who stayed. (I.T) ★★★★

7. No evidence could imply that we should hire students directly from high school. (I.E) ★★

8. The manager fails to consider the cost and possible negative effects of setting up such an apprenticeship program. (adv:disadv) ★

★★★

9. Other possible measures can be adopted to achieve the manager?s purpose. (necessity of the solution) ★★★

——————————————————————————————

53, (生命科学/科学假说/并列)Thirteen years ago, researchers studied a group of 25 infants(1) who showed signs of mild distress(4) when exposed to unfamiliar stimuli such as an unusual odor or a tape recording of an unknown voice. They discovered that these infants were more likely than other infants to have been conceived in early autumn(2), a time when their mothers' production of melatonin(3)—a hormone known to affect some brain functions—would naturally increase in response to decreased daylight. In a follow-up study conducted earlier this year, more than half of these children—now teenagers—who had shown signs of distress identified themselves as shy(5). Clearly, increased levels of melatonin before birth cause shyness(4) during infancy and this shyness continues into later life(6). ★★★★

1. The sample of the mentioned study is too limited. (quantity of the sample) ★★★

2. No direct evidence is provided to show that it is the high levels of mothers? production of Melatonin that resulted in the distress

showed by the infants studied. (confusing concurrence with causality) ★★★★

3. The author fails to prove that increased levels of mothers? production of Melatonin will directly affect their infants. (U.A) ★★

★★

4. The author simply equates mild distress with shyness. (U.C) ★★★★

5. Different individuals may define the term shy in various ways. (definition of the term ?shy?) ★★★★

6. Many other genetic or environmental factors will cause shyness in those teenagers. (I.T) ★★★★

——————————————————————————————

54, (保健/建议/大并列小对比)The following appeared in a newsletter on dental health.

"A recent research study reported the experience of dentists whose patients had, over a period of five years(2), regularly used Flux Dental Floss as part of their dental hygiene routine. The report indicates that these dentists had 50 percent fewer cases of gum disease(1,5) than did dentists whose patients did not use Flux regularly. In addition, most of the Flux users(3) who were surveyed by their dentists agreed that Flux's mint flavor would encourage people to floss more often(4). Thus, even though Flux may cost more than other brands of floss, it is clearly a worthwhile investment(7) for those who want to be assured of healthy teeth(5) and gums(6)." ★★★

1. Other causes may explain the fewer cases of gum disease of the patients who use Flux regularly. (NCR) ★★★★

2. The period of the survey is not long enough, thus there may be negative effects of Flux on gum health in the long run. (I.T) ★★★

3. People who favors mint flavor are more likely to choose Flux, (are the respondents representative?) or other people may not

favor mint flavor as much as those Flux users. (H.G) ★★★★

4. Other brands of floss may also have mint flavor. (I.I) ★★★

5. No evidence concerning the effect of Flux on teeth health is provided.(I.E) ★★★

6. Other brands of floss may have similar effects on gum health as Flux has. (necessity of the solution) ★★★

7. The author does not provide the actual price of Flux, thus we cannot evaluate if it is worthy to use Flux. (I.I) ★★

——————————————————————————————

55, (道路交通/论断,提议/大并列小类比)The following appeared in a Letter to the Editor of the Shady Village newspaper. 21

apparently attracted more commuters, judging from the fact that rush-hour traffic jams actually increased there this past winter(1,2). To reduce rush-hour traffic on Blue Highway, a bicycle lane should be added instead of a traffic lane(3). This approach will succeed because many citizens of Shady Village are avid bicyclists(4); 75 percent of respondents(5) to a recent questionnaire distributed there said they would like to bicycle more hours per week(6) than they currently do." ★★18,119

1. The increased traffic jams at Green Highway may result from other causes. (NCR) ★★★★

2. Problems emerged on Green Highway would not necessarily also take place on Blue Highway. (F.A) ★★★★

3. The author fails to consider possible negative effects of building a bicycle lane. (adv:disadv) ★★★★

4. We need to know how many of the avid bicyclists in Shady Village are using Blue Highway for commuting, and what percentage of

the whole citizens here are avid bicyclists. (V.D) ★★★★

5. People who are interested in bicycling are more likely to respond to the questionnaire. (are the respondents representative?) ★

★★★

6. The result of the recent survey does not indicate that those respondents will use bicycles for daily commuting. (U.C) ★★★ ——————————————————————————————

56, (艺术,考古/推测,预言/递进)of human heads and hands on Kali(1), we can now conclude that the ancient Kalinese artists used molds of actual bodies, not sculpting tools and techniques, to create these statues(2,3). This discovery explains why Kalinese miniature statues were abstract and entirely different in style: molds could only be used for life-size sculptures(4). It also explains why few ancient Kalinese sculpting tools have been found(5). In light of this development, collectors should expect the life-size sculptures to decrease in value and the miniatures to increase in value(6). ★★★★★

1. The recently discovered molds of human heads and hands are not necessarily used by Kalinese artists for sculpting. (U.A) ★★★

2. Granted those molds were used for sculpture purpose, we cannot hastily conclude that Kalinese artist also use molds to create all

life-size statues. (H.G) ★★★★

3. No sufficient evidence is provided to prove that Kalinese artists did not use any sculpting tools and techniques to create life-size

statues. (I.E) ★★★

4. Other possible factors would also lead to the differences between miniature statues and life-size statures. (F.A) ★★★★

5. Alternative explanation could also explain why few ancient Kalinese sculpting tools have been found. (NCR) ★★★

6. The author?s assumption that the life-size sculptures will decrease in value and the miniatures to increase in value is unwarranted.

(U.A) ★★★★

——————————————————————————————

57, (医药/论断,预言/对比)The following appeared in a newsletter on nutrition and health.

"Although the multimineral Zorba pill was designed as a simple dietary supplement, a study of first-time ulcer patients who took Zorba suggests that Zorba actually helps prevent ulcers. The study showed that only 25 percent of those ulcer patients who took Zorba(2) under a doctor's direction developed new ulcers, compared to a 75 percent recurrence rate among ulcer patients who did not take Zorba(1). Clearly, then, Zorba will be highly effective in preventing recurrent ulcers and if health experts inform the general public of this fact, many first-time ulcers(3) can be prevented as well." ★★★★

1. There may be many other differences between patients who took Zorba and who do not. (I.C/ex parte information) ★★★★★

2. The patients who took Zorba mentioned by the study may not be representative of the whole group of patients who took Zorba. (are

the respondents representative?) ★★★

3. No evidence about the effect of Zorba on preventing first-time ulcers is provided. (I.E) ★★★★

——————————————————————————————

58, (考古/论断/并列)The following appeared in an article from a popular newsmagazine. Scandinavians were not, however, exploring or emigrating to northern Wisconsin in the fourteenth century(1). Recent analysis proves, in fact, that the stone had been buried in the spot where it was found for no more than 100 years(2). Moreover, the community near the discovery site was home to a group of people who had formed a club to study medieval Scandinavian culture—a period that includes the fourteenth century. The stone, therefore, is not a genuine artifact of medieval Scandinavian culture inscribed in the fourteenth century but most likely a hoax perpetrated by the group(3)." ★★★★★

22

1. The fact that Scandinavians did not emigrate to northern Wisconsin does not necessarily illustrate that this stone was not produced

by Scandinavians, the stone could be brought here by some other people. (I.T) ★★★★★

2. The time when the stone was buried is not necessarily identical with the time it was produced. (U.C) ★★★★★

3. No evidence could actually demonstrate that the stone was forged by the group. (I.E) ★★★★

——————————————————————————————

59, (保健/建议/——)The following appeared in an article in the health section of a newspaper.

"According to the available medical records, the six worst worldwide flu epidemics(2) during the past 300 years occurred in 1729, 1830, 1918, 1957, 1968, and 1977. These were all years with heavy sunspot activity(3)—that is, years when the Earth received significantly more solar energy than in normal years(1). People at particular risk for the flu should therefore avoid prolonged exposure to the Sun(4,5,6)."

★★★★★

1. The writer provides no direct evidence to prove that it is the solar energy that resulted in the flu epidemics. (confusing concurrence

with causality) ★★★★★

2. The writer fails to inform us about the severity of the disease in years when sunspot activities were less heavy. (lack of controlled

experiment) ★★★★

3. Granted that heavy sunspot activity did result in the flu epidemics, exposure to the Sun in normal years may not have the same

effect. (H.G) ★★★

4. Exposure to the sun may have some positive effects for human health, thus we could not hastily conclude that people should

avoid it. (I.T) ★★★

5. Suffering from flu might be a result caused by multiple factors, thus adopting the writer?s suggestion alone may not prevent flu

sufficiently. (sufficiency of the solution) ★★

6. Other methods could also be used to prevent flu. (necessity of the solution) ★★

——————————————————————————————

60, (保健/提议/大递进小对比)The following appeared in the health section of Glenntown's local newspaper.

"Several national(4) medical studies suggest that older people who have pets tend to enjoy better health than those who do not have pets: those who have pets have lower rates of high blood pressure and arthritis(1,2). It seems clear that having to care for an animal promotes good health for the older person. Therefore, Glenntown should establish a program to give a small pet such as a dog or cat(3) to all(5) of its citizens who are over the age of 65. This will help to insure that our senior citizens enjoy good health and have fewer medical bills(6)."

★★114,129

1. No causal relationship between having pets and low rates of high blood pressure and arthritis among those older people mentioned

in the studies was established. (confusing concurrence with causality) ★★★★

2. There may be other differences between people who have pets and those who do not. (I.C) ★★★★

3. Pets could cause other health problems. (adv:disadv) ★★★★

4. The result of national studies does not necessarily apply to the situation in Glenntown. (C.S) ★★★

5. Not all senior citizens love small animals, not all citizens are capable of having pets, or could afford raising a pet. (C.S) ★★★★ The author does not provide any information about the composition of medical expenditure in Glenntown, if the medical bills were mainly paid for diseases other than high blood pressure and arthritis, the bills would not necessarily decrease. (sufficiency of the solution) ★★★★

61, (教育/论断/递进)The following appeared in a report by the School District of Eyleria. schools, the ratio of computers to students is 1:7. This number is sufficient to ensure that all of Eyleria's students, by the time they graduate from high school, will be fully proficient in the use of computer technology(2). Thus, there is no reason to spend any of the schools' budget on computers or other technology(4) in the next few years(3)." ★★★

1. The assumption that every student in every school across the country has access to computers is not guaranteed. (H.G) ★

2. No evidence could illustrate that the 1:7 ratio at Eyleria schools could sufficiently ensure computer proficiency of all Elyeria?s

students. (I.E) ★★★★

3. The author fails to consider many factors that would increase the demand for computers in the future. (C→F) ★★★★

4. The author hastily assumes that education on other technology is also not necessary in our education program.(I.E) ★★★ 23

————————————————————————————-

62, (金融流通/论断/并列)introduced 11 years ago, nor the Eagle dollar coin, introduced 3 years ago, remained in circulation long: people simply preferred spending paper dollars(1). However, a survey conducted soon after its release(2) indicates that the Pine Tree dollar is far more appealing than other dollar coins: 76 percent of respondents(3) own the coin; 62 percent of those owning multiple coins consider it so attractive that they are still saving rather than spending the most recent coin to come into their possession(4). Since the Pine Tree coin promises to become the dollar currency of choice, the Treasury is correct to begin saving money(7) by dramatically reducing the costly printing of paper dollars(5). ★★★

1. Other reasons could also explain the short circulation period of the two former dollar coins. (NCR) ★★

2. The result of a survey conducted right after the coin?s release could not be properly applied to future condition.(when was the survey

conducted?/ C→F) ★★★★★

3. People who favor the Pine Tree dollar might be more likely to respond to the survey. (are the respondents representative?) ★★

★★

4. The fact that many people prefer saving the new coin would be a check on the new coin?s circulation. (negative evidence) ★★

5. Many people would still prefer using paper dollars although the new dollar coin is popular. (U.A) ★★★

6. The author does not provide information about the cost of the production of dollar coins, thus the author?s assumption that the

Treasury will necessarily save money is open to doubt. (adv:disadv) ★★★★

——————————————————————————————

63, (社会/对策/but it is no longer heavily used. Video cameras mounted in the park's parking lots last month(2) revealed the park's drop in popularity: the recordings showed an average of only 50 cars per day(1,3). In contrast, tiny Carlton Park in the heart of the business district is visited by more than 150 people on a typical weekday(4). An obvious difference is that Carlton Park, unlike Stanley Park, provides ample seating(5). Thus, if Stanley Park is ever to be as popular with our citizens as is Carlton Park, the town will obviously need to provide more benches, thereby converting some of the unused open areas into spaces suitable for socializing(6). ★★★43

1. The number of cars at the park?s parking lots may not be a good indication of the park?s popularity. (U.C) ★★★★

2. There may be some special reasons last month that resulted in the dropped popularity of Stanley Park. (selective sample) ★★★★

3. The author does not provide information concerning the number of visitors at Stanley before the cameras were mounted.

(confusing comparison with variation) ★★★

4. Many other differences may render the two parks not comparable. (I.C) ★★★

5. The author fails to convince us that it is the ample seating that makes Carlton so popular. (NCR) ★★★★

6. The author fails to consider the possible negative effects of converting unused open areas into public social spaces. (adv:disadv)

★★★

——————————————————————————————

64, (教育/决策/并列)The following appeared in a memo from the Dean of the College of Education at Omni State University.

"Only seven of our state's high schools offer even one course in Latin, but over 80 percent(2) of the graduates who study Latin at those seven schools enroll in college(1,4). Furthermore, a recent study shows that students who have mastered Latin perform much better in logic and critical thinking(3). Thus, in order to increase the percentage of our high school students who graduate from college(5) as well as to ensure that they receive a better education in critical thinking, Omni State University should begin an intensive program to prepare our future teachers to teach high school Latin(6,7)." ★★★

1. The Dean fails to establish a causal relationship between offering Latin courses and high rate of enrollment in college at the seven

high schools. (NCR) ★★★★

2. We do not know the percentage of students who enroll in college in other high schools. (I.C/ex parte information) ★★★

3. Other factors may also explain why the students who mastered Latin also perform better in logic and critical thinking. (NCR) ★★

★★

4. The Dean does not provide any information concerning the performance of students from the seven high schools in their college

stage and workplace. (I.I) ★★

5. No evidence could justify that offering Latin courses in high schools will increase the percentage of high school students who

graduate from college. (U.A) ★★★

24

6. No evidence is provided to show that our state lacks high school Latin teachers. (necessity of the solution) ★★★

7. The Dean fails to consider if high schools in our state have the demand or capability of offering Latin courses. (feasibility of the

conclusion) ★★★

——————————————————————————————

65, (商业/决策/并列)The following appeared in a memo from the president of a chain of cheese stores located throughout the United States. best-selling cheeses at our newest store(2) were all domestic cheddar cheeses from Wisconsin. Furthermore, a recent survey by Cheeses of the World magazine indicates an increasing preference for domestic cheeses among its subscribers(3). Since our company can reduce expenses by limiting inventory, the best way(4) to improve profits(6) in all of our stores is to discontinue stocking many of our varieties of imported cheese(5) and concentrate primarily on domestic cheeses." ★★★

1. The situation last year may be unusual, it does not indicate that domestic cheddar cheeses will also be popular in the future. (P→F)

★★★

2. The situation at the newest store might not be representative of all stores. (selective sample/C.S) ★★★★

3. The president does not provide any information about the subscribers of Cheese of the World, thus they may not be representative of

United States consumers. (are the respondents representative?) ★★★★

4. Many other better ways might be available to improve profits of our stores. (necessity of the solution) ★★★

5. The president fails to consider the negative effects of discontinuing the inventory of imported cheeses. (adv:disadv) ★★★★

6. The president does not provide any information about the actual profit of domestic and imported cheese respectively, thus we

cannot evaluate the president?s conclusion. (I.I) ★★★★★

——————————————————————————————

66, (生活,经济/预言,建议/并列)The following appeared in a letter from a firm providing investment advice to a client. that region experienced 90 days with below-average temperatures, and climate forecasters at Waymarsh University(1) predict that this weather pattern will continue for several more years. Furthermore, many new homes have been built in this region during the past year(2,4). Because of these developments, we predict an increased demand for heating oil(3) and recommend investment in Consolidated Industries(5), one of whose major business operations is the retail sale of home heating oil." ★★★★15

1. The accuracy of Waymarsh University?s prediction is open to doubt. (credibility of the evidence) ★★★

2. The author does not provide any information about the actual amount of heating oil used due to the cold weather last year, thus we

cannot evaluate the argument. (I.I) ★★★★

3. Other new heating fuel may be available currently. (I.T) ★★★

4. No evidence could guarantee that new homes in this region will continue to use heating oil as fuel for heating. (U.A/ P→F) ★★★

5. Granted that the demand for heating oil will increase, we cannot guarantee that investing in Consolidated Industries will be

profitable. (H.G) ★★★★

——————————————————————————————

67, (城市/提议/大并列小类比)The following appeared in a letter to the editor of a newspaper serving the villages of Castorville and Polluxton. money and improve service, the two villages recently merged their once separate garbage collection departments into a single department located in Castorville, and the new department has reported few complaints about its service(2). Last year the library in Polluxton had 20 percent fewer users than during the previous year(3). It follows that we should now further economize and improve service(5), as we did with garbage collection(4), by closing the library in Polluxton(6) and using the library in Castorville to serve both villages(7)." ★★★ 192

1. The number of residents who pay property taxes does not necessarily indicate the actual number of residents in the two villages, and

the actual amount of taxes collected by the two villages. (U.C) ★★★

2. Few complaints about new department?s service do not indicate that citizens are satisfied with its service. (I.E) ★★★★

3. The situation in Polluxton?s library may not be typical. (selective sample/ P→F) ★★★

4. Garbage collection service and library are not comparable. (F.A) ★★★★★

5. No evidence could guarantee that emergence of the two libraries could improve service. (U.A) ★★★

25

6. The author does not provide any information about the number of users at the two libraries, thus we cannot evaluate whether we

should close the library in Polluxton if the emergence is indeed necessary. (I.E) ★★★

7. The author fails to consider the cost and other possible negative effects of merging the two libraries. (adv:disadv) ★★★ ——————————————————————————————

68, (社会/提议/先并列后递进)The following appeared in a memo from a budget planner for the City of Grandview. Symphony's concerts-in-the-park series doubled(2,3). The Symphony has also announced an increase in ticket prices for next year(4). Such developments indicate that the Symphony can now succeed without funding from city government and we can eliminate that expense from next year's budget. This action will surely prevent a budget deficit(5)." ★★★

1. We do not know the base amount of private contributions to the Symphony the year before last, thus we cannot evaluate if the

contributions were significant. (V.D) ★★★★

2. The fact that attendance at the concert series doubled does not necessarily indicate that the Symphony is operating successfully. (U.C)

★★★

3. The facts cited by the planner, including the increased private contributions, popularity of concert series, may not recur in the

following years. (P→F) ★★★★

4. The planner does not inform us how much will the ticket prices actually increase. (I.I) ★★

5. Eliminating the funding for the Symphony alone may not prevent a budget deficit sufficiently. (sufficiency of the solution) ★★★

6. Other measures could also be taken to prevent a budget deficit. (necessity of a solution) ★★★

——————————————————————————————

69, (动物/论断,提议/并列)moving several miles over land, removing trout from just a few lakes is clearly the way to restore the frog population to its former levels(4,5). ★★★★★

1. Many other factors would also lead to the decline in frog population. (I.T) ★★★★

2. No direct evidence is provided to prove that it is the trout that resulted in the decline in frog population. (I.E/NCR) ★★★★

3. We cannot ensure that frog population in other lakes will also increase after trout were removed (quantity of the sample/C.S) ★★

4. Removing trout from just a few lakes may not sufficiently ensure restoration of frog population to former levels. (sufficiency of the

solution) ★★★★

5. The author fails to consider the cost and negative effects of removing trout from the lakes. (adv:disadv) ★★★★

——————————————————————————————

70, (交通安全/论断/递进)Studies show that in 70 percent of traffic accidents(6), at least one driver involved is less than 10 miles from home(1,2) when the accident occurs. This statistic indicates that drivers have a tendency to drive incautiously(3) when they are close to home, probably because familiar surroundings(4) give them a false sense of security(5). Thus, the places(7) where people feel safest are the places where they are in fact at greatest risk of serious injury. ★★★★★

1. The author fails to provide any data concerning average distances of drivers from their homes in driving. (V.D) ★★★★★

2. The author fails to illustrate that the driver near home is mainly responsible for those accidents. (I.I) ★★★★

3. No direct evidence is provided to prove that it is the incautious driving that resulted in those accidents. (NCR) ★★★

4. The author fails to prove that drivers involved in those accidents do feel familiar with surroundings at the spot where accidents were

taken place. (I.E) ★★

5. Granted that people do drive incautiously close to home, we cannot guarantee that it is the false sense of security that resulted in the

accidents. (NCR) ★★★

6. The author does not provide any information about the severity of accidents. (I.I) ★★★

26

7. We cannot hastily generalize from the facts cited above that most situations where people feel safe are risky. (C.S) ★★★★ ——————————————————————————————

71, (工业/预言/electricity(3) than the older method to process the same amount of raw ore, especially when the proportion of copper in the ore is high(1,2). Therefore, we can expect the amount of electricity used by the copper-extraction industry to decline significantly(4,5,6). ★★★★★

1. The author fails to provide information regarding the efficiency of the new method when the proportion of copper in the ore is low.

(I.I ) ★★★★

2. The author fails to inform us about the fraction of ore in which the proportion of copper in the ore is low. (V.D) ★★★★

3. The author fails to consider other expenditures of the new technology. (adv:disadv) ★★★★

4. There may be some obstacles in the application of the new technology, thus the new method might not be practically feasible at

current stage. (feasibility of the solution) ★★★

5. We need to know what fraction of copper companies would be interested in the new technology, or are capable of using the new

method. (I.I) ★★★

6. The author fails to consider other factors that will lead to the increase in the amount of electricity used by the copper-extraction

industry. (I.T) ★★★★

——————————————————————————————

72, (生命科学,医疗/预言/对比)In measuring electrical activity in different parts of the brain, researchers found that people who describe themselves as generally happy(1) have more activity in the left prefrontal lobe of their brains than do other people(2). Therefore, a medication for stimulating(3) the left prefrontal lobe of the brain(5) would be an effective treatment for clinical depression(4). ★★★★

1. People may have misunderstanding about their level of happiness. (credibility of the evidence) ★★★★

2. The author falsely assumes that it is the activity in the left prefrontal lobe of brains that resulted in the happiness of those people.

(confusing concurrence with causality/confusing the cause and the effect) ★★★★★

3. There may be some differences between external stimulating and natural activity of brains. (F.A) ★★★★★

4. We are not informed about the accurate definition, actual cause and detailed symptoms of clinical depression, thus the evidence cited

by the author may have nothing to do with the conclusion. (definition of the term ?clinical depression?/U.C) ★★★★

5. The author fails to consider the negative effect of the proposed therapy. (adv:disadv) ★★★★

——————————————————————————————

73, (商业/决策/先并列后递进)The following appeared in a memo from a manager of a car dealership. that the majority of drivers will be driving leased, not individually owned cars. Therefore, we should change the focus of our business from selling cars to leasing them(4,6)." ★★★★

1. The fact that more people leasing new cars does not necessarily indicate that the demand for new cars is diminishing. (I.E) ★★★

2. The manager fails to provide the percentage increase in the number of people who lease cars and who buy new cars. (V.D) ★★★

3. The manager unfairly assumes that the situation in the past years will certainly continue in the following years. (P→F) ★★★

4. The manager fails to provide the actual cost and profit of leasing and selling cars, respectively. (I.I) ★★★★

5. The manager fails to rule out the possibility that many consumers will buy new cars after six years use of their former cars. (I.T) ★

★★★

6. The company may have advantage, or could still earn significant profits by selling new cars. (I.T) ★★★★

——————————————————————————————

74, (教育,就业/论断/递进)The following appeared in a newsletter about education policy. But the availability of such programs would probably encourage some students who would otherwise have graduated to drop out in order to 27

pursue vocational training(1). Hence the proposed programs would lower, not raise, the average earning potential of young people(2). Therefore, the state should not establish the vocational training programs(3) but should instead focus on improving academic programs(4,5) in our high schools." ★★★★

1. The author?s assumption that a substantial number of students will drop out to pursue vocational training is unwarranted. (U.A) ★

★★★

2. No evidence is provided to show that the proposed programs would not raise the earning potential of young people. (U.A) ★★★

3. The mere evidence cited in the argument could not sufficiently prove that the proposed program is not necessary. (I.E) ★★★★

4. The author does not provide any evidence to show that our academic programs need to be improved. (necessity of the solution) ★

★★

5. The author fails to convince us that improving academic programs will necessarily decrease the rate of dropouts. (NCR) ★★★ ——————————————————————————————

75, (健康,社会/论断/大并列小对比)The following appeared in a newsletter about health. increasing since the 1970's(2), and during that same time period, the proportion of children living in suburbs has increased(1). Whereas children in rural areas tend to have outdoor active chores and children in cities often walk to school or other places they need to go, children in suburbs are typically driven everywhere. Thus the increase in childhood obesity is probably due mainly to the suburbanization of America and the associated decrease in the opportunities children have for exercise(3,4)." ★★★★

1. The author fails to establish a causal relationship between living in suburbs and obesity of children. (confusing concurrence with

causality) ★★★★★

2. The author does not provide the obesity rates among children who live in suburbs, rural areas and cities, respectively. (lack of

comparison) ★★★★

3. No evidence is provided to show that children in suburbs have no sufficient opportunities for exercise. (U.A) ★★★

4. The author unfairly assumes that the lack of exercise will necessarily lead to obesity. (U.A) ★★★

——————————————————————————————

76, (保健/论断/——)The following appeared as part of an article in a health and beauty magazine.

"A group of volunteers(1) participated in a study of consumer responses to the new Luxess face cream. Every morning for a month, they washed their faces with mild soap(2) and then applied Luxess. At the end of that month, most volunteers reported a marked improvement in the way their skin looked and felt(3). Thus it appears that Luxess is truly effective in improving the condition of facial skin(4)." ★★★

1. The volunteers may not be representative of all consumers using Luxess. (are the respondents representative?) ★★★

2. The author fails to rule out the possibility that it is the mild soap that actually improved the condition of the volunteers? facial skin.

(NCR/lack of controlled experiment) ★★★★

3. The volunteers may have misunderstanding about the condition of their facial skin. (credibility of the evidence) ★★★★

4. The author fails to consider possible negative effects of Luxess. (adv:disadv) ★★★

——————————————————————————————

77, (城市,建筑/论断/higher(2) in East Sacunda than in West Sacunda(4), however, so we can conclude that building owners in East Sacunda were less likely to modify their buildings so as to bring them up to the 1985 code standards(6). ★★

1. The level of stringency of the codes in the two cities might be different. (I.C) ★★★★

2. The author fails to provide the total amount of residents in the two cities respectively. (V.D/I.C) ★★★★★

3. The author fails to provide information regarding the severity of earthquake at the two cities. (I.I/I.C) ★★★★★

4. We need to know what fraction of buildings destroyed in East Sacunda were built before 1985 and were not modified to the 1985

codes. (I.I) ★★★★

5. The number of homeless people is not a good indication of the number of buildings destroyed. (U.C) ★★★★

6. Other factors could also explain the differences between the situations of the two cities. (NCR) ★★★

——————————————————————————————

28

78, (自然环境、考古/推测/并列)The following appeared in a letter to the editor of an archaeological magazine. old, were from a deer species whose modern-day descendants are known to prefer woodland habitats, whereas most of the bones at more recent levels, dating from 30,000 to 10,000 years ago, were from a gazelle species whose modern-day descendants(2) are known to prefer grasslands(3). We can thus conclude that the climate of the area changed dramatically between 40,000 and 30,000 years ago(4), causing the terrain to change from woodland to grassland." ★★★★★

1. The author fails to rule out the possibility that the animals whose bones were found here in the cave were hunted or brought here

from other sites. (I.T) ★★★★★

2. The author unfairly assumes that the ancient deer and gazelle species share the same habitats with their present descendants. (C→P)

★★★★★

3. Certain species of animal may live in various terrain, they do not necessarily all prefer one kind of natural environment. (U.A) ★★

4. Factors other than climate changes could also lead to the changes of terrain in this area. (NCR) ★★★

——————————————————————————————

79, (交通安全/建议/并列)The following appeared in a magazine for the trucking industry. a significant pay increase to its drivers and increased its training standards. It also put strict limits on the number of hours per week each driver could drive. The following year, its trucks were involved in half the number(2) of accidents as before the changes were implemented(1). A survey of other trucking companies found that the highest-paid drivers were the least likely to have had an accident(3). Therefore, trucking companies wishing to reduce their accident rate can do so simply by raising their drivers' pay and limiting the overall number of hours they drive(4,5)." ★★★

1. Other possible factors could also lead to the decline in the number of accidents of Longhaul. (NCR) ★★★★

2. The author fails to provide the annual accident rate of Longhual after the changes were implemented. (or) The author does not

inform us the total mileage driven at Longhaul in that year. (V.D/I.I) ★★★★★

3. The author fails to rule out the possibility that those highest-paid drivers earn high salary just because they are safe drivers.

(confusing the cause and the effect) ★★★★★

4. Adopting the author?s proposal alone may not sufficiently reduce the accident rates. (sufficiency of the solution) ★★★

5. Other methods could also be used to achieve the same purpose. (necessity of the solution) ★★★

——————————————————————————————

80, (公共卫生,社区/建议/并列)The following appeared as an editorial in a health magazine. linked cooking with water containing clormium 5 to an increased incidence of allergies and skin rashes(1). Tests of the drinking water(2) in

several areas(3) have revealed the presence of clormium 5. Although it is possible to remove clormium 5 from water, the costs of routine testing and purification are higher than many communities can afford. Therefore, in order to prevent allergies and skin rashes(4), communities that cannot afford to rid their drinking water of clormium 5 should replace drinking fountains in public buildings, such as schools and libraries, with bottled-water coolers(5,6,7)." ★★★

1. The editorial fails to establish a causal relationship between the presence of C5 and high incidence of allergies and skin rashes.

(NCR) ★★★★★

2. The high incidence of allergies and rashes was only linked with cooking with water containing C5, no evidence could guarantee that

drinking water containing C5 would also lead to the same result. (U.C) ★★

3. The result of the test might not be representative of the whole region, in other words, maybe drinking water in most communities

does not contain C5. (selective sample/C.S) ★★★★

4. Allergies and skin rashes might be cause by other reasons, thus merely removing C5 might not sufficiently prevent allergies and

rashes. (sufficiency of the solution) ★★★

5. Granted that removing C5 is a necessary measure to prevent allergies and skin rashes, merely replacing drinking fountains with

bottled-water coolers might not be sufficient. (sufficiency of the solution) ★★

6. Other methods could also be applied to prevent these disease. (necessity of the solution) ★★★

29

7. There may be some negative effects of using bottled-water coolers. (adv:disadv) ★★★★

——————————————————————————————

81, (卫生保健/建议/对比)supports the second view: children who are washed especially frequently and whose parents clean their homes especially frequently(1) are more likely to develop allergies than are other children(2,4). So in order to reduce the incidence of allergies in children and adults, parents should not limit(5) children's exposure to irritants or bacteria. ★★★★

1.

2.

3.

4.

5. The author simply equates frequent wash and home cleaning with excessive hygiene. (U.C/definition of the term ?excessive hygiene?) ★★★★★ Other factors could also result in the higher incidents of allergies in children who are frequently washed and whose homes are frequently cleaned. (NCR) ★★★★ No direct evidence is provided to demonstrate that exposure to bacteria does lead to proper development of the immune system. (I.E) ★★★ The author fails to provide proper information about the incidence of allergies of the two groups of children when they grow up. (I.I/I.C) ★★★★ Limiting children?s exposure to dangerous irritants or bacteria to some extent would still be necessary. (H.G) ★★★★

——————————————————————————————

82, (卫生保健,环境,经济/论断,预言/对比)The following appeared in a letter to an editor.

"In many countries, wood is the primary fuel used for heating and cooking(1), but wood smoke can cause respiratory and eye problems, and extensive use of wood causes deforestation, a major environmental problem. In contrast, charcoal, made by partially burning wood(3) in a controlled process, is a fuel that creates less smoke than wood does(2). Moreover, although charcoal costs slightly more than wood, less charcoal is needed to produce the same amount of heat. Therefore, people who use wood as their primary fuel can, without experiencing economic hardship(4), switch to charcoal and can thereby improve their health and preserve the environment(5,6,7)." ★★★★

1.

2.

3.

4.

5.

6.

7. The author fails to inform us that how many countries are actually using wood as heating and cooking fuel extensively. (I.I) ★ The author fails to inform us how much more smoke will be created by burning wood than by burning charcoal to acquire same amount of heat. (I.C) ★★ The author fails to inform us that how much wood would be used to make enough amount of charcoal which can produce the same amount of heat as wood.(I.I) ★★★★ Other expenditures of producing and using charcoal might be very expensive. (adv:disadv) ★★★★ The author fails to consider the pollution and other negative effects of producing charcoal. (adv:disadv) ★★★★ Switching to charcoal may not achieve the author?s purpose sufficiently. (sufficiency of the solution) ★★★ Other effective fuels which have more advantages might be available. (F.D) ★★★★

——————————————————————————————

83, (医药,商业/决策/——)The following appeared in a memo from the business manager of Medicine, Inc., a local drugstore.

"Most business analysts for the drugstore industry have stated that even when a nation's economy is weak, drugstores' profits are unlikely to decline appreciably. While consumers might put off some kinds of purchases when the economy is slow, prescription and over-the-counter drug purchases are dictated by consumers' health needs, which are independent of the economy(1). Therefore, Medicine Inc. (2)is likely to continue to have increasing profits and should plan to open an additional drugstore next year(3,4)." ★★★★

1. The author?s assumption that the profits of drugstores are totally independent of the economy is unwarranted. (U.A) ★★★★

2. The general principle about the business does not necessarily apply to Medicine, Inc. (C.S) ★★★★★

3. The manager fails to provide any data concerning current profitability and performance of Medicine, Inc. (I.I) ★★★★

4. The manager fails to illustrate that it is necessary and feasible for Medicine, Inc to open a new drugstore. (necessity of the

solution) ★★★

——————————————————————————————

84, (保健,饮食/决策/先递进后并列)The following appeared in a memo written by the head of the Gorham School.

30

"Our school cafeteria should make changes in the lunches that it serves in order to improve the health of our students. Several teachers and I have observed that students(3) who eat these meals tend to eat the main course and the dessert, but seldom finish the vegetable portion. This pattern means that students are missing the vital nutritional benefits of fruits and vegetables(2), which are typically rich in vitamins and minerals(1). Therefore, the school cafeteria should serve larger fruit and vegetable portions along with smaller main course and dessert

portions(4,5,6). This change will be effective because, according to research conducted at the Rose Children's Hospital(7), children eat the same amount of food served to them—about 55 percent(8), on average—no matter what the main course is." ★★★

1. No evidence is provided to prove that the main course and the dessert do not contain sufficient vitamins and minerals to meet daily

requirements of students? health. (I.I/U.A) ★★★★

2. The head fails to consider other ways through which the students could obtain the vital nutritional benefits. (I.T) ★★★★

3. The head hastily assumes that all students who eat meals at school behave the same pattern as those students observed. (C.S) ★★

4. The head provides no evidence to guarantee that the proposed new lunch will be healthy enough for our students. (U.A) ★★

5. The head does not consider the negative effects of changing the proportion of vegetable, main course and dessert in our lunch.

(adv:disadv) ★★★

6. The manager fails to consider other possible methods which could be adopted to improve the health of our students. (I.T) ★★★

7. The children who participated in the research may not be representative of all children and our students. (selective sample/C.S)

★★★★

8. The result of the research does not indicate that students will also eat 55% of their meals if the composition of meals changes to

the level suggested by the head. (U.A) ★★

——————————————————————————————

85, (政治经济/论断/大对比小并列)The following appeared in the letter to an editor of a Myrian newspaper.

"Under Governor Winslow's leadership over the past four years, Myria has enjoyed unprecedented prosperity. Average wage increases this year are higher than they have been at any time since the 1970's(2), the number of people who report being unemployed has decreased by 10

percent(3,4), and construction of new homes is up by one third(1,5). In contrast, over the past four years Governor Winslow's opponent, Mr. Homer, has been mayor of a city in which only a small number of new businesses(5,7) have opened(6). If Myrians want another four years like the past four years(8), they should reelect Governor Winslow(9,10)."

★★★26,140,209,225

1. The author fails to prove that the prosperity of Myria resulted from Winslow?s leadership. (NCR) ★★★★

2. The author fails to inform us how much did average wage actually increase after adjustment for inflation. (I.I) ★★★

3. Other cities might enjoy higher rate of wage increase, and lower rate of unemployment. (confusing variation with comparison) ★★

4. No evidence about the rate of wage increase, and decrease in unemployment rate before Winslow?s governing is provided. (I.I/lack

of comparison) ★★★

5. The author unfairly equates the construction of new homes and opening of new businesses with the prosperity of a city. (U.C) ★★

6. The city under Mr. Homer?s leadership and Myria may not be comparable. (I.C) ★★★★★

7. The author fails to provide information about the size and profitability of new businesses opened at the city governed by Homer. (I.I)

★★★

8. The author unfairly assumes that Mr. Winslow will also perform excellent in the following years. (C→F) ★★

9. Many other factors would influence the development of a city, thus another four years? prosperity is not guaranteed. (sufficiency

of the solution) ★★★★

10. Other excellent candidates might also be available while ignored by the author. (necessity of the solution) ★★★

——————————————————————————————

86, (教育/提议/大并列小递进)The following appeared in a memo from a member of the Clark City School Board.

"Mason Elementary School is faced with a shortage of classroom space. At the same time, parents in our district are increasingly expressing the desire to see improvements in their children's reading abilities(1). Therefore, we should reduce the number of physical education classes at Mason Elementary and convert part of the gymnasium to class-room space(2,3,6). The school will gain the additional space for classrooms without the cost of extending the building and can use the time that students would have spent in physical education classes to provide more reading instruction(4). This plan will lead to improvements in students' reading skills(5)." ★★★

1. The member fails to provide any evidence to prove the necessity of fulfilling the parents? desire. (necessity of the solution) ★★ 31

2. The member fails to consider the cost and negative effects of replacing physical education classes with reading instruction and

converting gymnasium to classrooms. (adv:disadv) ★★★★★

3. The member fails to illustrate the feasibility of converting gymnasium to class-room space. (feasibility of the conclusion) ★★★★

4. Reducing the number of physical education classes does not necessarily ensure that students will spend more total amount of time

on improving reading skills. (U.A) ★★★

5. The member unfairly assumes that the proposed reading instruction will necessarily ensure improvements in students? reading

abilities. (sufficiency of the solution) ★★

6. The member ignores other possible methods to gain classroom space and to improve students? reading skills. (I.T) ★★★ ——————————————————————————————

87, (商业/决策/并列)The following appeared in a memo to managers of Christine's, a chain of craft stores.

"Several major newspapers have recently run articles noting an increased interest in pottery. The number of potters(2) in this country has risen by eighteen percent in the past five years, and pottery is particularly gaining popularity on college campuses(3). In order to take advantage of these trends, Christine's should immediately begin carrying a larger volume of pottery supplies(1,4). Since several of our stores(5) have recently experienced a decrease in sales of painting supplies, all store managers should reallocate shelf space from the painting area for the display of pottery(6). Stores should display posters showing pottery that is designed to appeal to college-age individuals(3,7). These actions will undoubtedly increase our profits dramatically(8,9)." ★★★

1. The author unfairly assumes that people who interested in pottery will consequentially purchase pottery at Christine?s. (U.A) ★★

2. The number of potters does not necessarily indicate the popularity of pottery. (U.C) ★★★

3. The author fails to consider the proportion of college consumers in our entire consumer population, and if they could afford

expensive pottery, and how many profits could they bring for us. (V.D/I.I) ★★★

4. The author unfairly assumes that the trend of increasing interest in pottery, and decrease in sales of painting supplies will continue in

the future. (C→F) ★★★★

5. The fact that some of our stores experienced decrease in sales of painting supplies does not indicate that those goods are also losing

popularity in all of our stores. (C.S) ★★★★

6. Granted that the demand for painting supplies indeed decreased, painting supplies could still generate profits for our stores. (I.T) ★

★★★

7. The author fails to prove that the posters will surely attract college-age individuals. (U.A) ★★★

8. The profits of our stores will be influenced by many factors, thus adopting the manager?s proposal alone may not ensure profits

sufficiently. (sufficiency of the solution) ★★★★

9. Other measures could also be taken to increase profits. (necessity of the solution) ★★★

——————————————————————————————

88, (保健,考古/论断/递进)The following appeared in a newsletter about health published in the country of Sauria. Because this was the diet(3) of the people who lived in ancient Sauria, one would expect them to have had long and healthy lives. Yet the

mummified remains(4) of Sauria's ruling classes(1) from two to three thousand years ago show the existence of many medical problems among the ancient Saurians, including dental problems, elevated blood pressure, obesity, heart disease, and early mortality(5). Clearly, the diet of the ancient Saurians was responsible for these problems(2). The high incidence of high blood pressure, obesity, and heart disease in Sauria today even among those who have tried low-fat, high-carbohydrate diets(6,7) further proves that Sauria's leading nutritional experts are wrong." ★★★★

1.

2.

3.

4.

5.

6.

The health status and of Sauria?s ruling classes might not be representative of all ancient Saurians. (selective sample/C.S) ★★★★ The author unfairly assumes that ancient Sauria?s ruling classes were having the diet mentioned above. (I.I/U.A) ★★★ There may be some difference between the diet of ancient Sauria and the diet defined by modern experts as optimal. (U.C) ★★★★ The study of mummified remains might be unreliable. (credibility of the evidence) ★★★★ The medical problems of Sauria?s ruling classes might result from many possible factors. (NCR) ★★★★★ The author fails to prove that medical problems of modern Sauria do result from their diets. (NCR) ★★★ 32

7. The author fails to rule out the possibility that those modern Saurians who have tried low-fat, high-carbohydrate diets do so merely because they are aware of their existing health problems. (confusing the cause and the effect) ★★★★

——————————————————————————————

89, (交通运输/提议/大并列小递进小类比)The following appeared in a newspaper published in the state of Celera.

"Speed limits on our state's highways should be eliminated(5) in order to increase our state's prosperity. Because greater speed(1) means more efficient travel, commercial deliveries will be faster(2,3), increasing business profits. Elimination of speed limits will also make driving more attractive to motorists(4), so that more people will use the highways, providing more highway toll revenues for the state. At the same time, safety on our highways will not be affected: daytime(8) speed limits were eliminated last year in the western states(6) of our country, and no significant increase(7) in the number of accidents in these states has been reported." ★★228

1.

2.

3.

4.

5.

6.

7.

8. Elimination of speed limits does not necessarily guarantee greater speed. (I.I/U.A) ★★★ The author fails to inform us how many travelers and commercial deliveries are using highways, thus we cannot evaluate to what extent the elimination of speed limit would increase our state?s prosperity. (I.I/U.A) ★★★★ The author fails to inform us what percentage does high way driving make up in the total driving mileage of travelers and commercial deliveries. (V.D) ★★★★ The assumption that elimination of speed limit will necessarily attract more motorists is unfounded. (U.A) ★★★ The author fails to consider the negative effects of eliminating speed limit. (adv:disadv) ★★★★★ The situation of highways in western states and in Celera may not be comparable. (F.A) ★★★★ The fact that the number of accidents has not increased significantly does not indicate that safety would not be affected. (U.C) ★★★★ The fact that eliminating daytime speed limits caused no negative effects on safety does not indicate that safety will be guaranteed if night speed limits are also eliminated. (H.G) ★★★★★

——————————————————————————————

90, (医疗保健,商业/建议/大并列小递进)The following appeared in an editorial on health problems in the country of Corpora. thickening of artery walls is associated with elevated stroke risk(1), and such scans typically find that anywhere from 5 to 20 percent of clients(5) have detectable plaque. All pharmacies(6) should offer this service(3,4), because early detection of increased risk for stroke will encourage clients to consult with their doctors and make important life-style changes(7) to reduce their risk of having a stroke(8). This in turn will cause a decrease in overall medical costs in Corpora(9,10)." ★★★

1.

2.

3.

4.

5.

6.

7.

8.

9. The author fails to establish a causal relationship between the thickening of artery walls and elevated stroke risk. (confusing concurrence with causality) ★★ Since the thickening of artery walls and stroke could be caused by many reasons, the service may not sufficiently find all people who have risk for stroke. (sufficiency of the solution) ★★★ The author fails to consider the feasibility of offering the service for many pharmacies. (feasibility of the conclusion) ★★★ Other methods of detecting risk for stroke are possibly available. (necessity of the solution) ★★ The clients who used the scans may not be representative of all people living in Corpora. (selective sample/C.S) ★★★★ The assumption that all pharmacies need to provide such service is not guaranteed. (C.S) ★★★★ The author unfairly assumes that clients who have been detected to have plaque will trust the detection, and will consult with doctors and make changes in life-style as a result. (U.A) ★★★ No evidence could guarantee that changes in life-style could reduce the risk of having a stroke. (NCR) ★★★ The author does not analyze the constitution of medical costs in Corpora, thus we cannot guarantee that the cost will definitely decrease after the proposal is adopted. (I.I/U. A) ★★★★

10. The author ignores the expenditure of treatment for stroke in people who are diagnosed as stroke, thus the medical costs would not decrease. (I.T) ★★★

——————————————————————————————

91, (运动,保健/预言,论断/并列)The following appeared in an article in Supplements Today.

"Dieticians have long known that professional athletes who eat a lot of nuts have higher levels of endurance than those who do not(1).

Researchers have recently discovered that the particular combination of vitamin N and fiber found in some nuts provides a boost(2) for those 33

who participate in strenuous physical activities(3) daily. Both vitamin N and fiber supplements are easily synthesized and widely available. As a result, all(6) those who participate in athletic activities will be able to increase their endurance and win more games(7) by taking vitamin N along with a fiber supplement(4,5)." ★★★

1. The argument simply assumes that eating a lot of nuts is the reason for higher levels of endurance in athletes. (confusing

concurrence with causality) ★★★★

2. The article fails to clearly describe what kind of boost the combination could provide. (definition of the term ?boost?/I.I) ★★

3. The effect of the combination on people who participate in strenuous physical activities may be different from what is needed for

athletes. (F.A) ★★★★★

4. The article unfairly assumes that taking vitamin N and a fiber supplement separately could have the same effect on endurance as

eating nuts. (U.A) ★★★★

5. The article assumes without any guarantee that synthesized vitamin N and fiber supplements would have the same effect on athletes

as those contained in natural foods. (U.A/U.C) ★★★

6. The article hastily assumes that the cases of professional athletes could also be applied to all people who participated in athletic

activities. (C.S) ★★★★

7. If people could win a game may be determined by many factors, thus taking the supplements alone does not necessarily ensure

more victories. (sufficiency of the solution) ★★★

——————————————————————————————

92, (人事/提议/并列+类比)The following is a recommendation from the human resources department at Techforce Computer Company. "Many working parents report that problems related to their childcare arrangements are a major reason for absenteeism from their jobs(1). Bridgewell Corporation, a tele-communications company, implemented an on-site childcare program and after one year, the company reported that unscheduled absences had declined by 25 percent(2,3). Since Techforce has had an increase in unscheduled employee absences(5) over the past year, we should therefore implement our own on-site childcare program(4). The program will undoubtedly reduce our unscheduled employee absences, resulting in significantly increased productivity(6)." ★★

1. Some employees may take childcare arrangements as an excuse for their absenteeism. (are the respondents being forthright?) ★★

2. The author fails to consider the possible differences between Techforce and Bridgewell. (F.A) ★★★

3. The author simply assumes that it is the childcare program that resulted in the decline in unscheduled absences at Bridgewell. (post

hoc, ergo propter hoc) ★★★★

4. The author fails to consider the negative effects caused by the childcare program. (adv:disadv) ★★★★

5. The author unfairly assumes that most unscheduled absences at Techforce are related to childcare problems. (I.I) ★★★

6. The author?s assumption that the proposed program will necessarily result in increased productivity is unwarranted. (U.A) ★★

——————————————————————————————

93, (工程建设/决策/并列+对比)The following appeared in a memo from a vice president of a large, highly diversified company.

"Ten years ago(4) our company had two new regional office buildings built in two different regions. The buildings were erected by two different construction companies—Alpha and Zeta. Even though the two buildings had virtually identical floor plans, the building constructed by Zeta cost 30 percent more to build, and its expenses for maintenance last year were twice those of the building constructed by Alpha. Furthermore, the energy consumption of the Zeta building has been higher(2) than that of the Alpha building every year since its construction(1). Such data, plus the fact that Alpha has a stable workforce with little employee turnover(3), indicate that we should use Alpha Construction Company, rather than Zeta(6), for all(5) future building projects." ★★17,41,233

1. The vice president fails to consider many other differences between the two regions and the two buildings. (I.C) ★★★★★

2. The high expenses for maintenance of the Zeta building last year might be an unusual case, and do not necessarily continue in the

future. (P→F) ★★★

3. A stable workforce with little employee turnover tells nothing about the construction quality of a construction company. (I.E/U.C)

★★★

4. The vice president unfairly assumes that the working quality of Alpha is still superior to that of Zeta today as decades ago. (P→F)

★★★★

5. The facts cited by the vice president do not necessarily indicate that Alpha has advantages over Zeta in constructing all kinds of 34

buildings. (C.S) ★★★★★

6. The vice president fails to consider other competent construction companies besides Alpha and Zeta. (F.D) ★★★

——————————————————————————————

94, (地产,建设/决策/并列)The following appeared in a memo from the president of Bower Builders, a company that constructs new homes.

"A nationwide survey(1) reveals that the two most desired home features(2) are a bathroom with a whirlpool tub and a large kitchen. Homes in a nearby development built by our competitor, Domus Construction, have whirlpool tubs and have sold much faster and at significantly higher prices than the average(3,4). To increase our sales and profits, we should include whirlpool tubs and larger kitchens as standard features(6) in all our new homes(5,7,8). Since our recent buyers(10) have voiced no complaints about small yards(9,10), we can also increase profits by decreasing the size of our yards." ★★

1. The result of the nationwide survey does not necessarily apply to all specific regions. (C.S) ★★★★

2. Many other features are also important for consumers when they buy new houses. (sufficiency of the solution) ★★★

3. The president unfairly assumes that it is the whirlpool tubs that resulted in the fast selling and higher price of the development built

by Domus. (NCR) ★★★★★

4. The president fails to provide any information about the profits Domus gained from the mentioned development. (I.I) ★★★

5. There may be many other differences between Bowers and Domus. (F.A) ★★★

6. Merely Including the new features in new homes may not sufficiently ensure our sales and profits to increase. (sufficiency of the

solution) ★★

7. The president fails to illustrate the necessity of including the features in all new homes. (C.S/necessity of the solution) ★★

8. The president fails to consider the possible cost of providing these features, thus we cannot ensure that our profits will certainly

increase. (adv:disadv) ★★★

9. The fact that no complaint about small yards has been received does not indicate that there will be no negative effects of

decreasing the size of yards at all. (I.E) ★★★★

10. The feedback from the recent buyers may not be representative of all consumers. (C.S) ★★★

11. The fact that recent buyers did not complain does not indicate there will not be any dissatisfaction in the future. (C→F) ★★★ ——————————————————————————————

95, (社会,教育/论断,预言/并列)The following appeared in a memo from a budget planner in Sophia County.

"Census data indicate that the number of retired couples without children who are moving into Sophia County is steadily rising(1); private schools in our county report substantial increases in enrollment(2,3); and a statewide survey(4) of parents shows that over 10 percent(5) now support the idea of home schooling instead of public schooling funded by the county government. Such demographic trends suggest that our county will not have to construct new school buildings(6) and that we can therefore decrease the budget for county-funded public schools.(7)" ★★★

1. The planner fails to provide the number of retired couples who moved to Sophia with children. (I.I) ★★

2. The fact that enrollment at private schools increased does not necessarily indicate that enrollment at public schools would decrease.

(U.A) ★★★

3. The planner provides no evidence to show that these past trends will continue in the future. (P→F) ★★★★

4. The result of the statewide survey does not indicate that parents in Sophia would also support the idea. (C.S) ★★★

5. 10 percent of respondents may not constitute a statistically representative sample. (quantity of the sample/are the respondents

representative) ★★★★

6. Other factors would make it necessary to construct new school buildings. (I.T) ★★★

7. The planner does not demonstrate that our funding for county-funded public schools is sufficient and could be decreased. (U.A)

★★★

——————————————————————————————

96, (医疗保健/建议/并列+对比)The following appeared in a magazine focusing on health and fitness.

"A recent study of several nutritional supplements showed that subjects who took a daily pill containing luceen had significantly better vision of distant objects than subjects in the study who took other nutritional supplements(1). Luceen appears naturally in a tasty fruit that grows on the island of Bonopia, and very few inhabitants of that island wear eyeglasses or other corrective lenses(3,4). Such evidence suggests that people who develop vision problems(6) should take regular luceen supplements(2,5,7)." ★★★

35

1. The author fails to consider other possible difference between the two groups of subjects. (I.C) ★★★

2. The author does not point out the actual and exact effect of luceen on vision. (I.I) ★★★

3. The fact that few inhabitants of Bonopia wear eyeglasses or other corrective lenses does not prove that most inhabitants there have

good vision. (U.C) ★★★★

4. Granted that most inhabitants of Bonopia do have good vision, we cannot ensure that luccen in the fruits is the reason. (NCR) ★★

★★★

5. The author fails to consider possible negative effects of taking luceen supplements. (adv:disadv) ★★★

6. The author hastily assumes that taking luceen supplements could cure other vision problems. (C.S) ★★★★

7. There may be other better treatment for vision problems. (necessity of the solution) ★★

——————————————————————————————

97, (传媒/决策/并列+类比)The following appeared in a memo from the manager of television station KICK.

"A nationwide survey(2) reveals that a sizeable majority of men(1) would like to see additional sports programs on television. After television station WACK(3) increased its sports broadcasts, its share of the television audience in its viewing area almost doubled(4). To gain a larger audience share in our area, and thus increase company profits(5), KICK should also revise its broadcast schedule to include more sports coverage(6,7)." ★★235

1. Only male audience were surveyed in the nationwide survey. (selective sample) ★★★★

2. The result of the national survey may not apply to the area at where KICK locates. (C.S) ★★★★

3. The situation of KICK and WACK might be quite different. (F.A) ★★★★

4. Other factors may also lead to the increased audience share of WACK. (NCR) ★★★

5. The manager unfairly assumes that the proposed change would necessarily lead to larger audience share and more company profits.

(NCR/sufficiency of the solution) ★★★

6. The manager does not analyze current performance, advantages and profitability of KICK to illustrate the necessity of carrying out

such revision. (I.I/necessity of the solution) ★★★★

7. Other methods could also be used to gain larger audience share and increase company profits. (I.T) ★★

——————————————————————————————

98, (医疗卫生/决策/并列+对比)The following appeared in a memo from the director of a large group of hospitals.

"In a laboratory study(1) of liquid antibacterial hand soaps, a concentrated solution of Nadasept killed 40 percent more bacteria(2) than the liquid hand soaps currently used in our hospitals. During a subsequent test of Nadasept at our hospital in Saluda, that hospital reported significantly fewer cases of patient infection(3) than did any of the other hospitals in our group(4). Therefore, to prevent serious patient infections(6), we should supply Nadasept(8,9) at all(5) hand washing stations throughout our hospital system(7)." ★★

1. The result of the laboratory study may not properly apply to normal hospital environment. (H.G) ★★★

2. The director fails to inform us if the bacteria killed by Nadasept are all harmful. (I.I) ★★★

3. The director unfairly assumes that it is Nadasept that resulted in the fewer cases of patient infection at the tested hospital. (NCR) ★

★★★

4. There may be many other differences between the tested hospital and its counterparts. (I.C) ★★★★★

5. The success of Nadasept at the tested hospital, if there is any, does not necessarily recur at other hospitals. (C.S) ★★★

6. The director fails to differentiate between the severity of infections at the tested hospital, thus we cannot ensure that Nadasept can

effectively prevent serious infections. (I.T) ★★★★

7. Granted that Nadasept is very effective in killing bacteria, the director does not demonstrate the necessity of supplying Nadasept at

all our hospitals. (necessity of the solution) ★★★

8. The director overlooks other effective measures of preventing patient infections. (I.T) ★★

9. The director fails to consider if Nadasept has any side effect. (adv:disadv) ★★

——————————————————————————————

99, (农业/建议,预言/类比)The following appeared in a memo from the economic minister of the small country Paucia.

"Using a newly developed variety of seed, farmers in our neighboring country Abundia produced 80 percent more rice(1) last year than in any previous year. To increase the income of farmers(5) in our own country(2), we should encourage them to cultivate this new variety of rice(4) rather than some of their traditional crops(3). Such high yields of rice will also improve our country's balance of trade by enabling us to begin exporting it(6)." ★★204

36

1. The minister fails to establish a causal relationship between using the new variety of seed and the high yields of rice in Abundia.

(post hoc, ergo propter hoc) ★★★★

2. Many factors that would influence yields of rice may be quite different in Abundia and Paucia. (F.A) ★★★★★

3. The minister fails to analyze the advantages and profitability of farmers in Paucia who cultivating traditional crops. (I.I) ★★★★

4. The minister fails to consider the feasibility of cultivating the new variety of rice in Paucia. (feasibility of the conclusion) ★★★

5. The minister does not analyze the cost, profit, and actual demand for the new variety of rice, thus cultivating it may not ensure

profits. (I.I) ★★★★

6. The balance of trade might be influenced by many factors, thus adopting the minister?s proposal alone may not necessarily

improve the balance of trade. (sufficiency of the solution) ★★★

——————————————————————————————

100, (生产,就业/决策/并列)The following appeared in a memo from the chief operating officer of the Presto Manufacturing Company. "Since orders for our major product increased by over 200 percent last quarter(1,2), we should promptly expand production by building a new manufacturing plant in Summit City. Summit City is the ideal location(5,7) for the new plant because it has low property taxes, extensive blocks of land available(4) for immediate purchase, and a large number of residents who are not currently employed(3,6)." ★24,226,240

1. The officer hastily assumes that orders for our products will also increase in the future. (P→F) ★★★

2. The officer does not provide information concerning the actual amount of production currently needed, thus we cannot evaluate if

the new manufacturing plant proposed by the officer is necessary. (V.D) ★★★

3. The low taxes, large amount of unused land and unemployed residents may just indicate a weak economy in Summit City, thus the

city may not be ideal for our new plant. (negative evidence) ★★★★

4. The officer fails to inform us about the requirements of the new plant, and fails to prove that the available land mentioned above are

suitable for constructing our new plant. (I.I) ★★★★

5. Many other factors would influence our decision on choosing the site while the officer does not consider them sufficiently. (I.T) ★

★★

6. The officer unfairly assumes that the residents in Summit City who are not employed are willing to and able to work at Presto. (U.A)

★★★

7. The officer neglects other cities suitable for building the new plant. (necessity of the solution) ★★

——————————————————————————————

101, (食品/决策,预言/对比)The following appeared in a memo from the president of a company that makes breakfast cereals.

"In a recent study, subjects who ate soybeans at least five times per week had significantly lower cholesterol levels than subjects who ate no soy products(1,2). By fortifying our Wheat-O cereal with soy protein(3,5), we can increase sales by appealing to additional consumers who are

concerned about their health(4). This new version of Wheat-O should increase company profits(6) and, at the same time, improve the health of our customers(7)." ★★

1. The study does not establish a causal relationship between eating soybeans frequently and lower cholesterol level. (confusing

concurrence with causality) ★★★★

2. Many other differences between the two groups of subjects studied may account for their different cholesterol levels. (I.C) ★★★

3. The president?s assumption that soy protein will act the same on cholesterol level as soybean is unwarranted. (U.C) ★★★★

4. Consumers who are concerned about their health are not necessarily interested in soy protein. (U.C) ★★★

5. The president fails to consider possible negative effects of adding soy protein to Wheat-O. (adv:disadv) ★★★

6. The president does not analyze the cost of the proposed change, thus the increased profits are not guaranteed. (adv:disadv) ★★★

7. Soy protein may be beneficial for some people, but the president?s assumption that adding soy protein in Wheat-O will improve

the health of most customers is unfounded. Not all customers are facing cholesterol related problems. (U.A) ★★★

——————————————————————————————

102, (交通运输,城市/决策/类比+并列)The following appeared in a letter from the mayor of Tudor City.

"After a major airline chose Yorkville as its regional flight hub, the number of businesses in that city almost doubled and local tax revenue increased by 50 percent(1). In addition, as travel to Yorkville became easier, most respondents to a survey of the country's top executives(2) identified Yorkville as a desirable place to transact business. To secure efficient transportation for Tudor City and stimulate our local economy(8), we should build a new airport(4,5,6), like the one in Yorkville(3), that is suitable for a regional flight hub(7)." ★★

37

1.

2.

3.

4.

5.

6.

7.

8. The mayor fails to consider many other factors that may lead to the increase in the number of businesses and local tax revenue in Yorkville. (post hoc, ergo propter hoc) ★★★★ Executives who prefer Yorkville as their transaction place may be more likely to respond to the survey. (are the respondents representative?) ★★★ The situation in Yorkville and Tudor City might be quite different. (F.A) ★★★★★ The mayor fails to illustrate the feasibility of constructing the new airport in Tudor. (feasibility of the conclusion) ★★★ The mayor fails to analyze the possible cost of the new airport, and if the city can afford it. (adv:disadv) ★★★★ The mayor fails to consider the negative effects of constructing the airport. (adv:disadv) ★★★ The mayor unfairly assumes that once the proposed new airport is built, major airlines will chose it as a regional flight hub. (U.A) ★★★★ The economy of Tudor may be influenced by many factors, thus adopting the mayor?s proposal alone may not stimulate our local economy sufficiently. (sufficiency of the conclusion) ★★★

——————————————————————————————

103, (道路交通/论断/对比)the highway has not been significantly lower in the six months since the redesign than it was in the six months(1) before the redesign. Therefore, the redesign clearly did not improve the curves and roadway visibility enough to make a difference(4). ★★★

1. Other differences between the first six months and the following six months could also explain why the number of accidents has not been lower significantly after the redesign. (I.C) ★★★★

2. The author does not inform us the accident rate on Blue City Highway after the redesign, thus we could not evaluate if the redesign is effective. (V.D) ★★★★

3. The author does not provide any information concerning the severity of accidents. (I.I) ★★★★

4. The number of accidents might be even more without the redesign. (lack of controlled experiment) ★★★★★

——————————————————————————————

104, (医疗保健/提议/对比)Under these uncomfortable conditions, it is doubtful that doctors can get accurate measurements(1). A new head-mounted device that measures eye movements(2) rather than blood flow and allows patients to move around(3) will undoubtedly provide better brain

measurements(4). Thus, the new head-mounted device should replace the older device(6) at all hospitals and research institutions(5). ★★★

1. The author?s assumption that the older device cannot generate accurate measurements is unwarranted. (U.A) ★★★★

2. The assumption that eye movements are a more accurate indicator of brain activity than blood flow is unfounded. (I.I/U.A) ★★★★

3. The new device might also be uncomfortable and cause some other problems, thus may not be preferable by patients. (adv:disadv) ★★★

4. The author unfairly assumes that the new device would necessarily generate accurate data. (U.A) ★★★

5. The author fails to consider the cost and feasibility of replacing older device at all hospitals. (adv:disadv/I.T) ★★★★

6. Granted that the new device is more advanced, the older one might still have some advantages and still necessary in some hospitals. (necessity of the solution) ★★★★

——————————————————————————————

105, (医疗保健/建议/并列)The following appeared in a health newsletter.

"According to a recent study, people with many social ties report catching colds(4) less often than do people with few social ties(1,2,3). Consequently, researchers conclude that having an active social life(5) probably helps strengthen the immune system(6). The researchers note that catching a cold—one of a family of highly contagious viruses—gives the cold-sufferer temporary immunity(6) to that virus in the future, but not to the many other related viruses. Merely being exposed to a new cold virus, however, is not enough for a person to catch a cold, since a strong immune system can successfully fight off some new viruses. Thus, in order to prevent catching a cold, people should strengthen their immune systems by becoming more active socially(7,8)."

38

★★★★

1. The author fails to establish a causal relationship between having many social ties and lower frequency of catching cold. (confusing

concurrence with causality) ★★★★

2. The author fails to rule out the possibility that some people avoid social activities just because they are more likely to catch colds.

(confusing the cause and the effect) ★★★★

3. There may be other differences between people who have many social ties and those who do not. (I.C) ★★★★

4. Different groups of people may have different definition of catching colds. (I.C/definition of the term ?catching colds?) ★★★

5. The researchers simply equate having many social ties with taking an active social life. (U.C) ★★

6. No evidence could indicate that exposure to virus could strengthen the immune system, and how long could the temporary

immunity be effective. (U.A/I.I) ★★★

7. Merely becoming more socially active may not strengthen people?s immune system sufficiently. (sufficiency of the solution) ★

★★

8. The author fails to consider negative effects for some people to have an active social life. (adv:disadv) ★★★★

——————————————————————————————

106, (生产,环境/预言/并列)The following editorial appeared in a newspaper in the country of Solaria. have opened factories there. A recent study found that water pollution in the region was worse than in any other region in the country(1,2). Moreover, the computer-chip factories, which use large quantities of water to manufacture the chips, are probably responsible for the low levels of water in the region's lakes and reservoirs(3). Therefore, if the region's computer-chip makers had limits placed on the amount of water(4) they could use, water quality would improve(5)." ★★★

1. The author unfairly assumes that it is the computer-chip factories that resulted in the water pollution. (NCR) ★★★★

2. The author fails to inform us the level of water pollution twenty years ago. (confusing comparison with variation) ★★★★

3. Other explanations could also account for the low levels of water in lakes and reservoirs. (NCR) ★★★★

4. The proposed limitation on the amount of water used may not necessarily ensure improving of water quality. (U.C) ★★★

5. Water quality is influenced by many factors, thus carrying out the limitation alone may not sufficiently improve water quality.

(sufficiency of the solution) ★★★

——————————————————————————————

107, (社会/建议/——)The following appeared in an editorial in the Seatown newspaper. would be forced out of the port(3). We should preserve the port for the fishing fleet(1,2), which, unlike pleasure boats, contributes to the prosperity of Seatown(4)." ★★★

1. In the absence of data concerning the total amount of fishing boats which still need the port, the necessity of the author?s

recommendation is unfounded. (I.I/necessity of the solution) ★★★★

2. The author fails to consider the negative effects of preserving the port for fishing fleet. (adv:disadv) ★★★★

3. The author ignores other possible settlement for fishing fleet besides preserving the whole port for them. (I.T) ★★★★★

4. The author unfairly assumes that only fishing boats could contribute to the prosperity of Seatown.(U.C) ★★★★★

——————————————————————————————

108, (环境/论断/大并列小类比)The following appeared in a Brenton newspaper. However, in Uptown, where the new system is used, a study found that the complex network of pipes in the new system tends to accumulate algae(1). The build up of algae can be avoided by scrubbing the pipes, which is costly, or by adding an herbicide to the water in the pipes to prevent algae accumulation(2,3). But water containing the herbicide cannot be released back into the river(4) and it is known that low water levels(6) can harm river ecosystems accustomed to higher levels. Therefore, Brenton power plant should continue to use the old cooling system exclusively(7)." ★★★

39

1. The problem in Uptown may not necessarily also occur in Brenton. (F.A) ★★★★★

2. The author does not provide any evidence to illustrate the necessity of eliminating algae. (necessity of the solution) ★★★

3. The author fails to inform us if there are any other possible methods to avoid, or to eliminate the build up of algae. (F.D) ★★★

4. Other possible methods might be available to eliminate the negative effect of herbicide contained in warmed water. (I.T) ★★★

5. The author fails to thoroughly evaluate the positive and negative effects of using the new system. (adv:disadv) ★★★★★

6. The author does not inform us how much water does the power plant actually use, thus using the new system may not necessarily

lead to low water level. (I.I/U.A) ★★

7. The author ignores other possible methods besides using the two systems mentioned in the argument. (F.D) ★★★

——————————————————————————————

109, (城市,政治经济/论断/对比)The following appeared in a letter to the editor of the Maple City newspaper.

"Twenty years ago Pine City established strict laws designed to limit the number of new buildings that could be constructed in the city. Since that time the average housing prices in Pine City have increased considerably(1). Chestnut City, which is about the same size as Pine City, has over the past twenty years experienced an increase in average housing prices similar to Pine City, but Chestnut City never established any laws that limit new building construction. So it is clear that laws limiting new construction have no effect on average housing prices(2). So if Maple City were to establish strict laws that limit new building construction, these laws will have no effect on average housing prices(3)." ★★★★★

1. The author fails to establish a causal relationship between carrying out limitation on new building construction and increased

housing prices in Pine City. (NCR) ★★★

2. The author does not demonstrate that Pine City and Chestnut City are comparable at every aspect. (I.C) ★★★

3. The author fails to prove that the situation of Chestnut City will also take place in Maple City, or that Maple City will not follow

Pine City?s pattern. (F.A) ★★★★★

——————————————————————————————

110, (学校建设/决策/大并列小类比)The following appeared in a memo from the Principal of Sherwood Junior High School.

"It appears that the lighting in Sherwood High's classrooms, which have large windows but minimal overhead light sources, is impairing student academic performance and teacher morale. Records show that during December and January, the two months with the fewest hours of

daylight(1), attendance rates fall, average daily class participation drops, and grades decline(2). Teacher resignations are also highest during these months. According to a study of Tundra Vocational School, which experiences entire seasons with little daylight, students' grade point averages increased(3) when Day Glow light bulbs, which mimic sunlight, were installed in classrooms. This study suggests that Sherwood can improve students' academic performance and teachers' morale by using Day Glow light bulbs(5) in winter(4)." ★★★★

1. No evidence is provided to show that sunlight in December and January is insufficient for students? academic study at Sherwood.

(I.E/U.A/necessity of the solution) ★★★

2. The Principal fails to prove that it is the short of lighting in our classrooms that resulted in the lower attendance rates, daily class

participation and grades. (NCR) ★★★★★

3. Other factors may explain the increase in students? grade point averages at Tundra Vocational School. (post hoc, ergo propter hoc)

★★★★★

4. There may be many other differences between Tundra Vocational School and Sherwood Junior High School. (F.A) ★★★★

5. The Principal ignores other possible ways to improve lighting besides using Day Glow light bulbs. (I.T) ★★★

——————————————————————————————

111, (商业,生产/决策/并列)The following appeared in a memo from the vice president of marketing at Dura-Sock, Inc.

"A recent study of Dura-Sock wearers suggests that our company is wasting the money it spends on its patented "Endure" manufacturing process(1), which ensures that our socks are strong enough to last for two years. Dura-Sock has always advertised its use of the "Endure" process, but the new study shows that the average Dura-Sock customer actually purchases new Dura-Socks every three months. Furthermore, Dura-Sock customers surveyed in our largest market, northeastern United States cities(2), say that they most value Dura-Sock's stylish

appearance and availability in many colors(3). These findings suggest that Dura-Sock can increase its profits(4,6) by discontinuing its use of the "Endure" manufacturing process(5)." ★★★

1. The vice president fails to rule out the possibility that the customers purchases new Dura-Socks frequently just because they favor

the “ Endure” process. (I.T) ★★★★

2. The customers surveyed in northeastern United States cities may not be representative of all our customers. (are the respondents 40

representative?) ★★★

3. The study cited by the vice president does not imply that our customers no longer care about the durability of socks. (I.E) ★★★★

4. The vice president fails to inform us the cost of using “Endure” process, thus we cannot evaluate the president?s conclusion.

(adv:disadv) ★★★

5. The vice president fails to consider the negative effect of discontinuing the use of the process. (adv:disadv) ★★★★

6. The vice president fails to consider and analyze other factors that would influence our profits. (sufficiency of the solution) ★★ ——————————————————————————————

112, (环境/建议/并列)The following proposal was raised at a meeting of the Franklin City Council. of the bay that have previously been damaged by industrialization. This plan should be adopted, for it is necessary to reduce the flight

delays(1,5), and the wetlands restoration part of the plan ensures that the bay's environment will actually be helped rather than hurt(2,3,4)." ★★★★

1. The author fails to prove that the construction of new runways could reduce the flight delays. (NCR) ★★★

2. The author simply assumes that funding for the proposed restoration of wetlands will be actually effective and can necessarily

restore the wetlands to their original situation. (U.A) ★★★★

3. The airport?s promise about funding for restoration of wetland does not demonstrate that we could continue harming our

environment. (U.A) ★★★★★

4. The argument fails to evaluate the positive and negative effects of the proposed actions thoroughly. (adv:disadv) ★★★★

5. The author ignores other solutions to reduce the flight delays. (necessity of the solution) ★★★★

——————————————————————————————

113, (保健/建议/并列)The following was published by a consumer protection agency.

"Three years ago, So-Low launched a nationwide ad campaign, focusing heavily on sunny regions and distributing free sunglasses there. But although So-Low sunglasses cost less than higher-priced brands, they block a smaller proportion of the Sun's rays, including the type of rays known to damage the eyes even when the person wearing the sunglasses feels no discomfort. A recent study suggests that So-Low sunglasses can actually increase the risk of damage to people's eyes(2) by creating a false sense of security. The study shows a sharp increase in the incidence of vision problems(1) in the sunny regions(3) over the past three years. These findings suggest that anyone concerned about eye damage from the Sun should avoid So-Low brand and instead either pay for higher-priced brands(4) or wear no sunglasses at all(5)." ★★★

1. We need to know detailed information about the vision problems emerged these years to evaluate if they are related to sunglasses.

(NCR) ★★★★

2. Granted that these problems are related to sunglasses, we cannot make the hasty inference that it is So-Low that caused them. (NCR)

★★★★★

3. The study conducted at the sunny regions may not be representative of all other regions, So-Low could still be suitable for people in

other regions. (C.S) ★★★

4. The author unfairly assumes that the problems caused by So-Low will not take place on higher-priced brands. (U.A) ★★★

5. The argument ignores other effective solutions besides those proposed by the author. (F.D) ★★★

——————————————————————————————

114, (保健/建议/对比)A recently completed study shows that people dwelling in stairs-only apartment buildings (that is, buildings without elevators) live an average of three years longer than do people who live in buildings with both elevators(4) and stairs. A second study shows that elderly residents of buildings with elevators make, on average, twice as many visits to doctors(3) each year as do elderly residents of buildings without elevators(1,2). These findings suggest that even a very moderate amount of daily exercise, such as that required to use the stairs leading to and from one's apartment, can increase people's health and longevity. The findings also suggest that new apartment buildings should be constructed with as few elevators as possible(5). ★★★60,129

1. The author fails to consider other possible differences between people who live in stairs-only apartments and people who live in

buildings with elevators. (I.C) ★★★★

2. The author fails to consider the possibility that some people choose to live in the buildings with elevator because of their weak

health and physique. (confusing the cause and the effect) ★★★★★

41

3. The fact that residents of buildings without elevator make fewer visits to doctors does not necessarily indicate that they are healthier

than other people. (U.C) ★★★★

4. The author unfairly assumes that people who live in buildings with elevators will necessarily use elevators. (U.A) ★★★

5. The author fails to consider the necessity of constructing sufficient elevators in new buildings, and the inconvenience with fewer

elevators. (adv:disadv) ★★★★

——————————————————————————————

115, (商业/建议/递进)The following appeared in a clothing industry trade journal in the country of Sartoria.

"A popular fashion magazine(3) recently conducted a survey of consumers' opinions about clothing prices, using prepaid mailing envelopes. Eighty-nine percent of the approximately 20,000 respondents(1) reported that they were either "dissatisfied" or "very dissatisfied"(2) with current clothing prices. The large number of responses indicates that the Sartorian clothing industry is at risk(5). Given the survey results,

clothing manufacturers must find a way to reduce prices(6) in order to keep their existing customers and maintain their businesses(4)." ★★★

1. We do not know the 20,000 respondents constitute what percent of the whole consumer population, thus the result of the survey may

not be representative. (quantity of the sample) ★★

2. We do not know what question was asked in the survey and if there are other options for the respondents, thus the result may not

reflect actual opinion of consumers. (what question was asked in the survey?) ★★★★

3. The readers of the fashion magazine may not be representative of all consumers. (who conducted the survey/are the respondents

representative?) ★★★

4. There may still be great demand for clothing although customers are dissatisfied with clothing prices. (I.E) ★★★

5. The author does not indicate whether the respondents are dissatisfied with prices of imported clothing or with domestic clothing

prices, thus the assumption that clothing industry at Sartorian is also at risk is unfounded. (C.S) ★★★★

6. The author fails to consider other possible solutions to keep customers and maintain business. (necessity of the solution) ★★★

——————————————————————————————

116, (商业/决策/递进+类比)The following appeared in a memo from Grocery Town's regional manager. the nearby residential development gets closer to full capacity(2). It follows that people prefer a pricing policy where they can find bargains on specific items(1). Since there is a new residential development planned in Oak City, we should change the pricing policy at all(4) of our Oak City stores from our current 'everyday low prices' policy(5) to a 'high-low' policy(3). This will increase the profits at all of our stores in Oak City(6)." ★★★

1. The manager unfairly assumes that it is the pricing policy that resulted in the increased profits of the new store. (NCR) ★★★★★

2. The manager hastily assumes that the profits of the new store will continue to increase in the future. (P→F) ★★★

3. The consumption patterns of consumers in Oak City may be different from those in Elm City. (F.A) ★★★★★

4. The assumption that all stores in Oak City need to change their pricing policy is unwarranted. (C.S) ★★★★

5. The manage fails to analyze the advantage and profitability of the current policy in Oak City, thus the necessity of changing the

pricing policy is not guaranteed. (I.I) ★★★★

6. The profits of our stores might be influenced by many factors, thus adopting the policy alone may not sufficiently ensure profits.

(sufficiency of the solution) ★★★

——————————————————————————————

117, (商业/决策/——)The following is a memo from the business manager of Valu-Mart stores.

"Over 70 percent of the respondents to a recent survey(1) reported that they are required to take more work home with them from the

workplace than they were in the past(2). Since Valu-Mart has not seen impressive sales in its office-supply departments in the past, we should take advantage of this work-at-home trend by increasing at all(4) Valu-Mart stores the stock of home office machines(3) such as printers, small copy machines, paper shredders, and fax machines. We will also increase stock of office supplies such as paper, pens, and staplers. With these changes, our office-supply departments will become the most profitable component of our stores(5)." ★★★14,219

1. We do not know to what extent the result of the survey is representative. (are the respondents representative?) ★★★

2. The result of the survey does not necessarily indicate that the demand for home office machines and office supplies mentioned by

the manager will increase. (I.E) ★★★★★

42

3. The manager fails to prove that Valu-Mart stores do not have sufficient stock of home office machines and supplies currently. (I.I)

★★

4. The necessity of increasing stock of home office machines and supplies at all Valu-Mart stores is not guaranteed. (C.S) ★★★★

5. The profits of our stores might be influenced by many factors, thus adopting the proposal alone may not sufficiently ensure

profits. (sufficiency of the solution) ★★★

——————————————————————————————

118, (行政/提议/递进)The following appeared in a newsletter from a national astronomy association.

"Various sources are predicting higher-than-average temperatures(1) across the country next winter, including in Sun City(2), the traditional location of our yearly winter conference. Higher winter temperatures are sure to result in higher-than-usual tourism in Sun City, a location already known for its attractive beaches and good weather. Hotels will have fewer rooms available, transportation will be more difficult to reserve, and public places such as parks and restaurants will be more crowded(3). These conditions are likely to significantly reduce attendance at the conference(4). We should therefore move our conference to a city less popular with winter tourists(5)." ★★★★

1. No evidence could guarantee that the higher-than-average temperatures will take place. (credibility of the evidence) ★★★

2. The author unfairly assumes that Sun City will also influenced by the weather pattern. (C.S/U.A) ★★★

3. The assumption that higher winter temperatures will inevitably result in more tourism, less hotel rooms, and more crowed public

spaces is unwarranted. (U.A) ★★★★

4. The author fails to substantiate the assumption that the cited conditions will surely reduce attendance at the conference. (U.A) ★★

★★

5. The author fails to consider the negative effects of moving the conference to other cities. (adv:disadv) ★★★★

——————————————————————————————

119, (交通运输/建议/类比+并列)The following is a letter to the editor of the Waymarsh Times.

"Traffic problems here in Waymarsh are obviously reaching record levels. While just three months ago it would take me 15 minutes to get to work, it now takes closer to 25(1,2). Waymarsh should follow the example of our neighboring city Gearsville. Last year, Gearsville implemented a policy that rewards people who share rides to work with coupons for free gas(7). Pollution levels in Gearsville have dropped since the policy was implemented(3), and several friends(4) who live in Gearsville tell me that their trip to work is quicker than it used to be. With the terrible traffic and high pollution(6) in Waymarsh, we must implement a policy similar to Gearsville's(5)." ★★18,55

1. The author fails to prove that the situation faced by the author is not an unusual case. (C.S) ★★★★

2. The author unfairly assumes that the current situation will continue in the future. (C→F) ★★★

3. The author fails to establish a causal relationship between implementing the policy and drop of pollution levels in Gearsville. (post

hoc, ergo propter hoc) ★★★★

4. The author?s friends may not be representative of all Gearsville residents. (C.S) ★★★

5. Implementing the policy similar to Gearsville?s might not be equally effective in Waymarsh. (F.A) ★★★★★

6. We do not know the actual level of pollution in Waymarsh and the causes for the pollution, thus implementing the policy may not

necessarily improve Waymarsh?s traffic and environment. (I.I/NCR) ★★★★

7. The proposed reward policy may result in some negative effects. (adv:disadv) ★★★

——————————————————————————————

120, (生活,安全/论断/递进)The following appeared in a health newsletter.

"A ten-year nationwide study of the effectiveness of wearing a helmet while bicycling indicates that ten years ago, approximately 35 percent of all bicyclists reported wearing helmets, whereas today that number is nearly 80 percent. Another study, however, suggests that during the same ten-year period, the number of accidents(1) caused by bicycling(3) has increased 200 percent(2). These results demonstrate that bicyclists feel safer because they are wearing helmets(4) and they take more risks as a result(5). Thus, to reduce the number of serious injuries from bicycle accidents(7), the government should concentrate more on educating people about bicycle safety(6) and less on encouraging or requiring bicyclists to wear helmets(8)." ★★

1. For lack of data concerning the accident rates and total number of bicyclists, the assumption that wearing helmets actually increased

bicyclists? risks is open to doubt. (V.D) ★★★★★

2. Other factors may also result in the increase in the number of accidents. (NCR) ★★★★★

3. The author fails to indicate that how many bicyclists involved in accidents are wearing helmets. (I.I) ★★★★

4. The author unfairly assumes that wearing helmets will necessarily create false senses of safety in bicyclists. (U.A) ★★★

43

5. The assumption that bicyclists will take more risk when they feel safe is unfounded. (U.A) ★★★

6. The author falsely assumes that educating people about bicycle safety will be effective in reducing the number of serious injuries.

(NCR/U.A) ★★★

7. The author fails to consider accidents caused by other vehicles and the severity of injuries, thus the proposed education may not

be sufficient for ensuring safety of bicyclists. (I.I/U.C) ★★★

8. Requiring bicyclists to wear helmets would still be necessary to ensure bicyclists? safety. (I.T) ★★★★

121, (劳动生产/决策/对比+递进)The following is a memo from the president of Cyberell Computer Company.

"All of our customer-service employees recently attended a two-day retreat during which they received retraining in effective customer service. Subsequently, Cyberell's employee-performance study showed that the retreat benefited new employees—those who have worked with Cyberell for less than two years—far more than it did experienced employees. According to the study, after the retreat new employees were able to handle an average of ten percent more calls per hour, and the total number of customer complaints(2) about new employees decreased(1), but experienced employees showed little improvement in these areas. Therefore, Cyberell should send only(3) new employees(4) to future retreats and should use the resulting savings to double the length of the retreats so that the retreats will be more likely to yield optimum employee performance(5)." ★★★

1. The president fails to establish a causal relationship between attending the retreat and better performance among new employees.

(NCR) ★★★

2. The decreased total number of customer complaints does not necessarily indicate a better performance of new employees. (U.C) ★

★★★

3. The fact that this retreat did not much benefit experienced employees does not indicate that these employees do not need future

retreat at all. (C.S/U.A) ★★★★

4. The president simply assumes that the cited case at customer-service department will also take place in all other departments of

Cyberell. (C.S) ★★★

5. The president assumes without any guarantee that the lengthened retreats will yield optimum employee performance. (U.A) ★

★★

——————————————————————————————

122, (节能/对策/——)The following appeared in the Pine City Gazette.

"Fifteen years ago, Pine City launched an electricity-conservation program that reimbursed residents some of the cost for replacing

energy-wasteful motors, home office equipment, and home appliances with energy-efficient ones. For ten years, spending on this program increased annually, and annual total energy consumption declined. But spending on the program began to decline five years ago, and since then Pine City's total electricity consumption has increased sharply(2). If this increased usage continues, the city will have to build a costly new power plant(3). Obviously the best way to avoid this expense is to increase reimbursement(4) to residents for replacing energy-wasteful equipment(1,5). This will reduce energy usage to the levels of five years ago(6)." ★★

1. The author fails to substantiate the critical assumption that there are still significant amount of such devices could be replaced. (U.A)

★★★★★

2. The author simply assumes that it is those energy-wasteful devices that resulted in the increased electricity consumption. (NCR) ★

★★★

3. The author fails to provide information about the capacity of our existing power plant(s) and actual amount of electricity

consumption, thus we cannot evaluate if the new power plant will be necessary. (V.D/necessity of the solution) ★★★

4. We do not know how much money would be spent for the reimbursement to reduce energy usage to former levels, maybe a new

power plant will be more economically efficient if the reimbursement is very expensive. (adv:disadv/lack of comparison) ★★★★

5. The author ignores other solution to reduce energy usage. (necessity of the solution) ★★

6. Granted that the author?s proposal is necessary, we cannot ensure that energy consumption will decline to the levels of five years

ago. (U.A) ★★★

——————————————————————————————

123, (保健/建议/并列)The following appeared in a health newsletter.

"Eating a heavy meal may increase the risk of heart attack. A recent survey of 2,000 people(1) who had had a heart attack revealed that 158 of them(2) said they had eaten a heavy meal within 24 hours before their heart attack, and 25 of them said they had eaten a heavy meal within 2 hours before their heart attack(3). Eating and digesting food releases hormones into the bloodstream and temporarily increases heart rate and 44

blood pressure slightly(4). Both of these things put stress on the heart. Therefore, people who are at risk of having a heart attack can lower that risk by not overeating(5)." ★★★

1. The result of the survey lacks reliability because 2,000 people could constitute only a very limited sample. (quantity of the sample)

★★★

2. 158 out of 2,000 people are so insignificant a proportion that could hardly be statistically representative. (selective sample) ★★★

3. The author unfairly assumes that it is the heavy meals that resulted in the heart attack of people cited in the survey. (post hoc, ergo

propter hoc) ★★★★

4. The author does not inform us to what extent will overeating increase heart rate and blood pressure, and if the increase will

sufficiently cause heart attack. (I.T) ★★★★

5. Since heart attack may result from many factors, the author?s assumption that avoiding overeating would sufficiently lower the risk

of heart attack is unwarranted. (sufficiency of the solution) ★★★

——————————————————————————————

124, (发行/决策/并列)The following is a memo from the business manager of National Daily News.

"To expand the home delivery service of our national newspaper, we should concentrate on the state of Urba rather than on the state of

Sylva(6). First, the population of Sylva is more widely dispersed, which would require us to spend more money to deliver our papers(1) in that area, resulting in less profit per customer(2). Second, a long-term study of television viewing habits(3) suggests that Sylvans prefer local to national news, since they spend twice as much time viewing local news programs as they do viewing national programs(4). Finally, because events in Urba receive more coverage in our newspaper than do events in Sylva, we can expect Urbans to be more interested in reading our newspaper(5)." ★★★

1. The manager ignores many other factors that may influence delivery costs, thus the cost we spend in Sylva would not necessarily be

higher. (I.T) ★★★

2. The total amount of potential subscribers in Sylva may be significant, thus we could still have great profits there although the profit

per customer might be low. (V.D) ★★★★

3. The television viewing habits may not necessarily indicate people?s attitude toward newspapers. (F.A) ★★★★

4. The amount of time spent on viewing news is not a good indication of whether people prefer local or national news. (U.C) ★★★

5. The fact that our newspaper covers more events in Urba does not necessarily guarantee that Urbans will be interested in our

newspaper. (sufficiency of the solution/NCR) ★★★

6. The manager ignores the negative effects of putting our concentration on Urban instead of Sylva. (adv:disadv) ★★★

——————————————————————————————

125, (学校建设/论断/类比)The following is a recommendation from the president of Appleby College.

"Whereas Appleby College holds class reunions every five years, Edelston College holds annual reunions for all classes, during which Edelston's alumni are treated to banquets, lectures, and student performances(4), enhancing their loyalty to the college and their willingness to donate money(3). Edelston College receives most of its alumni donations(1) during or shortly after these reunions(2). Therefore, the best way(8) for Appleby to increase its alumni donations(6) is to offer similar reunion activities(5) and to have each graduating class hold annual reunions(7)." ★★

1. The president does not inform us the total amount of donations received by the two colleges, thus we cannot evaluate if Edelston?s

reunion activities are more effective. (I.T) ★★★★

2. Granted that Edelston did receive more donations per person than Appleby, we cannot guarantee that it is the activities mentioned

that contributed to the donation. (NCR) ★★★★★

3. The president?s assumption that Edelston receive more donations because of alumni?s loyalty and willingness to donate money is

open to doubt. Alumni may donate their money for other purpose. (NCR/U.A) ★★★

4. The president fails to consider the cost of Edelston?s reunion activities. (adv:disadv) ★★★★

5. Edelston and Appleby may not be comparable, thus merely copying Edelston?s activities may not generate the same result at

Appleby. (F.A) ★★★★

6. The president fails to substantiate a critical assumption that Appleby?s alumni could give out significantly more money if

reunions are held more frequently. (feasibility of the conclusion) ★★★

45

7. The president fails to consider if most alumni are able to or willing to attend the reunions if reunions are hold annually.

(feasibility of the conclusion) ★★★

8. The president overlooks other possible methods to increase Appleby?s alumni donations. (necessity of the solution) ★★★ ——————————————————————————————

126, (保健/建议/对比)The following appeared in a health magazine. habits found that people who reported sleeping eight or more hours a night had a higher rate of certain health problems(2) than did those who reported sleeping seven hours a night(1). People who reported sleeping five hours a night also had an increased rate of the health problems, but this increase was slight compared to that for people who reported sleeping eight or more hours a night. Clearly, people(6) should try to get seven hours(4) of sleep, and they should worry more about getting too much sleep than too little(5)." ★★★

1. The author fails to consider other possible differences between these groups of people. (I.C) ★★★★

2. For those people who sleep eight or more hours, there is a possibility that it is their health problems that caused them to sleep more

hours. (confusing the cause and the effect) ★★★★★

3. The lasting time of the study is not long enough, thus many problems related to less sleep might not have emerged yet. (procedure of

the survey) ★★★★

4. The evidence cited by the author could not sufficiently illustrate that seven hours of sleep is the best. (U.A) ★★★

5. Getting too little sleep might have significant negative effects on health although the increase rate of problems caused by less sleep

might be slight. (I.T) ★★★

6. The author fails to consider that people of different age might need different amount of sleep. (C.S) ★★★★

——————————————————————————————

127, (管理/提议/——)The vice president for human resources at Climpson Industries sent the following recommendation to the company's president.

"In an effort to improve our employees' productivity(1), we should implement electronic monitoring of employees' Internet use from their workstations. Employees who use the Internet from their workstations(3) need to be identified and punished if we are to reduce the number of work hours spent on personal or recreational activities(2), such as shopping or playing games(4). By installing software(6) to detect employees' Internet use on company computers, we can prevent employees from wasting time, foster a better work ethic(5) at Climpson, and improve our overall profits(7)." ★★★

1. The vice president fails to provide any information about current performance and productivity of our employees, thus the

assumption that their productivity should be and could be improved is open to doubt. (I.I/U.A/necessity of the solution) ★★

2. The vice president unfairly assumes that most of our employees use internet for personal or recreational activities. (U.A/I.I/necessity

of the solution) ★★★

3. The vice president ignores the necessity for some employees to use the Internet in their work. (adv:disadv) ★★★★

4. Employees could still spend much time on personal or recreational activities even though the Internet access is banned. (sufficiency

of the solution) ★★★★★

5. The vice president overlooks the negative effects of the proposed activity on employees? morale, thus the better work ethic

mentioned above is not guaranteed. (adv:disadv) ★★★★★

6. The vice president fails to consider possible negative effects of installing such software on the performance of computers, and fails

to provide the cost of installing the software. (adv:disadv) ★★★

7. Since Climpson?s profits would be determined by many factors, adopting the proposal may not sufficiently ensure greater profits.

(sufficiency of the solution) ★★★

——————————————————————————————

128, (社会/提议/递进)The following appeared in an editorial in the Garden City Gazette. pedestrian plaza is an important attraction that draws people to the downtown area, improvements to it will increase business for downtown merchants(3). The merchants' higher profits(4) will ultimately produce increased tax revenues for the city. Therefore, we should invest in the plaza improvements first and then use the revenues thus generated to pay for the construction of the parking garage(5)." ★★★

1. The profits brought by the construction of the parking garage might be even higher than that brought by the plaza improvements. 46

(adv:disadv/lack of comparison) ★★★★

2. The parking garage could generate revenue through parking fees, thus the project might have little effect on the city?s budget. (I.T)

★★★

3. The author unfairly assumes that the improvements of the plaza will necessarily attract more people. (U.A/sufficiency of the solution)

★★★

4. People attracted by the new plaza would not necessarily make purchases here, thus higher profits could not be guaranteed. (I.T/U.A)

★★★

5. The author fails to consider the negative effects of the deferment of constructing the parking garage. (adv:disadv) ★★★★

——————————————————————————————

129, (保健/建议/并列+对比)The following appeared in the Sherwood Times newspaper.

"A recent study reported that pet owners have longer, healthier lives on average than do people who own no pets(1,2). Specifically, dog owners tend to have a lower incidence of heart disease. In light of these findings, Sherwood Hospital should form a partnership with Sherwood Animal Shelter to institute an 'adopt-a-dog' program(3,5). The program would encourage dog ownership for patients recovering from heart disease(4,7), which will help reduce medical costs(6) by reducing the number of these patients needing ongoing treatment. In addition, the publicity about the program will encourage more people(9) to adopt pets(8) from the shelter, which will reduce the risk of heart disease in the general population(10)." ★★60,114

1. No causal relationship between owning pets and healthier lives of the pet owners is established. (confusing concurrence with

causality/confusing the cause and the effect) ★★★★★

2. There may be other differences between people who own pets and those who do not. (I.C) ★★★★

3. Pets could cause other health problems. (adv:disadv) ★★★★

4. The author fails to illustrate that owning a dog would have the same positive effects on recovery of heart disease as it has on

preventing heart disease. (C.S) ★★★

5. Some patients may not be willing to adopt a dog, or capable of owning dogs, or could afford raising a dog. (feasibility of the

conclusion) ★★★

6. The patients may suffer form other health problems, thus their medical expenses would not necessarily decrease even if owning a

dog could indeed lower the risk of heart disease. (sufficiency of the solution/U.A) ★★★

7. The author unfairly assumes that at least a significant number of people who adopt pets from the shelter will have risks of heart

disease. (U.A) ★★

8. The author fails to illustrate that adopting other pets would have the same effect on preventing heart disease as owning a dog has.

(C.S) ★★★

9. The author hastily assumes that the proposed program would be appealing to the general public. (feasibility of the conclusion)

★★★

10. Since the risk of heart disease in the general population might be influenced by many factors, implementing the program alone

may not sufficiently reduce the risk. (sufficiency of the solution/U.A) ★★★

——————————————————————————————

130, (家庭生活/预言/大并列小递进)The following appeared in the annual report of a chain of stores that sell supplies for do-it-yourself home improvements.

"Current reports indicate that sales of new homes nationwide declined by 9 percent during the past year, while sales of existing homes increased by 16 percent(1,2). Thus we can expect that the percentage of homeowners who will be making repairs or improvements to their homes(3) will also increase. In addition, people who buy existing homes will have more money left over(4) after the purchase to spend on home

improvements. Finally, people who buy existing homes are more likely to make improvements themselves(5), as opposed to paying someone else. These factors combined indicate we can expect an increasing demand for products in all(6) our stores(7) nationwide, thus increasing our profits in the coming year(8)." ★★

1. The author simply assumes that the trends of sales of homes will continue in the coming year. (P→F) ★★★

2. The author fails to inform us the total amount of new homes and existing homes respectively, maybe the increase in the sales of

existing homes is insignificant. (V.D) ★★★

3. The author?s assumption that most existing homes sold last year need repairs or improvements, and that most homeowners of

such homes are willing to make repairs and improvements is open to doubt.. (U.A/I.I) ★★★

47

4. The author fails to provide the prices of new homes and existing homes, and the financial status of people who buy the two kinds

of homes, thus the assumption that people who buy existing homes will have more spare money is unwarranted. (I.I/U.A) ★★★

5. No evidence could imply that people who buy existing homes are necessarily interested in DIY works. (U.C) ★★★

6. The argument fails to rule out the possibility that the demand for DIY products will not increase in some regions. (C.S) ★★★

7. The evidence cited above, if true, does not indicate that homeowners will choose products of the author?s stores. (I.E) ★★★

8. The profits of the stores would be influenced by many factors, thus the conclusion that their profits will necessarily increase is

doubtful. (I.T) ★★★

——————————————————————————————

131, (环境/提议/对比)The following appeared in an environmental newsletter published in Tria Island. In contrast, the marine sanctuary on Omni Island has regulations that ban dumping, offshore oil drilling, and fishing within 10 miles of Omni and Omni reports no significant decline in its fish populations(1). Clearly, the decline in fish populations in Tria's waters is the result of overfishing, not pollution(2). Therefore, the best way(4) to restore Tria's fish populations and to protect all of Tria's marine wildlife(5) is to abandon our regulations and adopt those of Omni(3)." ★★★

1. Many other factors might be different between Omni and Tria. (I.C) ★★★★

2. The author provides no direct evidence to prove that pollution is not responsible for the decline in fish population, and that

overfishing is the actual cause. (NCR/I.E) ★★★★★

3. Copying Omni?s regulations indiscriminatingly to Tria may not generate the same result. (F.A) ★★★★

4. The author ignores other possible and better solution to restore fish population. (sufficiency of the solution) ★★★

5. The author hastily assumes that the proposed actions will also successfully protect all other marine wildlife. (C.S) ★★★ ——————————————————————————————

132, (管理,健康/提议/对比)The following appeared in an editorial in the local newspaper of Workville.

"Workers should be allowed to reduce their workload from 40 to 25 or even 20 hours per week because it is clear that people who work

part-time(8) instead of full-time have better health and improved morale(1). One store(3) in Workville, which began allowing its employees to work part-time last year, reports that fewer(4) days of sick leave(2) were taken last year than in previous years. In contrast, the factory in

Workville(3), which does not allow any of its employees to work part-time, had a slight increase(4) in the number of days of sick leave taken last year(5,6). In addition, a recent survey reports that most of the store employees stated that they are satisfied with their jobs, while many of the factory employees stated that they are dissatisfied with their jobs(7)." ★★

1. The author fails to illustrate that the better health and improved morale are because of their part-time work. (NCR) ★★★

2. Alternative explanations could also explain why the number of days of sick leave declined last year in the store. (NCR/U.C) ★★

★★

3. Merely one store and one factory could not be representative of all business and industries in Workville. (quantity of the sample)

★★★★

4. No data concerning the total number of days of sick leave at the store and the factory is provided, thus the number of sick leaves

at the store, although declined, might still be higher than that at the factory. (confusing comparison with variation) ★★★★★

5. Alternative explanations could also explain why the number of days of sick leave increased last year in the factory. (NCR/U.C)

★★★★

6. There may be many other differences between the situations of the store and the factory. (I.C/F.A) ★★★

7. The author fails to illustrate that employees? satisfaction mentioned above is related to their workload. (U.A) ★★★★★

8. Many workers may not prefer part-time for some reasons. (feasibility of the conclusion) ★★★★

——————————————————————————————

133, (教育/决策/类比+并列)The following is a memo from the principal of Academia High School.

"Academia High School should abolish its after-school performing-arts programs and replace them with computer-technology programs(8). When nearby Techno High School did so last year, total enrollment in all of its after-school activities remained about the same. Moreover, on entering college(7), many(3) Techno students chose a major directly related to their after-school activities(1,2,4,5). On the other hand, last year 48

only 10 percent of Academia's graduating seniors chose performing arts as their major field of study in their first year of college(7), clearly indicating that most students do not have a strong interest in the performing arts(1,6)." ★★236

1. What major do students choose on entering college is not a good indication of what kind of after-school programs are they interested

in. (U.C) ★★★★

2. Many Techno students may already have interests in the major field they chose later on before they attending high school.

(confusing the cause and the effect) ★★★★

3. The principal does not inform us what percent of all Techno graduating seniors chose a major directly related to their

after-school activities. (V.D) ★★★

4. The principal fails to inform us that among those students who chose computer-technology as their after-school activities, how

many of them chose computer-technology as their major on entering college.. (V.D) ★★★

5. The principal fails to establish a causal relationship between offering computer-technology program at Techno and the fact that

many Techno students chose a major directly related to their after-school activities. (NCR/U.C) ★★★

6. Other differences between the two schools might also explain the differences in their graduating seniors? choice. (F.A/I.C) ★★

★★★

7. First year?s choices at college could not necessarily indicate students? later choices. (U.C) ★★★

8. The principal fails to consider Academia students? attitude toward after-school programs, and possible negative effects of

abolishing performing-arts programs. (adv:disadv) ★★★★

——————————————————————————————

134, (著作出版/建议/并列)The following was taken from a guide for aspiring writers. mystery novels published in the last two years has grown significantly, and more people(2) read mysteries than any other type of novel. In addition, almost half of the mystery novels published last year were written by first-time novel writers(3,4). Since there is apparently an expanding market for mystery novels(1), all(5) publishers will want to increase the number of mystery novels they publish. Therefore, new writers(4) should write mystery novels to increase their chances for first-time publication with a larger, prestigious company(6,7)." ★★★

1. The author simply assumes that the trend of increasing number of mystery novels published will continue in the future. (P→F) ★

★★★

2. The author fails to inform us the actual number of people who prefer mystery novels. (V.D) ★★

3. The author fails to provide information concerning the actual sales of mystery novels written by first-time novel writers. (I.T) ★

★★

4. The argument simply equates first-time novel writers with new writers. (U.C) ★★★

5. The author?s assumption that all publishers will be interested in publishing mystery novels is unwarranted. (C.S) ★★★★

6. No evidence could illustrate that large, prestigious publishers would accept mystery novels written by new writers. (U.A) ★★

★★★

7. The author ignores other possible, maybe better ways to increase the chances for first-time publication of new writers.(necessity

of the solution) ★★★

——————————————————————————————

135, (传媒/决策/并列)The following is a memorandum from the business manager of WLSS television station.

"Over the past year, our late-night news program has devoted increasingly more time to covering national news and less time to covering

weather and local news. During the same time period, most of the complaints(1,4) we received from viewers were concerned with the station's coverage of weather and local news(2,3). In addition, several local businesses that used to run advertisements during our late-night news program have just cancelled their advertising contracts with us(5). Therefore, in order to attract more viewers to our news programs and to avoid losing any further advertising revenues, we should expand the coverage of weather and local news on all(7) our news programs(6)." ★★★173

1. The manager fails to provide the total number of complaints received before our coverage of news were changed. (confusing

comparison with variation/V.D) ★★★

2. The manager simply assumes that those complaints are representative of the opinions of our entire audience. (C.S) ★★★

3. The manager fails to analyze the actual attitude of our audience toward the late-night news program, the program might be welcome 49

although there are a few complaints about it. (I.I) ★★★★

4. The manager fails to inform us the details of the complaints we received, thus we cannot evaluate if these audience are dissatisfied

with the quantity of weather and local news. (I.I) ★★★★

5. The manager unjustifiably assumes that it is the change in our coverage of news that caused those businesses to cancel their

contract with us. (NCR) ★★★★

6. The manager ignores many other possible methods to attract more viewers and to increase advertising revenues. (necessity of the

solution) ★★★

7. The necessity of expanding the coverage of weather and local news on all our news programs is open to doubt. (C.S) ★★★ ——————————————————————————————

136, (学校建设/提议/——)The following is part of a letter from the Brookfree School Board. however, another group of consultants determined that several schools in our district had problems because their principals were

inexperienced(1,2). Thus, if we want to see immediate improvement(4) in our schools and save the district the money it would spend on hiring more consultants(6), we should replace all the principals in the ten worst schools with ten of our most experienced principals in the

district(3,4)." ★★★

1. The author fails to provide information concerning the current situation of the schools which were identified by the consultants as

having problems two years ago. (I.T) ★★

2. The consultants we hired two years ago failed to provide any substantial evidence to illustrate that those inexperienced principals

were responsible for the problems of their schools, and that they did not have the ability to solve these problems. (NCR/credibility of the evidence/I.E) ★★★★

3. The author simply assumes that the problems of the ten schools are also caused by their principals? experience. (U.A) ★★★★★

4. Merely changing principals in the worst schools might not sufficiently resolve the problems and bring forth immediate

improvements. (sufficiency of the solution) ★★★

5. The proposed replacement might cause some negative effects which are not sufficiently considered by the author. (adv:disadv) ★★

6. The problems emerged two years ago might be different from problems we currently face, thus additional consultants and new

solutions might still be necessary. (P→C) ★★★★

——————————————————————————————

137, (城市,生活休闲/预言/递进+并列)The following appeared in an editorial in the Mason City newspaper.

"At present, Mason City residents seldom use the nearby Mason River for any kind of recreational activity, even though surveys of the region's residents consistently rank water sports (swimming, fishing, and boating) as a favorite form of recreation. Since there have been complaints about the quality of the water in the river, residents must be avoiding the river because they think that it is not clean enough(1). But that situation is about to change: the agency responsible for rivers in our region has announced plans to clean up Mason River(2). Therefore, recreational use of the river is likely to increase(3), so the Mason City council will need to increase its budget for improvements(5) to the publicly owned lands along the Mason River(4)." ★★

1. The author simply assumes that it is the quality of the water in Mason River that prevents residents from using it for recreational

activities. (NCR) ★★★★

2. No evidence could illustrate the effectiveness of the proposed plan, and how much time would it take for the plan to be effective,

thus we cannot ensure that recreational use of the river will automatically increase. (I.I) ★★★★★

3. The author fails to consider if Mason River is suitable for those water sports favored by residents in Mason, and if residents are

willing to use the river for recreational activities. (feasibility of the conclusion) ★★★★

4. Granted that recreational use of the river is to increase, we are not convinced that the use of the publicly owned lands along the river

will also increase. (U.A/H.G) ★★★

5. The author does not provide evidence to illustrate the necessity of increasing budget for improvement s to the public lands.

(necessity of the solution) ★★★

——————————————————————————————

138, (交通运输/决策/递进)The following report appeared in a memo from the vice president of the Southside Transportation Authority. 50

"We should abandon our current five-year plan to purchase additional buses to serve the campus of Southside University, because students there are unlikely to use them. Consider the results of the recent campaign sponsored by the Environmental Club at Southside University: in a program on the campus radio station, the club asked students to call in and pledge that they would commute to school by bus instead of by automobile at least one day per week. Only ten percent of the students called in and pledged(1,3). In view of the campaign's lack of success, we can assume that the bus service we currently offer will continue to be sufficient to serve the university(2,4)." ★★★★

1. The fact that only 10 percent of the students called in and pledged in the campaign does not indicate that few students are willing to

use buses. (F.D) ★★★★

2. The facts cited by the vice president do not prove that the bus service we currently offer could sufficiently meet students? demand.

(I.E) ★★★★

3. The vice president fails to rule out the possibility that it is the shortage of bus service that cause many students abandoned using

buses. (negative evidence) ★★★

4. The vice president fails to consider many factors that could increase the demand for bus services in the future. (C→F) ★★★★

——————————————————————————————

139, (教育/提议/先对比后递进)The following appeared in the editorial section of an educational publication.

"One study at Lee University(2) found that first-semester grades of teenage students who had always attended public, tax-supported schools were slightly lower than the grades of students who had received some home schooling instruction by parents at home(1,3), although the grade differences disappeared in the second semester(4). These results suggest that home schooling is the best(5) way to educate teenage children. Therefore, instead of spending more money on public education, the government should provide financial incentives so that home schooling is an option for more parents(6). After all, children schooled at home receive more attention, since they are taught by the best possible teacher: a parent who has a high stake in educating them well(7)." ★★★

1. The author fails to consider other differences between the two groups of students. (I.C) ★★★★

2. The result of the study at one university might not be representative of all teenage students. (quantity of the sample) ★★★★

3. No direct evidence is provided to show that it is the home schooling that resulted in the higher grades of students who received it.

(NCR) ★★★★

4. The fact that the grade differences disappeared in the second semester might well illustrate that home schooling was not responsible

for the higher grades. (negative evidence) ★★★

5. The author ignores other better ways to educate teenage children. (necessity of the solution) ★★★

6. The author assumes without any guarantee that the incentives will be effective and most parents are willing to respond to the

policy. (feasibility of the conclusion) ★★★

7. The author fails to consider if some parents are capable of being effective tutors, merely more attention does not equal better

education. (U.C) ★★★

——————————————————————————————

140, (人事/提议/并列)The following appeared in a report of the Committee on Faculty Promotions and Salaries at Elm City University. "During her seventeen years as a professor of botany, Professor Thomas has proved herself to be well worth her annual salary of $50,000. Her classes are among the largest at the university(1), demonstrating her popularity among students. Moreover, the money she has brought to the university in research grants(2) has exceeded her salary in each of the last two years(3). Therefore, in consideration of Professor Thomas' demonstrated teaching and research abilities, we recommend that she receive a $10,000 raise and a promotion to Department Chairperson(4); without such a raise and promotion(7), we fear that Professor Thomas will leave Elm City University for another college(5,6,8)." ★★26,85,209,225

1. The size of the classes could hardly be a good indication of the popularity of a professor among students. (U.C) ★★★★

2. The amount of research grants brought by a professor could not indicate the research abilities of that professor. (U.C) ★★★★

3. The committee simply assumes that research grants brought by Prof. Thomas will continue to exceed her salary in the future. (P→F)

★★★

4. Granted that Prof. Thomas has demonstrated excellent teaching and research abilities, we cannot guarantee that she will also be

highly competent as a chairperson. (H.G) ★★★★

5. The committee fails to demonstrate that Prof. Thomas has other choices currently, and that she is willing to teach at another

university. (U.A) ★★★

51

6. The committee unfairly assumes that Prof. Thomas will leave for sure if we do not provide such a raise and promotion. (U.A) ★

7. Other measures could also be used to keep Prof. Thomas staying at Elm City University. (necessity of the solution) ★★★

8. The committee fails to illustrate that the proposed promotion and salary raise are sufficiently attractive for Prof. Thomas.

(sufficiency of the solution) ★★★

——————————————————————————————

141, (环境/对策/递进)The following appeared in a newsletter distributed at a recent political rally.

"Over the past year, the Consolidated Copper Company (CCC) has purchased over one million square miles of land(4,5) in the tropical nation of West Fredonia. Mining copper on this land will inevitably result in pollution and environmental disaster(3,8), since West Fredonia is home to several endangered animal species. But such disaster can be prevented if consumers simply refuse to purchase(2) products that are made with CCC's copper(1) until the company abandons its mining plans(6,7)." ★★

1. The author fails to demonstrate the critical assumption that most consumers can reliably distinguish products that are made with

CCC?s copper. (feasibility of the conclusion) ★★★★

2. The author fails to consider if most consumers are willing to cooperate in the boycott. (feasibility of the conclusion) ★★★★

3. CCC?s mining activities do not necessarily lead to pollution and environmental disaster. (NCR/I.I) ★★★★★

4. We do not know what portion of CCC?s land is inhabited by endangered animals. (I.I) ★★★

5. We do not know what fraction of the one hundred square miles of land will be used for CCC?s mining activities. (V.D) ★★★

6. The author ignores the possibility that other measures could also be taken to prevent the harmful result. (necessity of the solution)

★★★

7. The author hastily assumes that the proposed boycott will sufficiently prevent pollution and environmental disaster. (sufficency

of the solution) ★★★

8. The definition of the term disaster is very vague, thus we cannot evaluate if the disastrous results will necessarily emerge.

(definition of the term ?disaster?) ★★

——————————————————————————————

142, (保健/论断/并列)The article entitled 'Eating Iron' in last month's issue of Eating for Health reported that a recent study(1) found a correlation(2) between high levels of iron in the diet and an increased risk of heart disease. Further, it is well established that there is a link(3) between large amounts of red meat in the diet and heart disease, and red meat is high in iron. On the basis of the study and the well-established link between red meat and heart disease, we can conclude that the correlation between high iron levels and heart disease, then, is most probably a function of the correlation between red meat and heart disease(4,5). ★★★★

1. The reliability of the result of the study is open to doubt. (reliability of the survey) ★★

2. Merely a correlation between high levels of iron in the diet does not necessarily indicate a causal relationship between them.

(confusing concurrence with causality) ★★★★

3. The author fails to provide evidence to substantiate the causal relationship between eating much red meat and heart disease.

(confusing concurrence with causality) ★★★★

4. The author unfairly assumes that all people who have risk of heart disease would necessarily eating large amounts of red meat.

(U.A) ★★★

5. High iron levels in the diet may result from many other factors, not only from red meat. (I.T) ★★★★

——————————————————————————————

143, (就业/论断/并列)The following appeared as a letter to the editor of a national newspaper.

"Your recent article on corporate downsizing* in the United States is misleading(8). The article gives the mistaken impression that many competent workers who lost jobs as a result of downsizing face serious economic hardship, often for years, before finding other suitable employment. But this impression is contradicted by a recent report on the United States economy, which found that since 1992 far more

jobs(1,2) have been created than have been eliminated. The report also demonstrates that many(3) of those who lost their jobs have found new employment(4). Two-thirds of the newly created jobs have been in industries that tend to pay above-average wages(6), and the vast majority of these jobs are full-time(5)." ★★★183

*Downsizing is the process in which corporations deliberately reduce the number of their employees.

1. The author does not provide the actual number of new jobs created, perhaps the number is still lower than the total number of

laid-off workers although it is higher than the number of jobs eliminated. (V.D) ★★★

52

2. The author fails to demonstrate that the newly created jobs since 1992 are suitable for those workers downsized by corporations.

(I.I/feasibility of the conclusion) ★★★★

3. The report does not clearly demonstrate what fraction of workers who lost their jobs have found new employment. (V.D) ★★★

4. We are not informed what kind of new employment did those workers find, and if these jobs are high-paying jobs. (I.T) ★★★

5. The author fails to inform us how many laid-off workers engaged in those highly paid and full-time jobs mentioned by the author.

(I.I) ★★★

6. The fact that many new jobs are in industries that tend to pay high wages does not necessarily guarantee that these new jobs are also

highly paid. (U.C) ★★★

7. The statistics cited by the author still does not rule out the possibility that many laid-off worker do face serious economic

hardship before they find a new job. (I.E) ★★★

——————————————————————————————

144, (社会/论断/对比)According to a poll(1) of 200 charitable organizations, donations of money to nonprofit groups increased by nearly 25 percent last year, though not all charities gained equally. Religious groups gained the most (30 percent), followed by environmental groups (23 percent), whereas educational institutions experienced only a very small increase in donations (3 percent)(2,3). This poll indicates that more people(5) are willing(6) and able to give money to charities but that funding for education is not a priority for most people. These differences in donation rates must result from the perception(4) that educational institutions are less in need of donations than are other kinds of institutions. ★★★199

1. The size of the sample, procedure of sampling could be cast doubt on. (reliability of the survey) ★★

2. We cannot evaluate the statistics provided by the author because we do not know the base amount of donations each group received

the year before last. (V.D) ★★★★

3. The amount of donations received by educational institutions is not necessarily lower than others since they may receive donations

from many other ways. (I.T) ★★★★★

4. No evidence is provided to show that it is the people?s perception suggested by the author that resulted in the differences in donation

rates. (NCR) ★★★★

5. We cannot infer from the argument that more people are giving money to charities. (I.I/U.A) ★★★

6. People may donate their money for other purpose, thus the assumption that people are willing to give money is open to doubt.

(I.T/U.A) ★★

——————————————————————————————

145, (生命科学,保健/建议/递进)condition called sleep apnea. The interruption of breathing wakes the person—often so briefly that the waking goes unnoticed—and can leave the person too tired(3) during the day to exercise(2,8). Anyone(4) who snores, therefore, should try to eat less(6) than the average person and to exercise more(5,7). ★★

1. We do not know what fraction of people who snore actually suffer from sleep apnea. (V.D) ★★★

2. People who suffer from sleep apnea may actually exercise during the day although they are tired. (U.A) ★★

3. The author fails to illustrate the actual effect of sleep apnea, thus the assumption that people who suffer from it will necessarily be

tired is unwarranted. (I.I/U.A) ★★

4. The assumption that all people who snore will necessarily eat more and exercise less than normal is unwarranted. (C.S/U.A) ★★

★★

5. The author?s assumption that people who snore should exercise more is contradictory to the assumption that those people are too

tired to exercise. (negative evidence) ★★★

6. No evidence could illustrate that eating less could effectively result in reducing weight. (NCR) ★★★

7. The author ignores many other factors that would contribute to snore and gaining weight. (I.T/sufficiency of the solution) ★★★

8. The author fails to consider the possibility that obesity could reversely result in people snore. (confusing the cause and the effect)

★★★★★

——————————————————————————————

146, (教育/提议/递进)The following appeared in the editorial section of a local newspaper.

53

"The librarians in our town's school system have reported that the number of trips that our students make to their school library on a voluntary basis has decreased significantly in recent years. For example, the average seventh-grade student(1) visited the school library five times last year(2), but four of those visits were part of required classroom activities. This shows that our students are reading less than in the past(3). To address this problem, our town needs to improve the atmosphere of the libraries so that they will be comfortable places(4) in which to work(6). If students view the libraries as uncomfortable, then they are unlikely to want to spend much time there(5)." ★★★

1. The statistic result of seventh-grade students might not be representative of all our students. (selective sample) ★★★★

2. The author fails to illustrate that the situation last year was not an aberration and would likely to continue. (P→F) ★★★

3. The fact that the number of students? trips to library decreased does not necessarily indicate that students do not make enough

reading. (U.C) ★★★★★

4. The author simply assumes that our students make fewer trips to library because they feel the libraries uncomfortable. (U.A) ★★

★★

5. The author hastily assumes that the proposed improvements on the libraries? atmosphere would suffice to attract students to make

more trips to libraries. (sufficiency of the solution) ★★★

6. Other possible solutions could also be used to ensure students do more reading. (necessity of the solution) ★★★

——————————————————————————————

147, (文化娱乐,商业/预言/并列)The following appeared in an editorial in a business magazine.

"Although the sales of Whirlwind video games have declined(1) over the past two years, a recent survey of video-game players suggests that this sales trend is about to be reversed. The survey asked video-game players what features they thought were most important in a video game(2). According to the survey, players prefer games that provide lifelike graphics, which require the most up-to-date computers(4). Whirlwind has just introduced several such games with an extensive advertising campaign directed at people 10 to 25 years old(5), the age-group most likely to play video games. It follows, then, that the sales of Whirlwind video games are likely to increase(6) dramatically in the next few months(3)." ★★★212

1. The author fails to take into account other factors that might also lead to the decline in their sales of video games. (NCR) ★★

2. Many other questions that are directly related to the conclusion are ignored by the author. (what question was asked in the survey)

★★★★

3. The author fails to consider other features that may also influence the overall quality of a game. (I.T) ★★★

4. The author fails to analyze how many players have access to such most up-to-date computers. (I.I) ★★★

5. No evidence could indicate that players 10-25 years old will also prefer such kind of games. (C.S) ★★★

6. Sales of video games would be determined by many factors, thus the mere fact cited by the author could not ensure increase in

our sales. (sufficiency of the solution) ★★★

——————————————————————————————

148, (选举/论断/并列)The following appeared in the editorial section of Monroetown's local newspaper. most people in our town favored Mayor Brown's proposal for tax reduction over that of her opponent, Mr. Greene, who proposed raising taxes(4) to improve education. It has been shown that voters nationwide(2) tend to re-elect people already in office, regardless of candidates' proposals(1). In fact, a local survey after the election showed most people in Monroetown disagreed with Mayor Brown's proposal. Clearly most people in Monroetown favor improving education and therefore approve of Mr. Greene's proposal(3) despite the fact that they did not vote for him." ★★★7

1. The author fails to demonstrate that Mayor Brown?s re-election was not because people favored her proposal, but because of

people?s voting habit mentioned in the argument. (I.E) ★★★★★

2. The fact that voters nationwide tend to re-elect people already in office does not indicate that voters in Monroetown will also have

such behavior. (C.S) ★★★

3. Granted that people disagree with Mayor Brown?s proposal, it does not follow that they would favor improving education. (F.D)

★★★★

4. Granted that people in Monroetown favor improving education, we could not hastily infer that they would approve Mr. Greene?s

proposal which proposed education improvements through raising taxes. (U.A) ★★★

——————————————————————————————

54

149, (人事,管理/提议/类比+并列)The following is a memorandum from the director of personnel to the president of Get-Away Airlines. "Since our mechanics are responsible for inspecting and maintaining our aircraft, Get-Away Airlines should pay to send them to the

Quality-Care Seminar, a two-week seminar on proper maintenance procedures. I recommend this seminar because it is likely to be a wise

investment, given that the automobile racing industry recently reported that the performance of its maintenance crews improved markedly after their crews had attended the seminar(1). These maintenance crews perform many of the same functions as do our mechanics, including refueling and repairing engines(2). The money we spend on sending our staff to the seminar will inevitably lead to improved maintenance(3) and thus to greater customer satisfaction(4) along with greater profits for our airline(5)." ★★★

1. The author fails to establish the causal relationship between attending the seminar and improved performance of maintenance

crews in automobile racing industry. (NCR) ★★★★

2. The seminar may not be also effective for Get-Away mechanics, since there are many differences between the maintenance

procedures of automobiles and airplanes (F.A) ★★★★

3. The quality of maintenance might be affected by many factors, thus the assumption that attending the seminar will inevitably

lead to improved maintenance is not guaranteed. (sufficiency of the solution) ★★★★

4. Merely improving maintenance might not suffice to increase customer satisfaction. (sufficiency of the solution) ★★★★

5. The profits of a company would be influenced by many factors, thus the airline?s profits would not necessarily increase.

(sufficiency of the solution) ★★★★

——————————————————————————————

150, (环境/论断/对比)The following is a letter to the editor of an environmental magazine.

"The decline in the numbers of amphibians worldwide clearly indicates the global pollution of water and air(1,4,5). Two studies of amphibians in Yosemite National Park(2) in California confirm my conclusion. In 1915 there were seven species of amphibians in the park, and there were abundant numbers of each species. However, in 1992 there were only four species of amphibians observed in the park, and the numbers of does not explain the worldwide decline." ★★★★207

1. The author does not provide information concerning the level of pollution at different regions, thus the assumption that pollution

is responsible for the decline in the numbers of amphibians everywhere is unwarranted. (I.I/C.S) ★★★★

2. A specific case in Yosemite National Park could hardly be representative of the situation throughout the world. (quantity of the

sample) ★★★

3. The argument fails to rule out the introduction of trout as a cause for the decline in the numbers of amphibians despite it could

not explain the worldwide trend. (I.E) ★★★

4. The author provides no direct evidence to show that it is the pollution that resulted in the decline in the number of amphibians.

(NCR) ★★★

5. The author ignores factors other than the pollution and the introduction of trout which could explain the decline in amphibian

numbers in Yosemite. (F.D) ★★★★

——————————————————————————————

151, (社会/论断/对比)The following is a letter to the editor of the Atticus City newspaper.

"Former Mayor Durant owes an apology to the city of Atticus. Both the damage to the River Bridge, which connects Atticus to Hartley, and the traffic problems(2) we have long experienced on the bridge were actually caused 20 years ago by Durant. After all, he is the one who approved the construction of the bridge. If he had approved a wider and better-designed bridge(1), on which approximately the same amount of public money would have been spent, none of the damage or problems would have occurred(3). Instead, the River Bridge has deteriorated far more rapidly over the past 20 years than has the much longer Derby Bridge up the river(4). Even though the winters have been severe in the past several years, this is no excuse for the negligence and wastefulness(5) of Durant(6)." ★★★

1. The author fails to illustrate that there has been any better design available at that time. (credibility of the evidence) ★★★★

2. There may be no any problem with the bridge when it was constructed. (C→P) ★★

3. The assumption that all those problems would have been avoided if Durant had approved a wider and better-designed bridge is

unwarranted. (U.A) ★★★

4. Maybe the River Bridge was more heavily used than the Derby Bridge during the past 20 years. (I.C) ★★★

5. The arguer ignores the positive effects of the construction of River Bridge when considering whether the construction is wasteful 55

or not. (adv:disadv) ★★★

6. There are many other possible causes that could result in the bad condition of River Bridge, thus there?s no reason to blame

Mayor Durant alone. (I.T) ★★★

——————————————————————————————

152, (城市/提议/并列)The following is a letter to the head of the tourism bureau on the island of Tria.

"Erosion of beach sand along the shores of Tria Island is a serious threat to our island and our tourist industry. In order to stop the erosion(1), we should charge people for using the beaches(2,3,5). Although this solution may annoy a few tourists in the short term, it will reduce the number of people using the beaches and will raise money for replenishing the sand(4). Replenishing the sand, as was done to protect buildings on the nearby island of Batia(7), will help protect buildings along our shores(6), thereby reducing these buildings' risk of additional damage from severe storms(8). And since the areas along the shore will be more attractive as a result(9), the beaches will be preserved and the area's tourist industry will improve over the long term(10)." ★★★

1. The author fails to illustrate the extent of erosion in Tria, thus we cannot evaluate if any measures are necessary to resolve the

problem. (necessity of the solution) ★★★

2. The author ignores other possible methods to stop the erosion. (necessity of the solution) ★★

3. The author fails to illustrate that charging people for using the beach could effectively stop the erosion. (sufficiency of the

solution) ★★★

4. The author fails to analyze how much money should we charge to raise enough money for replenishing the sand, and if the

charge is feasible. (I.I/feasibility of the conclusion) ★★★

5. The author ignores some negative effects of the charging policy. (adv:disadv) ★★★★

6. The author fails to demonstrate if the buildings alone our shores need to be protected. (necessity of the solution) ★★★

7. The fact that replenishing beach sand has served to protect shoreline buildings on nearby Batia does not indicate that Tria would also

achieve its goals by following Batia's example. (F.A) ★★★

8. The author unfairly assumes that replenishing sand would suffice to reduce the building?s risk of damage from severe storms.

(sufficiency of the solution) ★★★

9. The assumption that the proposed actions will necessarily make the seashore more attractive is open to doubt. (U.A/sufficiency

of the solution) ★★★★

10. Adopting the actions proposed by the author might not sufficiently improve Tria?s tourist in the long term. (sufficiency of the

solution) ★★★

——————————————————————————————

153, (社会,教育/对策/并列)The following is from an editorial in the Midvale Observer, a local newspaper.

"Ever since the 1950's, when television sets began to appear in the average home, the rate of crimes committed by teenagers in the country of Alta has steadily increased. This increase in teenage crime parallels the increase in violence shown on television(1,5). According to several national studies, even very young children who watch a great number of television shows featuring violent scenes display more violent

behavior(2,3) within their home environment than do children who do not watch violent shows. Furthermore, in a survey conducted by the Observer, over 90 percent of the respondents(4) were parents who indicated that prime-time television—programs that are shown between 7 p.m. and 9 p.m.—should show less violence. Therefore, in order to lower the rate of teenage crime in Alta, television viewers should demand that television programmers reduce the amount of violence shown during prime time(6,7)." ★★

1. The author fails to establish a causal relationship between the increase in violence shown on TV and the increase in teenage

crime. (confusing concurrence with causality) ★★★★★

2. The author fails to define the term violent behavior, thus we cannot evaluate if watching a great number of TV shows featuring

violent scenes has any relation with teenage crime. (U.C/definition of the term ?violent behavior?) ★★★

3. There exists the possibility that those children watch many television shows featuring violent scenes because they have inborn

tendency of violence. (confusing the cause and the effect) ★★★

4. We do not know what kind of people responded to the survey, thus the respondents? opinions may not represent those of all

citizens. (are the respondents representative?) ★★★

5. We need to know how many teenagers who committed crimes frequently watch violent programs on TV, and how about the rate

compared with normal teenagers. (lack of comparison) ★★★★

6. Merely reducing the amount of violence shown during prime time may not sufficiently lower the rate of teenage crime. 56

(sufficiency of the solution) ★★★

7. Other solutions could also be used to achieve the author?s purpose. (necessity of the solution) ★★★

——————————————————————————————

154, (保健/建议/对比)The following appeared in the editorial section of a health and fitness magazine.

"In a study of the effects of exercise on longevity, medical researchers tracked 500 middle-aged men(1,2) over a 20-year period(3). The subjects represented a variety of occupations in several different parts of the country and responded to an annual survey in which they were asked: How often and how strenuously do you exercise? Of those who responded(4), the men who reported that they engaged in vigorous outdoor exercise nearly every day lived longer than the men who reported that they exercised mildly only once or twice a week(5). Given the clear link that this study establishes between longevity and exercise, doctors should not recommend moderate exercise to their patients(6) but should instead encourage vigorous outdoor exercise on a daily basis." ★★★

1. The 500 middle-aged men could only constitute a limited sample and can hardly be representative. (quantity of the sample) ★

★★

2. The study only tracked middle-aged men, thus the effect of vigorous outdoor exercises on women is still open to question.

(selective sample/H.G) ★★★

3. A 20-year time span may not be a period which is long enough to ensure a decisive conclusion. (reliability of the survey) ★★

★★

4. The author only considers people who responded to the survey, we still do not know the relationship between exercise and

longevity in those who did not respond. (are the respondents representative?) ★★★

5. The author fails to rule out the possibility that those who engaged in vigorous outdoor exercises do so merely because they had

outstanding physique. (confusing the cause and the effect) ★★★★

6. Vigorous exercise may be beneficial for some people, but it does not follow that vigorous outdoor exercise will benefit all

individuals, especially the patients. (C.S) ★★★★★

——————————————————————————————

155, (教育/建议/并列+递进)The following appeared in a letter to the editor of a local newspaper.

"Too much emphasis is placed on the development of reading skills in elementary school. Many(2) students who are discouraged by the lonely activity of reading turn away from schoolwork merely because they are poor readers. But books recorded on audiocassette tape provide an important alternative for students at this crucial stage in their education(1), one the school board should not reject merely because of the expense involved(3). After all, many studies attest to the value(4) of allowing students to hear books read aloud; there is even evidence that students whose parents read to them(5) are even more likely to become able readers. Thus, hearing books on tape can only make students more eager to read and to learn. Therefore, the school board should encourage schools to buy books on tape and to use them in elementary education(6)." ★★★

1. No direct evidence could illustrate that hearing books on tape could indeed improve students? reading ability. (I.E) ★★★

2. The author fails to provide the actual number of dropouts who leave school because of their poor reading ability, and the fraction

of them in all students, thus we cannot evaluate if the problem is significant. (V.D/necessity of the solution) ★★★

3. The author fails to point out the actual cost of using books on tape, and if the school and parents could afford. (I.I/feasibility of

the conclusion) ★★★

4. The author fails to demonstrate what kind of value do those studies attest, thus we cannot evaluate the actual effect of books on

tape on improving students? reading ability. (I.I) ★★★★

5. Hearing books on tape may not have the same effect as having parents read to students. (U.C/F.A) ★★★★

6. The author ignores other better ways to resolve the problem. (necessity of the solution) ★★★

——————————————————————————————

156, (人事,生产/建议/——)The following is taken from an advertisement placed in a weekly business magazine by the Dickens Academy. "We(1) distributed a survey to senior management(2) at International Mega-Publishing, Inc. The result of the survey clearly indicates that many employees were well prepared in business knowledge and computer skills, but lacked interpersonal skills to interact gracefully with customers. International Mega-Publishing decided to improve customer satisfaction by sending their newly hired employees(3) to our one-day seminars. Since taking advantage of our program, International Mega-Publishing has seen a sharp increase in sales(4), an indication that the number of their disgruntled customers has declined significantly(5). Your company should hire Dickens and let us turn every employee(6) into an ambassador for your company." ★★★

57

1. The conductor of the survey is D.A itself, an institute which may have vested interest in the result of the survey. Thus the result

may not be reliable. (who conducted the survey) ★★★

2. The survey was distributed to senior management, the accuracy of their understanding towards the actual ability of their

employees is open to doubt. (selective sample) ★★★

3. The author unfairly assumes that the Mega employees who attended the seminar are employees whose work directly related to

Mega?s sales and customer relations. (U.A) ★★★

4. The causal relationship between attending the seminar and the increase in sales is doubtful. (NCR) ★★★★

5. Increasing sales do not necessarily indicate a decline in the number of dissatisfied customers. (U.C) ★★★★

6. The assumption that every employee in every company lacks or needs interpersonal skills is unwarranted. (C.S) ★★★★ ——————————————————————————————

157, (政治经济,社会/建议/并列)The following is a letter to the editor of a local newspaper.

"As a local merchant, I wish to support the development of a ski resort in the state park north of our township(2). Along with many other merchants who favor the proposal by Ski the Slopes(5), Incorporated, I would, of course, experience a growth in my business(1). But I also know how much more prosperous, not to mention lively and interesting, our community would be if tourism increased. Since the main

opposition comes from a few environmentalists* who do not even live in this community, I see no reason to give in to their views(3). The First National Bank has finally researched the project and agreed to fund it(4). As a result, I see no reason to delay development of the resort." ★★★

*Environmentalists are people who advocate the preservation of the natural environment.

1. The author unfairly assumes that the development of the ski resort will necessarily result in a growth in business and tourism.

(U.A) ★★★★

2. The author fails to consider the negative effects of developing the resort in our town. (adv:disadv) ★★★★

3. The author fails to provide substantial evidence to illustrate why we should ignore the environmentalists? opposition. (I.E) ★★

★★

4. We do not know how much fund the First National Bank will provide and if it is enough for developing the resort. (sufficiency of

the solution) ★★★

5. Granted that a ski resort is necessary to be built, the author still fails to illustrate that the proposal by Ski the Slopes is the best

project and if there is any better choice. Thus we should not hastily commence development of the resort. (necessity of the solution) ★★★

——————————————————————————————

158, (卫生健康,社会/论断/——)The Trash-Site Safety Council has recently conducted a statewide study(5) of possible harmful effects of garbage sites on the health of people living near the sites. A total of five sites and 300 people(1) were examined. The study revealed, on average, only a small statistical correlation between the proximity of homes to garbage sites and the incidence of unexplained rashes(2) among people living in these homes. Furthermore, although it is true that people living near the largest trash sites had a slightly higher incidence of the rashes, there was otherwise no correlation between the size of the garbage sites and people's health(4). Therefore, the council is pleased to announce that the current system of garbage sites does not pose a significant health hazard(3). We see no need to restrict the size of such sites in our state or to place any restrictions(6) on the number of homes built near the sites. ★★★

1. For lack of data concerning the total amount of garbage sites and people living near them in the state, the reliability of the study

is open to doubt. (quantity of the sample) ★★★

2. The argument treats a lack of proof that current system of garbage sites will pose some extent of health hazard as constituting

sufficient proof that it will not pose any significant health hazard. (do the statistics make any difference?) ★★★★★

3. The Council unfairly assumes that the current health status of people living near garbage sites result from the safety of current

system of garbage sites, not other explanations. (NCR) ★★★

4. The fact that people living near the largest trash sites had a higher incidence of the rashes contradicts the Council?s assertion that

there was no correlation between the size of the garbage sites and people?s health. (negative evidence) ★★★

5. The result of the statewide study could not sufficiently rule out the possibility that garbage sites do pose significant health hazard

in some specific regions. (C.S) ★★

6. Certain restriction on the size of garbage sites and number of homes built near the sites would still be necessary in the future

although the current system pose no significant health hazard. (C→F) ★★★★

58

——————————————————————————————

159, (社会/建议/对比)experiencing rising costs of electricity. A recent study of household electric costs in Claria found that families who cooled their houses with fans alone spent more on electricity than did families using air conditioners alone for cooling. However, those households that reported using both fans and air conditioners spent less(3) on electricity than those households that used either fans or air conditioners alone(2). Thus, the citizens of Claria should follow the study's recommendation and use both air conditioners and fans in order to save money on electricity(4). ★★★★

1. The author does not point out what fraction of citizens of Claria experienced rising costs of electricity. (V.D) ★

2. The argument does not consider other possible differences between these groups of families, e.g. the cost of electricity,

geographical factors, etc. (I.C) ★★★★

3. The author fails to establish a causal relationship between using both appliances simultaneously and the lower expenditure on

electricity. (NCR) ★★★★★

4. The author unfairly assumes that the proposed action is both sufficient and necessarily for saving money on electricity.

(sufficiency/necessity of the solution) ★★★

——————————————————————————————

160, (生命科学/提议/——)As people grow older, an enzyme known as PEP increasingly breaks down the neuropeptide chemicals involved in learning and memory(1). But now, researchers have found compounds that prevent PEP from breaking neuropeptides apart. In tests, these compounds almost completely restored lost memory in rats(2). The use of these compounds should be extended to students(3,4) who have poor memory and difficulty in concentrating(5,6)—and therefore serious problems(7) in school performance(8). Science finally has a solution for problems neither parents nor teachers could solve. ★★★

1. The author fails to illustrate that it is the breakdown of neuropeptide that results in lost memory. (NCR) ★★

2. The effect of the compound on rat memory might not necessarily take place on humans. (F.A) ★★★★

3. The author fails to consider if the compound has any side effect. (adv:disadv) ★★★★

4. The author fails to prove that the breakdown of the neuropeptide will also occur among young students. (U.A) ★★★★

5. The author unfairly assumes that it is the breakdown of neuropeptide that results in poor memory and difficulty in concentrating

among young students. (NCR) ★★★

6. Difficulty in concentrating does not necessarily have any relation with poor memory, thus the assumption that the compound will

also resolve the problem of concentrating is unwarranted. (U.C) ★★★★

7. Students? problems in school performance may result from various causes, thus using the compound alone might not sufficiently

resolve these problems, even if the compound does have significant effect on memory. (sufficiency of the solution) ★★★

8. The author ignores other better solutions to address problems in school performance. (sufficiency of the solution) ★★★ ——————————————————————————————

161, (社会/论断/对比)In a study of reading habits of Leeville citizens conducted by the University of Leeville, most respondents(1) said they preferred literary classics as reading material. However, a follow-up study(4,5) conducted by the same researchers found that the type of book most frequently checked out(2) of each of the public libraries(6) in Leeville was the mystery novel(3). Therefore, it can be concluded that the respondents in the first study had misrepresented their reading habits(7). ★★★

1. We do not know how many, and what kind of citizens responded to the survey, thus the representativeness of the result is open to

doubt. (are the respondents representative?) ★★★

2. The frequency of certain type of book being checked out from the public libraries is not a good indication of what kind of

reading material do citizens prefer. (U.C) ★★★★

3. The author does not analyze to what extent the literary classics and mystery novels mentioned in the argument overlap. (I.I) ★

★★★

4. The author fails to provide evidence to show that the result of the follow-up study is representative of all-time reading habits of

Leeville citizens. (C→F) ★★★

5. We do not know how long is the interim period between the two studies, many conditions may change after sufficient long

time.(P→C) ★★★★

6. The author unfairly assumes that the respondents in the first study borrow most of their reading materials from public libraries.

(U.A) ★★★

7. The author fails to consider other explanation that could explain the discrepancy between the respondents? answer and the result 59

of the follow-up study, e.g. if the respondents were forthright, if they correctly understood the survey?s question, etc. (necessity of the solution) ★★★

——————————————————————————————

162, (保健/建议/并列)A recent study shows that people living on the continent of North America suffer 9 times more chronic fatigue and 31 times(2) more chronic depression(1) than do people living on the continent of Asia. Interestingly, Asians, on average, eat 20 grams of soy per day(3), whereas North Americans eat virtually none. It turns out that soy contains phytochemicals called isoflavones(4), which have been found to possess disease-preventing properties. Thus, North Americans should consider eating soy on a regular basis as a way of preventing fatigue and depression(5,6).★★★187

1. The definition of fatigue and depression may be different in North America and in Asia. (I.C) ★★★

2. The rate of incidence of fatigue and depression may not reflect the actual health status of each group of people, many other

factors could explain the differences between the rates. (I.C) ★★★★

3. The author unfairly attributes the lower incidence of these problems in Asians to eating soy regularly. (NCR) ★★★★★

4. Even if eating soy is effective on preventing these problems, we cannot hastily assume that it is the isofalvones contained in soy

that result in the effectiveness. (NCR) ★

5. Granted that eating soy do benefit Asians on their health, no evidence could ensure the same good effect will also occur among

North Americans through eating soy.(F.A) ★★★

6. Eating soy may not be the only means of obtaining the substance, the author ignores many other solution to solve the problem.

(necessity of the solution) ★★★

——————————————————————————————

163, (社会,建设/提议/并列)The following is taken from the editorial section of the local newspaper in Rockingham.

"In order to save a considerable amount of money(1), Rockingham's century-old town hall should be torn down(2,3) and replaced by the larger and more energy-efficient building that some citizens have proposed. The old town hall is too small to comfortably accommodate the number of people who are employed by the town. In addition, it is very costly to heat the old hall in winter and cool it in summer(4). The new, larger building would be more energy efficient, costing less per square foot to heat and cool than the old hall(5). Furthermore, it would be possible to rent out some of the space in the new building(7), thereby generating income for the town of Rockingham(6)." ★★164

1. The author fails to consider the cost of constructing the new building, thus the assumption that the construction will save a

considerable amount of money is open to question. (adv:disadv) ★★★

2. The author fails to consider the negative effects of tearing down the old town hall. (adv:disadv) ★★★★

3. Granted that a new hall is necessary, the necessity of tearing down the old one is unwarranted. (necessity of the solution) ★★

4. Other possible measures could also be taken to solve the problems with the old town hall. (necessity of the solution) ★★★

5. The author fails to justify the assumption that the energy problems will automatically be solved once the new building is

constructed. (U.A) ★★★

6. The author fails to investigate if there are any individuals and institutions who are willing to rent the space in the new hall, and

how much rent could we receive. (feasibility of the conclusion) ★★★

7. The author does not take into account the negative effects of renting out some of the space in the new hall. (adv:disadv) ★★★ ——————————————————————————————

164, (学校建设/建议/并列)Claitown University needs both affordable housing for its students and a way to fund the building of such housing. The best solution(1) to this problem is to commission a famous architect(2) known for experimental and futuristic(3) buildings. It is common knowledge that tourists are willing to pay money to tour some of the architect's buildings, so it can be expected that tourists will want to visit(4) this new building(5). The income from the fees charged to tourists will soon cover the building costs. Furthermore, such a building will attract new students as well as donations from alumni(6). And even though such a building will be much larger than our current need for student housing, part of the building can be used as office space(7,8). ★★★163

1. The author ignores other solutions to solve the problem. (necessity of the solution) ★★★

2. The cost of commissioning a famous architect would be expensive, thus the income of Claitown University will increase would

not necessarily increase. (adv:disadv) ★★★

3. Granted that a new dorm is inevitably necessary, if the building should be experimental and futuristic style is open to doubt. The

author fails to consider if the unique building could harmoniously coexist with our existing buildings. (U.A/feasibility of the 60

conclusion/adv:disadv) ★★★

4. The author?s assumption that a significant number of tourist will be willing to pay to visit a students? dormitory is unwarranted.

(U.A) ★★★★

5. The author?s fails to consider the negative effect of attracting tourists to visit the dormitory, and if students will accept such

activity. (feasibility of the conclution/adv:disadv) ★★★★

6. The number of enrollments and donations from alumni would be influenced by many factors, thus merely constructing a new

dorm might not sufficiently achieve the author?s purpose. (sufficiency of the solution) ★★★

7. The author fails to consider the feasibility and negative effect of converting part of the building into office space. (feasibility of

the conclution/adv:disadv) ★★★★

8. The author fails to demonstrate that Claitown University currently needs more office space. (necessity of the solution) ★★ ——————————————————————————————

165, (健康卫生/论断/——)The following appeared in a business magazine. based on the fact that the chemists from Promofoods tested samples of the recalled cans(2) and found that, of the eight chemicals most

commonly blamed for causing symptoms of dizziness and nausea(4), five were not found in any of the tested cans. The chemists did find that the three remaining suspected chemicals(3) are naturally found in all other kinds of canned foods." ★★★

1. We do not know how many tuna cans did Promofoods sell in total last year, thus could not evaluate if the eight million return

cans are among the batch which caused the problems.(I.I/V.D) ★★★

2. We do not know if tuna cans are the only kind of food Promofoods sell last year, hence we could not evaluate if the test makes

any sense. (does the test make any difference?) ★★★

3. The author fails to rule out the possibility that it is the three remaining chemicals that resulted in the problem. (I.E) ★★★

4. The author generalizes too hastily that the tested cans do not contain any chemicals that pose a health risk at all. (C.S) ★★★

5. The chemists failed to examine the samples for other chemicals or substances that could cause dizziness and nausea. (does the

study make any difference?) ★★★★

——————————————————————————————

166, (医药卫生/建议,论断/对比)The following appeared in a local newspaper.

"People should not be misled by the advertising competition between Coldex and Cold-Away, both popular over-the-counter cold medications that anyone can purchase without a doctor's prescription. Each brand is accusing the other of causing some well-known, unwanted side

effect(1): Coldex is known to contribute to existing high blood pressure and Cold-Away is known to cause drowsiness(2). But the choice should be clear for most health-conscious people: Cold-Away has been on the market for much longer and is used by more hospitals(3) than is Coldex. Clearly, Cold-Away is more effective(4)." ★★★

1. The author fails to point out the severity of the side effects caused by each kind of medication, and the actual jeopardy for

patients. (I.I/I.C) ★★★★

2. The author fails to compare the advantages possessed by each medication. (I.I/I.C) ★★★★

3. The longer and wider use of Cold-Away might explained by other factors, it does not necessarily indicate that Cold-Away is

more effective. (U.C) ★★★★

4. Maybe there are other better cold medications available on the market. (F.D/necessity of the solution) ★★

——————————————————————————————

167, (医药保健/论断/——)A folk remedy* for insomnia, the scent in lavender flowers, has now been proved effective. In a recent study, 30 volunteers(1,2) with chronic insomnia slept each night for three weeks on lavender-scented pillows in a controlled room where their sleep was monitored(7). During the first week, volunteers continued to take their usual sleeping medication. They slept soundly but wakened feeling tired(4). During the second week, the volunteers discontinued their medication. As a result, they slept less soundly than the previous week and felt even more tired. During the third week, the volunteers slept longer and more soundly than in the previous two weeks(3,5). This shows that over a short period of time lavender cures insomnia(6,8). ★★★

*A folk remedy is usually a plant-based form of treatment common to traditional forms of medicine, ones that developed before the advent of modern medical services and technology.

1. 30 volunteers are too small a sample that can hardly be representative. (quantity of the sample) ★★★

61

2. We do not have any detailed information about the 30 volunteers, e.g. the severity of their insomnia, their physique, etc., thus we

could not evaluate if they are representative. (are the respondents representative?) ★★★★

3. The author does not inform us about the volunteers' sleep patterns prior to the experiment. (I.C/ex parte information) ★★★

4. How soundly or long a person sleeps, or how tired a person feels after sleep, is irrelevant to whether the person suffers from

insomnia. (U.C/definition of the term ?insomnia?) ★★★

5. The study fails to inform us how the volunteers felt after sleep during the third week, thus we could not evaluate the actual effect

of lavender flowers on sleep. (I.I) ★★★

6. Granted that the volunteer?s sleep was improved through the three weeks, we cannot ensure it is the lavender that caused the

result, other factors could also account for the improvement. (NCR) ★★★

7. The study should include another group of people who do not sleep on such pillows as counterparts. (lack of controlled

experiment) ★★★★

8. The author cannot hastily conclude that the volunteers? insomnia has been thoroughly cured without any follow-up studies.

(C→F) ★★★★

——————————————————————————————

168, (保健/建议/对比)concludes that the most effective way(9) to reduce the risk of fractures in later life is to take twice the recommended dose of vitamin D and calcium daily(10). The three-year study followed a group of French women(1,2) in their eighties who were nursing-home residents. The women were given daily supplements of twice the recommended dose of vitamin D and calcium. In addition, the women participated in a light

weightlifting program(5). After three years, these women showed a much lower rate(3) of hip fractures(6) than is average for their age(4,7,8). ★★

1. The study only followed female sample, thus the effectiveness of the treatment on male could not be assessed. (selective sample)

★★★

2. The group of French women may not represent the general population. (are the respondents representative?) ★★★★

3. The author does not inform us the physical condition of the French women before they participated in the study, thus we cannot

evaluate the actual effect of vitamin D and calcium on aged people. (confusing comparison and variation) ★★★★

4. The author fails to illustrate that it is the intake of vitamin D and calcium that helped lowering the rate of hip fractures among the

French women. (NCR) ★★★

5. The author fails to rule out the light weightlifting program and other factors as the cause of the lower rate of hip fractures among

these women. (NCR) ★★★★

6. We do not know the level of incidence of other kind of bone fractures among these women, thus the effect of the treatment

cannot be efficiently evaluated. (I.I) ★★★

7. Without another group of similar women were included as a counterpart, the reliability of the study is not guaranteed (lack of

controlled experiment) ★★★★★

8. Lacking any subsequent studies of the health status, especially levels of bone fractures of these women in the following years,

we could not hastily conclude that the treatment is effective to reduce the risk. (C→F) ★★★★

9. The author ignores other effective ways to reduce the risk of fractures in later life. (necessity of the solution) ★★★

10. The author fails to analyze if there is any negative effect of taking more vitamin D and calcium than recommended dose on old

people?s health. (adv:disadv) ★★★★

——————————————————————————————

169, (人事,学校建设/提议/类比)The following appeared in a letter from a department chairperson to the president of Pierce University. "Some studies conducted by Bronston College, which is also located in a small town, reveal that both male and female professors are happier(1) living in small towns when their spouses are also employed in the same geographic area. Therefore, in the interest of attracting the most(5) gifted teachers and researchers to our faculty and improving the morale of our entire(3) staff(5), we at Pierce University should offer

employment to the spouse of each new faculty member we hire(2,4,7,9). Although we cannot expect all offers to be accepted or to be viewed as an ideal job offer, the money invested in this effort will clearly be well spent(6) because, if their spouses have a chance of employment, new professors will be more likely to accept our offers(8)." ★★

1. The fact that professors are happier living in small towns might be due to other factors, but not because they are living together

with their spouses. (NCR) ★★

62

2. The chairperson fails to consider the possible differences between Bronston College and Pierce University, thus merely copying

Bronston?s activity may not be same effective at Pierce. (F.A) ★★★

3. The chairperson fails to point out what fraction of our faculty will be new members, thus the assumption that the morale of entire

staff could be improved is open to doubt. (I.I/U.A) ★★★

4. The chairperson fails to consider many other relevant factors that may influence new professors? decision. (I.T/sufficiency of the

solution) ★★★★

5. Even if the proposed actions are effective in attracting new professors, we cannot guarantee that the action will attract the most

gifted teachers and researchers, and necessarily improving the morale of stuff. (sufficiency of the solution) ★★★

6. The chairperson does not analyze the cost of the proposed actions, hence we cannot evaluate if the new professors attracted by

the action are worthy of the investment. (adv:disadv) ★★★

7. The chairperson fails to consider if the university has the capability of offering employment to the spouse of every new faculty

member. (feasibility of the conclusion) ★★★★

8. The chairperson fails to illustrate that new gifted professors will not accept our offers if we do not offer such employment.

(necessity of the solution) ★★★

9. The chairperson overlooks other better ways to achieve the same result. (necessity of the solution) ★★

——————————————————————————————

170, (商业/预言/并列)oysters(3). But scientists have now devised a process for killing the bacteria(1,2). Once consumers are made aware of the increased safety(4) of Gulf Coast oysters, they are likely to be willing to pay as much for Gulf Coast as for northeastern Atlantic Coast oysters(5), and greater profits for Gulf Coast oyster producers will follow. ★★★

1. The author does not inform us the costs of employing the process, Gulf Coast oyster producers will be profitless if the costs are

high. (adv:disadv) ★★★

2. There may be some undesired effect of using the process, e.g. affecting the taste of oysters, killing some beneficial nutrition in

oysters, etc. (adv:disadv) ★★★★

3. The author unfairly assumes that the difference in the prices of the two kinds of oysters results from the discovery of harmful

bacteria in Gulf Coast oysters. (post hoc, ergo propter hoc) ★★★

4. The author fails to inform us the actual effectiveness of the process, thus the assumption that installing the process will

necessarily lead to increased safety of Gulf Coast oysters is unjustifiable. (I.I/U.A) ★★★★

5. We cannot guarantee that consumers are willing to pay the same amount of money once Gulf Coast oysters are proved to be safe.

(sufficiency of the solution) ★★★

——————————————————————————————

171, (商业/提议/——)The following appeared in a memo from the marketing director of Bargain Brand Cereals.

"One year ago(3) we introduced our first product, 'Bargain Brand' breakfast cereal. Our very low prices quickly drew many customers away from the top-selling cereal companies(1). Although the companies producing the top brands have since tried to compete with us by lowering their prices, and although several plan to introduce their own budget brands(4), not once have we needed to raise our prices to continue making a profit(2). Given our success selling cereal, Bargain Brand should now expand its business and begin marketing other low-priced food products(5) as quickly as possible." ★★★

1. The director unfairly assumes that it is the low price of Bargain Brand that attracted those customers. (NCR) ★★★★

2. The argument contains no information concerning the actual profits of the company after ?Bargain Brand? cereal was introduced.

(I.I) ★★★

3. The director over optimistically assumes that the strong sales of Bargain Brand breakfast cereal last year will continue in the

following years. (P→F) ★★★

4. The director fails to provide any substantial evidence to show why other companies? strategies constitute no threat to our

company. (I.E) ★★★

5. The director hastily assumes that the company will succeed in selling other food products through their low-price strategy. (C.S)

★★★★

——————————————————————————————

63

172, (教育/论断/并列)The Mozart School of Music should obviously be the first choice(2) of any music student aware of its reputation(1). First of all, the Mozart School stresses intensive practice and training(3), so that students typically begin their training at a very young age(4). Second, the school has ample facilities and up-to-date professional equipment(5), and its faculty includes some(6) of the most distinguished music teachers in the world(7). Finally, many(10) Mozart graduates have gone on to be the best known and most highly paid(9) musicians in the nation(8). ★

1. The author does not inform us the amount of tuition charged by Mozart Music School, and if the gifted students could afford it.

(I.I/feasibility of the conclusion) ★★★★

2. Other music schools may be more suitable for some students. (necessity of the solution) ★★★★

3. The author fails to illustrate that intensive practice and training are important and necessary for all music students. (U.A) ★★

★★

4. Beginning intensive practice and training at very young age may cause some negative effects on music students. (adv:disadv) ★

★★★

5. We are not sure whether the equipment and facilities are available to all new students. (I.I) ★★★

6. We are not informed about the performance of other faculty members, students are not necessarily taught by those most

distinguished music teachers mentioned in the argument. (I.I) ★★★★

7. Other music schools may also have ample facilities and famous teachers. (ex parte information) ★★★

8. The author unfairly attributes the graduates? success to the education they received in Mozart School of Music. (NCR) ★★★

★★

9. The argument simply equates being best known and highly paid with musical achievements. (U.C) ★★★

10. The argument fails to provide information concerning the general employment condition of the school?s graduates. (I.I/C.S) ★

——————————————————————————————

173, (传媒/提议/并列)The following is a memorandum issued by the publisher of a newsmagazine, Newsbeat, in the country of Dinn. "Our poorest-selling magazine issues over the past three years were those that featured international news stories on their front covers(1). Over the same period, competing news-magazines have significantly decreased the number of cover stories that they devote to international news(2,3). Moreover, the cost of maintaining our foreign bureaus(4) to report on international news is increasing. Therefore, we should decrease our emphasis on international news(5) and refrain from displaying such stories on our magazine covers(6)." ★135

1. The publisher fails to establish a causal relationship between featuring international news stories on magazines? front covers and

the poor sales of these issues. (confusing concurrence with causality) ★★★★★

2. No information concerning the profits of Newsbeat and its competing news-magazines is provided, thus we cannot evaluate if

decreasing the emphasis on international news is a wise strategy. (I.C/confusing comparison and variation) ★★★

3. The fact that the competing news-magazines have decreased their emphasis on international news might just give Newsbeat the

advantage of being the only magazine covering international news. (negative evidence) ★★★

4. The publisher ignores other means to report on international news besides maintaining foreign bureaus. (necessity of the solution)

★★★★

5. Granted that featuring international news stories on the front covers did cause problems, we cannot hastily generalize that the

amount of international news should be decreased. (U.C) ★★★★

6. The publisher ignores the negative effects of the proposed strategy. (adv:disadv) ★★★

——————————————————————————————

174, (教育/提议/——)The following recommendation was made by the president and administrative staff of Grove College, a private institution, to the college's governing committee. eighty percent of the students responding to a survey(1) conducted by the student government wanted the school to remain all female, and over half of the alumni who answered a separate survey(2) also opposed coeducation. Keeping the college all-female, therefore, will improve morale among students(4) and convince alumni to keep supporting the college financially(5)." ★★★

1. Students who favor all-female education might be more likely to respond to the survey. (are the respondents representative?) ★

★★

64

2. The reliability of the second survey is also open to doubt. (reliability of the survey) ★★

3. The author fails to consider the positive effects of introducing coeducation. (adv:disadv) ★★★★★

4. The assumption that preserving all-female education will suffice to improve morale among students is unwarranted. (U.A) ★★

5. No information concerning the attitude of alumni toward coeducation is provided, thus we cannot ensure that keeping the college

all-female will necessarily convince alumni to keep supporting the college financially. (I.I/U.A) ★★★★

——————————————————————————————

175, (社会/提议/并列)The following appeared in a letter to the school board in the town of Centerville.

"All(1) students should be required to take the driver's education course at Centerville High School. In the past two years several accidents(2,3) in and around Centerville have involved teenage drivers. Since a number of(6) parents in Centerville have complained that they are too busy to teach their teenagers to drive(5), some other instruction is necessary to ensure that these teenagers are safe drivers. Although there are two driving schools in Centerville, parents on a tight budget(6) cannot afford to pay for driving instruction. Therefore an effective and mandatory program sponsored by the high school(8,9) is the only(7) solution to this serious problem(4)." ★★

1. The author?s assumption that all students in Centerville need to take the course is open to doubt. (C.S) ★★★

2. The arguer fails to point out the severity of accidents, and who actually caused these accidents.(I.I/necessity of the solution) ★

★★★

3. In the absence of comparative accident rate during earlier years, we cannot evaluate if the situation are getting worse and if the

program is necessary. (lack of comparison) ★★★

4. The high accident rate might be caused by many factors, thus implementing the program alone may not suffice to ensure lower

accident rate. (sufficiency of the solution) ★★★

5. Having no time to teach their children to drive does not necessarily imply that the parents are unable to ensure their children to

be safe drivers, the safe-driving course is not equal to driving instruction. (U.C/necessity of the solution) ★★★

6. The author fails to inform us the percentage of parents who have no time to teach their children to drive and who are on tight

budgets. (V.D) ★★★

7. The proposed program sponsored by the high school may not be the only solution to the problem. (F.D) ★★★★

8. The author fails to consider if the high school are qualified to offer the program. (feasibility of the conclusion) ★★★

9. The author fails to consider if the high school has the responsibility and enough funding to sponsor the program, and if students

will accept it.(feasibility of the conclusion) ★★★★

——————————————————————————————

176, (商业/预言/对比+并列)The following is a memorandum from the sales director to the president of the Healthy-and-Good food company.

"A recent study indicates that Venadial, a new margarine currently produced only in the country of Alta, actually reduces cholesterol levels.

Derived from the resin of pine trees, Venadial works by activating a metabolic response that is not yet well understood(4). However, cholesterol levels fell ten to fifteen percent among participants in the study who consumed Venadial daily(2), and the risk of heart attack by one-third(1,3). In addition, the new margarine is so popular that stores in Alta(5) are unable to keep it on their shelves. Therefore, if our company obtains the exclusive right(6) to sell Venadial internationally(7), our profits are sure to increase substantially within a very short time." ★★

1. The director fails to establish a causal relationship between the consumption of Venadial and the fallen cholesterol levels and the

risk of heart attack among the participants. (NCR) ★★★★

2. The study fails to include another group of people who do not consume Venadial as a counterpart. (lack of controlled experiment)

★★★★

3. We are not informed about the actual procedure of the study, thus the reliability of the method used by the study to determine the

risk of heart attack is questionable. (reliability of the study) ★★

4. Since the process of how Venadial works is not well understood yet, we cannot evaluate if Venadial has any health benefits, or if

there is any negative effect of consuming Venadial. (credibility of the evidence/adv:disadv) ★★★★

5. The fact that Venadial is very popular in Alta does not guarantee that it will also be popular all over the world. (C.S) ★★★★

6. The director fails to inform us about the cost of obtaining the exclusive right and manufacturing and distributing Venadial, thus

we cannot evaluate if the profits will necessarily increase substantially in a short time. (adv:disadv) ★★★★

7. The director fails to consider if the consumers in other regions and other countries will accept margarine. (feasibility of the 65

conclusion) ★★★★

——————————————————————————————

177, (城市,政治/论断/大并列小类比)The following is a letter that recently appeared in the Oak City Gazette, a local newspaper. "Membership in Oak City's Civic Club—a club whose primary objective is to discuss local issues—should continue to be restricted to people who live in Oak City. People who work in Oak City but who live elsewhere cannot truly understand the business and politics of the city(1). It is important to restrict membership to city residents because only residents pay city taxes(2) and therefore only residents understand how the

money could best be used to improve the city. At any rate, restricting membership in this way is unlikely to disappoint many of the nonresidents employed in Oak City, since neighboring Elm City's Civic Club(3) has always had an open membership policy, and only twenty-five nonresidents(4,5) have joined Elm City's Club in the last ten years." ★★★

1. The assumption that residents necessarily understand the business and politics of the city better than nonresidents is unwarranted.

(U.A) ★★★★

2. Paying city taxes is neither sufficient nor necessary for one to fully understand local economical and political issues.

(U.A/sufficiency/necessity of the solution) ★★★

3. The two cities might not be comparable at many aspects. (F.A) ★★★★

4. We do not know what percentage of Elm City?s Civic Club members, and what percentage of Elm City?s citizens do the 25

nonresident members make up. (V.D) ★★★

5. We should not simply exclude nonresident members merely because they are minority. The 25 nonresidents at Elm City?s civic

club might have great contributions to Elm City?s business and economy. (I.T) ★★★★

——————————————————————————————

178, (生产,管理/决策/并列)The following appeared in the annual report from the president of the National Brush Company.

"In order to save money, we at the National Brush Company have decided to pay our employees for each brush they produce(1,6) instead of for the time they spend producing brushes. We believe that this policy will lead to the production of more and better(2) brushes(3,4), will allow us to reduce our staff size, and will enable the company factories to operate for fewer hours(5)—resulting in savings on electricity and security costs. These changes will ensure that the best workers keep their jobs and that the company will earn a profit(7) in the coming year." ★★★

1. The president fails to consider the cost and feasibility of paying workers for the number of brushes they produce.

(adv:disadv/feasibility of the conclusion) ★★★

2. The president?s assumption that this policy will ensure better quality of brushes is open to question. (negative evidence) ★★★

3. Without any further study and report, the assumption that the new policy will increase workers? productivity for sure is

unwarranted. (U.A) ★★★

4. The president fails to illustrate that the productivity of our workers could be further improved. (feasibility of the conclusion) ★

★★

5. A smaller staff size and fewer operating hours would be contradictory to the president?s conclusion that the policy will

necessarily guarantee more profits. (negative evidence) ★★★★

6. There might be some negative effects of encouraging workers to produce more brushes per unit of time. (adv:disadv) ★★★★

7. The president ignores other factors that would contribute to the company?s profitability, thus implementing the policy alone

might not sufficiently ensure earning a profit. (sufficiency of the solution) ★★★★

——————————————————————————————

179, (餐饮,经营/建议/并列)The following is a memorandum written by the director of personnel to the president of the Cedar Corporation.

"It would be a mistake to rehire the Good-Taste Company to supply the food in our employee cafeteria next year. It is the second most

expensive(2) caterer in the city. In addition, its prices have risen in each of the last three years(1), and it refuses to provide meals for people on special diets(3). Just last month three employees(5) complained to me that they no longer eat in the cafeteria because they find the experience 'unbearable.'(4) Our company should instead hire Discount Foods. Discount is a family-owned local company and it offers a varied menu of fish and poultry(6). I(8) recently tasted a sample lunch(7) at one of the many companies that Discount serves and it was delicious—an indication that hiring Discount will lead to improved employee satisfaction." ★★

1. The director fails to inform us if the prices of other caterers have also been rising through the past years. (confusing comparison

and variation) ★★★

66

2. The director fails to inform us how much does Discount charge for its services. (ex parte information) ★★★

3. The mere fact that Good-Taste charges the second most expensive price and it does not provide meals for people on special diet

is insufficient evidence to illustrate that we should not continue to hire the company. (I.E) ★★★★

4. Without clear understanding of what the employees? word ?unbearable? refers to, we cannot unfairly assumes that Good-Taste is

responsible for the complaints. (definition o f the term ?unbearable?/I.I) ★★★★

5. The three unsatisfied employees could hardly constitute a statistically representative sample. (quantity of the samle) ★★★

6. The director fails to investigate if the menu provided by Discount could satisfy people on special diet. (I.I/U.A) ★★★

7. A sample lunch could hardly be representative of all lunches provided by Discount. (selective sample) ★★★

8. The director?s tastes do not necessarily represent the collective tastes of Cedar employees, thus the assumption that hiring

Discount will lead to improved employee satisfaction is unwarranted. (selective sample) ★★★★

——————————————————————————————

180, (生产,管理/提议/并列)The following is a recommendation from the personnel director to the president of Acme Publishing Company.

"Many other companies(1) have recently stated that having their employees take the Easy Read Speed-Reading Course has greatly improved productivity. One graduate of the course was able to read a five-hundred-page report in only two hours(4,5); another graduate(2) rose from an assistant manager to vice president of the company in under a year(3). Obviously, the faster you can read, the more information you can absorb in a single workday. Moreover, Easy Read costs only $500 per employee—a small price(6) to pay when you consider the benefits to Acme. Included in this fee is a three-week seminar in Spruce City and a lifelong subscription to the Easy Read newsletter. Clearly, Acme would benefit greatly(8) by requiring all(7) of our employees to take the Easy Read course." ★★

1. The director commits a fallacy of false analogy, are these companies comparable? (F.A) ★★★★

2. The condition of only two graduates tells little about the actual effect of the course. (quantity of the sample) ★★★★

3. The director fails to establish the causal relationship between taking the Easy Read course and the reading ability of the first

graduate, and the promotion of the second graduate as well. (post hoc, ergo propter hoc) ★★★★★

4. No information about the reading speed of the first graduate before taking the course is provided, thus we could not evaluate the

effect of the course. (confusing comparison and variation) ★★★

5. We do not know what kind of reading material did the first graduate read, thus cannot evaluate this piece of evidence. (I.I) ★★

6. Without providing the prices of other similar courses, the assumption that 500 dollars is a cheap tuition is open to doubt. (lack of

comparison) ★★★

7. The director fails to convince us that all employees at Acme Publishing Company need to improve their reading speed. Not every

worker has to attain a lot of information in their workplace. (C.S) ★★★

The director assumes without warrant that the benefits of the course will outweigh its costs. (adv:disadv) ★★★★

181, (保健,教育/对策/并列)From a letter to the editor of a city newspaper.

"One recent research study has indicated that many adolescents(1) need more sleep than they are getting, and another study has shown that many high school students in our city are actually dissatisfied(3) with their own academic performance(2,4). As a way of combating these problems, the high schools in our city(1) should begin classes at 8:30 A.M. instead of 7:30 A.M., and end the school day an hour later(6). This arrangement will give students an extra hour of sleep in the morning(5), thereby making them more alert and more productive(7). Consequently, the students will perform better on tests and other assignments, and their academic skills will improve significantly(8)." ★★★12

1. The author fails to indicate if the adolescents in our city need more sleep. (I.I) ★★★

2. The author does not make it clear whether the students who lack sleep are those who feel dissatisfied with their academic

performance. (I.I) ★★★

3. The fact that students are dissatisfied with their academic performance does not necessarily mean that their performance is poor.

(U.C) ★★★★

4. The author assumes without any warrant that the poor performance of those students, if it does exist, is due to inadequate sleep.

(NCR) ★★★★★

5. The proposed change in the class schedule does not guarantee that students will gain a greater amount of sleep. (U.A) ★★★★

6. The author fails to consider the negative effects of adopting the proposal. (adv:disadv) ★★★★

7. More sleep does not necessarily make students more alert and productive. (U.A) ★★★

67

8. The assumption that the change in school hours would suffice to improve students? performance on tests and other assignments,

as well as academic skills is open to doubt. (sufficiency of the solution) ★★★

——————————————————————————————

182, (餐饮/提议/并列)Butter has now been replaced by margarine in Happy Pancake House restaurants throughout the southwestern United States. Only about 2 percent of customers have complained, indicating that 98 people out of 100 are happy with the change(1). Furthermore, many(3) servers have reported that a number of(3) customers who still ask for butter do not complain when they are given margarine instead. Clearly, either these customers cannot distinguish margarine from butter, or they use the term "butter" to refer to either butter or margarine(2). Thus, to avoid the expense of purchasing butter(5), the Happy Pancake House should extend this cost-saving change to its restaurants in the southeast and northeast(4) as well. ★★★185,213

1. Some customers might be unhappy but didn?t complain. (F.D) ★★★★

2. The fact that few customers complain when they are given margarine might be explained by other factors, not necessarily by the

explanations provided by the author. (F.D) ★★★

3. The author fails to indicate the actual number of servers who reported, and what percentage of the whole server group do they

make up, thus we cannot evaluate if the fact cited above is significant. Also, the author does not tell us the percentage of customers who do not complain. (V.D) ★★★

4. The author hastily assumes that customers in other regions will respond to the change similarly as southwestern customers. (F.A)

★★★

5. The author fails to provide any information concerning the possible change in the profit of Happy Pancake House after the

replacement, thus we cannot evaluate the overall effect of the recommendation mentioned above. (adv:disadv) ★★★★

——————————————————————————————

183, (失业就业/论断/but this fear is largely unfounded(9). According to a recent study, a majority of companies(1,2) expected to make new hires in the coming year(3), while fewer companies(1,2) expected to lay off employees. In addition, although it is very disturbing to be laid off, the proliferation of programs and of workshops(4) designed to improve job-finding skills(5,6) has made being laid off far less painful(7,8) than it once was. ★143

1. The author fails to rule out the possibility that a large number of workers will suffer from unemployment in the future, because

we do not know how many new jobs will actually be created and how many workers will be laid off. (V.D) ★★★★

2. The author fails to inform us what kind of companies expected to make new hires and lay of employees respectively, thus we

cannot evaluate the situation. (I.I) ★★★

3. The result of the recent study was based only on the employment conditions in the coming year, we cannot guarantee that

large-scale unemployment will not occur in the future. (C→F) ★★★

4. We are not informed if the number of programs and workshops is enough to guarantee every laid off worker receiving necessary

training. (V.D) ★★

5. The author fails to provide any information about the actual effect of the programs and workshops, they may not be sufficiently

helpful for those who looking for new jobs. (I.I) ★★★

6. Since the programs and workshops are designed to improve job-finding skills instead of working skills, they may not be helpful

if there are no suitable job opportunities at all. (U.C) ★★★

7. The proliferation of those programs and workshops is irrelevant to whether current employees will be laid of in the future. (do the

statistics make any difference?) ★★★★

8. The proliferation of job-finding programs may well indicate increasing job attrition. (negative evidence) ★★★★

9. Since the situation of employment would be affected by various factors, the mere facts cited above in themselves could not

guarantee that the fear for employment in the future is utterly unfounded. (sufficiency of the conclusion) ★★★

——————————————————————————————

184, (社会/论断/——)In the Bayhead Public Library, books that are rarely borrowed continue to take up shelf space year after year, while people who want to read a recent novel frequently find that the library's only copy is checked out. Clearly, the library's plan to replace books that are borrowed no more than once a year with sufficient copies of more recent books(1,2) will solve this problem(3). The protest we have heard since this plan was made public has come from a small, and thus unrepresentative, group of some thirty people(4,5) and so should therefore be ignored. ★★★★

1. The author ignores the negative effects of replacing books that are rarely borrowed with more recent books. (adv:disadv) ★★★ 68

2. The author ignores the fact that the recent books will lose their popularity after a period of time, thus stocking large amount of

recent book might cause many problems. (adv:disadv) ★★★

3. The author unfairly implies that the library has only two options: to maintain the status quo or to follow the proposed plan, while

ignores other possible solutions to solve the problem. (F.D) ★★★★

4. The fact that only 30 people protest the proposal does not indicate that all other people support the proposal. (F.D) ★★★★

5. The author fails to provide detailed information about the 30 people who protest the proposal, they many have great influence on

the community. (I.I) ★★★

——————————————————————————————

185, (经营/论断/并列)The following appeared in a letter from the owner of the Sunnyside Towers apartment building to its manager. "One month ago, all the showerheads on the first five floors of Sunnyside Towers were modified to restrict the water flow to approximately 1/3 of its original force. Although actual readings of water usage before and after the adjustment are not yet available(3), the change will obviously result in a considerable savings for Sunnyside Corporation, since the corporation must pay for water each month. Except for a few complaints(4) about low water pressure, no problems with showers(2) have been reported since the adjustment(1). Clearly, restricting water flow throughout all the 20 floors(6) of Sunnyside Towers(5) will increase our profits further(7)." ★★182,213

1. The modification was taken place only one month ago, thus many negative effects might not have emerged yet. (reliability of the

survey) ★★★★

2. The owner does not provide detailed information about the usage of the first five floors of Sunnyside Towers, perhaps water flow

is not a problem here at all. (I.I/selective sample) ★★★

3. Without any actual readings of water usage before and after the adjustment we cannot evaluate if the modification could actually

save water. (I.I) ★★★

4. The fact that a few people complained about water pressure does not necessarily indicate that all other clients are satisfied with

the modification. (F.D) ★★★

5. The owner ignores many negative effects of restricting the water flow. (adv:disadv) ★★★★

6. The owner hastily assumes that the restricting on water flow could be successfully carried out to all other floors of Sunnyside

Tower. (C.S) ★★★★★

7. Adopting the owner?s suggestion alone may not suffice to ensure increased profits. (sufficiency of the solution) ★★★

——————————————————————————————

186, (生产,管理/预言/对比)The following is a recommendation from the director of personnel to the president of Professional Printing Company.

"In a recent telephone survey(1) of automobile factory workers, older employees were less likely to report that having a supervisor present increases their productivity. Among workers aged 18 to 29, 27 percent said that they are more productive in the presence of their immediate supervisor, compared to 12 percent for those aged 30 or over, and only 8 percent for those aged 50 or over(2). Clearly, if our printing

company(3) hires mainly older employees(4), we will increase productivity and save money because of the reduced need for supervisors." ★★★★

1. Since the survey is a telephone survey, the reliability of its result is suspicious at many aspects, e.g. the size of the sample, who

responded to the survey, if the respondents could exactly understand the questions, if the respondents were forthright when answering the questions, etc.(reliability of the survey) ★★★

2. The director fails to make any comparison between the actual productivity of different groups of employees. (confusing

comparison and variation/I.C) ★★★★

3. The automobile industry may not be comparable with a printing company. (F.A) ★★★

4. The director fails to consider many negative effects of hiring mainly older employees. (adv:disadv) ★★★★★

——————————————————————————————

187, (保健/建议/并列)The following appeared as part of an article in a health magazine.

"A new discovery warrants a drastic change in the diets of people living in the United States. Two scientists have recently suggested that omega -3 fatty acids (found in some fish and fish oils) play a key role in mental health. Our ancestors, who ate less saturated fat and more

polyunsaturated fat, including omega -3 fatty acids, were much less likely to suffer from depression than we are today(1). Moreover, modern societies—such as those in Japan and Taiwan—that consume large quantities of fish report depression rates(2,3) lower than that in the United 69

States(4). Given this link between omega -3 fatty acids(5) and depression, it is important for all people(9,10) in the United States(6) to increase their consumption of fish(7,8) in order to prevent depression." ★162

1.

2.

3.

4.

5.

6.

7.

8.

9. Many other factors could explain why our ancestors were less likely to suffer from depression than we are today. (I.C) ★★★★ The definition of depression may be different in the United States and in Japan or Taiwan. (I.C) ★★★ The reported depression rates may not reflect the actual status of mental health of people, many other factors could explain the lower reported rates. (U.C) ★★★ The author unfairly attributes the lower incidence of depression in Japan and Taiwan to consuming large quantities of fish. (NCR) ★★★★★ Even if eating fish is responsible for the lower depression rates, we cannot hastily assume that it is the omega-3 fatty acids contained in fish that result in the case. (NCR) ★★★ Granted that eating a lot of fish do benefit Japanese and Taiwanese on their mental health, no evidence could ensure the same good effect will also occur among people in the United States through eating fish.(F.A) ★★★ Omega-3 fatty acids are not necessarily contained only in fish, the author ignores many other solution to solve the problem. (necessity of the solution) ★★★★ Omega-3 fatty acids are not necessarily contained in all fish, thus consuming fish indiscriminatingly may not suffice to prevent depression. (sufficiency of the solution) ★★★★ The author fails to substantiate the assumption that all people in the United States need to consume a large amount of fish. (C.S)

★★★★

10. The author ignores the possibility that some Americans may not be willing to eat a lot of fish, some may not be fit for eating

more fish, or some simply could not afford to do so. (feasibility of the conclusion) ★★★★

——————————————————————————————

188, (医药卫生/建议/对比)A new report suggests that men and women experience pain very differently from one another, and that doctors should consider these differences when prescribing pain medications(3). When researchers administered the same dosage of kappa opioids—a painkiller—to 28 men and 20 women(1) who were having their wisdom teeth extracted, the women reported(5) feeling much less pain than the men, and the easing of pain lasted considerably longer in women(2). This research suggests that kappa opioids should be prescribed for women whenever pain medication is required(4), whereas men should be given other kinds of pain medication(6). In addition, researchers should reevaluate the effects of all medications(7) on men versus women.★★200

1. The number of subjects might be insufficient to constitute a representative sample. (quantity of the sample) ★★★

2. Other differences between those men and women could also explain the difference between their feelings on pain. (I.C) ★★★

3. The effect of kappa opioids on pain feeling could not be hastily generalized to all other pain medications. (C.S) ★★★★

4. The effect of kappa opioids on patients who have their wisdom teeth extracted could not be hastily generalized to all conditions

where pain medication is required. (C.S) ★★★

5. The level of pain reported by the patients may not reflect their actual level of pain. (U.C) ★★★

6. Without more information concerning the thorough effect of kappa opioids, and lacking comparison between the effectiveness of

kappa opiods and other available pain medications, we cannot conclude that it is not suitable for all male patients merely based on the fact cited above. (adv:disadv/confusing comparison and variation) ★★★

7. Granted that pain medications should be prescribed discriminatingly, the necessity of reevaluating the effects of all medications

on men versus women is unwarranted. (C.S/necessity of the solution) ★★★★

——————————————————————————————

189, (教育/提议/——)The following is a recommendation from the dean at Foley College, a small liberal arts college, to the president of the college.

"Since college-bound students(1) are increasingly concerned about job prospects after graduation, Foley College should attempt to increase enrollment by promising to find its students jobs(2) after they graduate(3,4). Many administrators feel that this strategy is a way for Foley to

compete against larger and more prestigious schools(5) and to encourage students to begin preparing for careers as soon as they enter college(7). Furthermore, a student who must choose a career path within his or her first year of college and who is guaranteed a job after graduation is more likely to successfully complete the coursework(8) that will prepare him or her for the future(6)." ★★

70

1. The dean fails to investigate actually how many students, and what percentage of all students are highly concerning about job

prospects. (V.D) ★★

2. The dean fails to consider if the college are capable of finding all its students jobs. (feasibility of the conclusion) ★★★★

3. The dean ignores many negative effects of promising to find every student a job after they graduate. (adv:disadv) ★★★★

4. The dean fails to consider if the students will accept, or welcome the policy. (feasibility of the conclusion) ★★★★★

5. Many other measures could be taken to help Foley to compete against larger and more prestigious schools. (necessity of the

solution) ★★★

6. Adopting the dean?s proposal alone may not suffice to guarantee that students will successfully complete the coursework and

prepare themselves for the future, and consequently help Foley to compete against other schools. (sufficiency of the solution) ★★★

7. The necessity of encouraging students to begin preparing for careers as soon as they enter college is open to doubt, it may cause

some undesirable results. (necessity of the solution/adv:disadv) ★★★★

8. The students may work less diligent in their coursework because of the existence of such promise. (negative evidence) ★★★★ ——————————————————————————————

190, (商业经营/论断,对策/并列)The following is a letter to the editor of the Glenville Gazette, a local newspaper.

"Over the past few years(1), the number of people who have purchased advance tickets(2) for the Glenville Summer Concert series has declined, indicating lack of community support(3). Although the weather has been unpredictable in the past few years, this cannot be the reason for the decline in advance ticket purchases, because many people(4) attended the concerts even in bad weather. Clearly, then, the reason for the decline is the choice of music(5), so the organizers of the concert should feature more modern music(6) in the future and should be sure to include music composed by Richerts, whose recordings Glenville residents purchase more often than any other contemporary recordings(7). This strategy will undoubtedly increase advance ticket purchases and will increase attendance at the concerts(8)." ★★

1. The author unfairly assumes that the decline in the sales of advance tickets will continue in the following years. (C→F) ★★

2. The number of people who purchased advance tickets does not necessarily indicate the actual attendance at the concerts. (U.C)

★★★★

3. The author fails to convince us that the lack of community support is the reason for the decline in the sales of advance tickets.

(NCR) ★★★

4. The author fails to rule out the possibility that many other people did not attend the concert because of the weather. (I.E) ★★★

5. The author unfairly assumes that it is the choice of music is responsible for the lack of community support. (F.D) ★★★★★

6. Granted that it is necessary to change the music featured on the concert, we cannot assess if the organizers should provide more

modern music. (I.E) ★★★

7. The fact that Richerts? recordings are popular in music stores does not indicate that his music will also be preferred on the

concert. Perhaps his music is not suitable for live performance, or perhaps people who purchase recordings are not the same group that would attend live concerts, or many people would not go to the concert just because they already have too many Richerts? recordings. (NCR/negative evidence) ★★★★★

8. The author unfairly assumes that the proposed actions are both necessary and sufficient to increase advance ticket purchases and

attendance at the concerts. (sufficiency/necessity of the solution) ★★★

——————————————————————————————

191, (学校建设,教育/论断/并列)The following is a letter from a professor at Xanadu College to the college's president.

"The development of an extensive computer-based long-distance learning program(2,4) will enhance the reputation of Xanadu College. This program would allow more students to enroll in our courses, thereby increasing our income from student tuition(1). Traditional courses could easily be adapted for distance learners, as was shown by the adaptation of two traditional courses(1,5) for our distance learning trial project last year. Also, by using computer programs and taped lectures, faculty will have fewer classroom obligations(6) and more time to engage in extensive research(7), thereby enhancing the reputation of Xanadu(8)." ★★★

1. The professor fails to inform us the possible cost of developing such a program and the amount of tuition we could charge for the

program, thus the assumption that their income from student tuition will increase is open to doubt. (adv:disadv) ★★★★

2. The professor fails to consider the disadvantages of using such program on teaching, and similarly, the advantages of using

traditional teaching method. (adv:disadv) ★★★

71

3. The professor does not provide any information concerning the effect of the two courses after adaptation, thus we cannot

evaluate if the adaptation could be easily carried out. (I.I) ★★★★

4. The professor fails to analyze the actual effect of this program and if students would accept it. (I.I/feasibility of the conclusion)

★★★

5. The fact that two traditional courses have been successfully adapted does not imply that all other courses could be easily adapted.

(quantity of the sample/C.S) ★★★

6. No evidence is provided to show that once the program is developed, the need for traditional teaching method will decrease.

(U.A) ★★★

7. The assumption that faculty will engage in more extensive research once they have time is unwarranted. (U.A) ★★

8. The reputation of a college might be determined by many factors, thus installing the program alone may not suffice to enhance

Xanadu?s reputation. (sufficiency of the solution) ★★★★

——————————————————————————————

192, (社会/论断/大并列小类比)The following is a letter to the editor of the Roseville Gazette.

"Despite opposition from some residents of West Roseville(1), the arguments in favor of merging the townships of Roseville and West

Roseville(2) are overwhelming. First, residents in both townships are confused about which authority to contact when they need a service; for example, the police department in Roseville receives many calls from residents of West Roseville. This sort of confusion would be eliminated with the merger(3). Second, the savings in administrative costs would be enormous(4), since services would no longer be duplicated: we would have only one fire chief, one tax department, one mayor, and so on. And no jobs in city government would be lost—employees could simply be reassigned(5). Most importantly, the merger will undoubtedly attract business investments as it did when the townships of Hamden and North Hamden merged(6,7) ten years ago." ★★★67

1. The author provides no evidence to show why we should ignore the opposition from the residents of West Roseville. (negative

evidence) ★★★

2. The author ignores many negative effects of merging the two townships. (adv:disadv) ★★★★

3. The problem of residents? confusion could be solved in other ways. (necessity of the solution) ★★★

4. The proposed merger may not necessarily ensure savings in administrative costs and higher efficiency. (U.A) ★★★

5. The author?s claim that there will be no duplicative jobs would be contradictory to the author?s assumption that no jobs in city

government will be lost. (negative evidence) ★★★

6. The author fails to convince us that merging of townships of Hamden and North Hamden actually contributed to the prosperity

of business and economy in those regions. (I.I/NCR) ★★★

7. Without evidence that the newly merged Roseville would be similar to Hamden in every aspect, we cannot guarantee that a

merger will carry the same result for Roseville as for Hamden. (F.A) ★★★★

——————————————————————————————

193, (教育/建议/对比)the district of Sanlee, 86 percent of the teachers reported assigning homework three to five times a week, whereas in the district of Marlee(6), less than 25 percent of the teachers reported assigning homework three to five times a week(2). Yet the students in Marlee earn better grades overall and are less likely to be required to repeat a year of school(3) than are the students in Sanlee(4,5,9). Therefore, all teachers in our high schools should assign homework no more than twice(7) a week(8), if at all. ★★

1. The result of the survey lacks credibility because many teachers in other subjects were actually excluded from the survey.

(selective sample) ★★★

2. The author provides no information about the respective total amount of homework assigned each time in Sanlee and Marlee.

(U.C/I.I) ★★★

3. The author fails to establish the causal relationship between homework frequency and students? performance. (NCR) ★★★★

4. Many other differences between Sanlee and Marlee could explain the differences in their students? grades and likelihood to

repeat a year of school. (I.C) ★★★★★

5. The students? grades and likelihood to repeat a year of school are not a good indication of the quality of education. (U.C) ★★

★★

72

6. The two districts may not be representative of the state's school districts overall. (C.S) ★★★

7. The author fails to illustrate why we should assign homework no more than twice a week, but not more or less. (I.E) ★

8. The author ignores some negative effects of assign inadequate homework. (adv:disadv) ★★★

9. The author unfairly assumes that any educational activity is valuable only to the extent that it enhances overall grades. (I.T) ★

★★

——————————————————————————————

194, (社会,就业/建议/对比)A recent study suggests that people who are left-handed are more likely to succeed in business than are right-handed people. Researchers studied photographs of 1,000 prominent business executives(3) and found that 21 percent of these

executives(1) wrote with their left hand(2). So the percentage of prominent business executives who are left-handed (21 percent) is almost twice the percentage of people in the general population who are left-handed (11 percent). Thus, people who are left-handed(5) would be well advised to pursue a career in business(4), whereas people who are right-handed would be well advised to imitate the business practices(6,7,8) exhibited by left-handers(9). ★★★★★

1. The author fails to indicate what fraction of all business persons are left-handed. (V.D) ★★★

2. People who write with their left hand are not necessarily left-handed. (U.C) ★★★

3. The 1,000 executives from the study are not necessarily representative of the overall population of prominent business

executives. (are the respondents representative?) ★★

4. The photographs in themselves prove little about the causal relationship between left-handedness and the ability to succeed in

business, many prominent executives might have risen to their status not by way of their achievements or business acumen but through other means. (NCR) ★★★★

5. The facts cited by the author do not indicate that all people who are left-handed are suitable for pursuing careers in business.

(C.S) ★★★

6. The author unfairly assumes that people who are right-handed also have strong will to succeed in business. (U.A) ★★★

7. Merely imitating the business practices exhibited by left-handers may not suffice to guarantee business success.

(NCR/sufficiency of the solution) ★★★★

8. Some business activities exhibited by left-handers could not be easily imitated. (feasibility of the conclusion) ★★★

9. The author assumes without substantiation that the way in which left-handed people conduct business is the only way to succeed

in business. (necessity of the solution) ★★★

——————————————————————————————

195, (出版,经营/对策/——)The following is a letter from an editor at Liber Publishing Company to the company's president.

"In recent years, Liber has unfortunately moved away from its original mission: to publish the works(1) of regional small-town authors instead of those of big-city authors(2). Just last year, 90 percent of the novels(1) we published were written by authors who maintain a residence in a big city(2). Although this change must have been intended to increase profits(3), it has obviously backfired, because Liber is now in serious financial trouble(4). The only way(5) to address this problem is to return to our original mission(7). If we return to publishing only the works of regional small-town authors, our financial troubles will soon be resolved(6)." ★★★

1. The fact that 90 percent of the novels we published were written by big city authors does not necessarily indicate that Liber has

moved away from the original mission. (C.S) ★★★★

2. Authors who maintain a residence in a big city are not necessarily equal to big city authors. (C.S/definition of the term ?big city

authors?) ★

3. The author unfairly assumes that the change must have been intended to increase profits. (U.A) ★★

4. The author fails to establish a causal relationship between publishing many novels written by big city authors and the financial

problem. (post hoc, ergo propter hoc) ★★★★★

5. Publishing only the woks of regional small town authors may not be the only solution. (F.D) ★★★★

6. The author fails to illustrate that returning to the original mission would suffice to solve the problem. (sufficiency of the solution)

★★★

7. The author fails to consider if regional small-town authors can write enough amount of works for the company to publish, and if

readers will prefer works written by small-town authors. (feasibility of the conclusion) ★★★★

——————————————————————————————

73

196, (传媒,社会/论断/并列)Sadly, widespread negative images of businesspeople have been created in large part by television(4). Consider the fact that, although they make up a mere 10 percent of the characters in dramatic roles(1) on television, businesspeople are responsible for about one-fifth of all the crime on television shows(2). In fact, in a recent survey of television producers(2), only 35 percent of the television roles for businesspeople were viewed as positive ones(3). ★★★★

1. The author fails to inform us what fraction of all characters on television shows do business people make up, or what fraction of

businesspeople in dramatic roles on television are responsible for crime in dramas. (U.C/V.D) ★★★

2. The television producers? attitude toward businesspeople may not necessarily represent that of the general public. (selective

sample) ★★★

3. The fact that 35 percent of the television roles for businesspeople were viewed as positive does not imply that all other roles for

businesspeople are negative ones. (F.D/definition of the term ?positive roles?) ★★★★

4. The author ignores other possible factors that would contribute to the widespread negative images of businesspeople. (necessity

of the solution) ★★★★

——————————————————————————————

197, (教育/提议/——)The following appeared as part of a recommendation made by a faculty member to the president of a large university. "Never once in our 150-year history as a university have we clarified our objectives. How, then, can we hope to adapt as an institution to the new challenges facing higher education(1). As a first step in this evolutionary process, therefore, we should send out questionnaires(2) asking faculty members why they teach, asking students what they want from this university, and asking former students what they gained from their own education here(3). When the replies come in, we can tabulate them and formulate an official statement of our educational mission. This will surely result in improved programs at our university(4)." ★★★★★

1. The author assumes unjustifiably that the university cannot handle problems in higher education without clarify their objectives

first. (U.A) ★★★★

2. Granted that the university need to clarify their mission, conducting the survey proposed by the author might not be necessary

for that purpose. (necessity of the solution) ★★★

3. The answers of faculty members, current and former students may not necessarily help the university to clarify their educational

missions, they may not understand the purpose of the surveys exactly, or their answers might be too trivial, or simply they are not insightful enough. (sufficiency of the solution) ★★★★★

4. The author?s assumption that the proposed actions would necessarily result in improved programs at their university is

unwarranted. (U.A) ★★★

——————————————————————————————

198, (文化艺术/论断/并列+But this belief is unwarranted: translators are sometimes distinguished authors themselves(1), and some authors may even translate their own works(2). As the translator pointed out in the preface to an English version of Dante's works, the violin and the piano make different sounds, but they can play what is recognizably the same piece of music(3).

★★★★★

1. The fact that some translators are distinguished authors themselves does not guarantee that they could preserve all the qualities

of the original work when they are translating some other?s works. (U.C/I.E) ★★★★

2. We do not know the quality of the translations done by the same author of the original works, thus the fact cited above could

lend little support to the argument. (I.I) ★★★

3. Although different instruments could play the same piece of music, for certain kind of instrument there will inevitably be some

distinctive features which others cannot imitate. (F.A) ★★★★★

——————————————————————————————

199, (社会/论断/对比)the number of meals(1) charged on the bill. They found that, while most tips were around 15 percent, the minimum percentage considered appropriate, people dining alone tipped consistently more (19 percent) and those dining in groups of four or more tipped considerably less (13 percent) than this 15 percent standard(3,4). These results strongly suggest that people dining in a group are less likely to feel personally responsible for leaving an adequate or generous tip(5). ★★★★★144

74

1. The number of meals charged on the bill does not necessarily indicate the number of people dined. (U.C) ★★

2. The argument contains no information concerning the method used by the researchers to analyze the amount of tips, thus the

reliability of the result is open to doubt. (are the respondents being forthright?) ★★★

3. The author does not inform us the total amount of money charged per person of people dining alone and those dining in groups,

waiters could still receive more tips if people dinning in groups expend significantly more money per person on their meals. (V.D) ★★★★

4. Without accurate data concerning the amount of tips received from people dining in groups of different number of people, i.e.

dining in groups of three, four, five, six, etc. respectively, we cannot evaluate if the general idea is not true. (I.I) ★★★★

5. Granted that waiters and waitresses do receive less tips from large groups of people, we cannot hastily conclude that it is because

people dining in a group are less likely to feel personally responsible for leaving an adequate tip. (NCR) ★★★★

——————————————————————————————

200, (医疗,经营/建议/对比)Statistics collected from dentists indicate that three times more(1,3) men than women faint while visiting the dentist(2). This evidence suggests that men are more likely to be distressed about having dental work done than women are. Thus, dentists who advertise to attract patients should target the male consumer(4) and emphasize both the effectiveness of their anesthetic techniques and the sensitivity of their staff to nervous or suffering patients(5,6,7). ★★★188

1. Without the base amount of male and female patients respectively, we cannot assess which group is more likely to be distressed.

(V.D) ★★★★

2. The author fails to describe the detailed circumstances when those patients faint, thus we could not evaluate the argument.

(I.I/I.C) ★★★

3. The times of people faint are not a good indication of whether people are distressed. (U.C) ★★★

4. The author ignores the negative effects of advertisement which focusing on the male consumers. (adv:disadv/necessity of the

solution) ★★★

5. The author fails to consider what fraction of all patients are likely to be nervous or suffering, and how many consumers really

care about their anesthetic techniques and sensitivity. (V.D/necessity of the solution) ★★★★

6. The author ignores many important features of a dentist which could also attract patients. (necessity of the solution) ★★★

7. The proposed advertising techniques might have the opposite effect—by calling attention to the very sorts of images that cause

distress and fainting. (negative evidence) ★★★★

——————————————————————————————

201, (生活保健/论断/并列)The citizens of Forsythe have adopted healthier lifestyles(7). Their responses to a recent survey(1) show that in their eating habits they conform more closely to government nutritional recommendations(2) than they did ten years ago(3). Furthermore, there has been a fourfold increase in sales of food products containing kiran(5), a substance that a scientific study has shown reduces cholesterol(4). This trend is also evident in reduced sales of sulia, a food that few of the healthiest citizens regularly eat(6). ★★★48

1. The reliability of the recent survey is open to doubt, who responded? The number of respondents? (are the respondents

representative?) ★★

2. The government nutritional recommendations do not necessarily represent standards of healthy lifestyles (U.C) ★★★

3. The government nutritional recommendations may have changed during the past decade. (C→P) ★★★

4. The increase in sales of food products containing kiran does not necessarily indicate that the citizens are leading healthier lives,

people may purchase these food products for other reasons. (U.C/NCR) ★★★★

5. The author fails to inform us if there are any unhealthy ingredients in those food products which contain kiran. (adv:disadv) ★

★★

6. The fact that few of the healthiest citizens regularly eat sulia does not demonstrate that sulia is unhealthy. (U.C) ★★★★

7. The author ignores many other factors that could be used to evaluate if people are having healthier lives. (sufficiency of the

evidence/I.E) ★★★★★

——————————————————————————————

202, (考古,环境/推测/extinctions(2), because there is no evidence that the humans had any significant contact with the mammals(1). Further, archaeologists have

discovered numerous sites(3) where the bones of fish had been discarded, but they found no such areas containing the bones of large mammals, 75

so the humans cannot have hunted the mammals(4). Therefore, some climate change or other environmental factor must have caused the species' extinctions(5). ★★★★

1. The fact that no evidence showing significant contact between humans and the mammals has been found does not sufficiently

prove that there was no any such contact at all.

The argument treats a lack of proof that humans might be responsible for the extinctions as constituting sufficient proof that they were not. (does the study make any difference?/I.E) ★★★★

2. Even if humans had no significant contact with mammals, human activities might cause negative effects on mammals in

different ways, thus human activities might have caused the species? extinctions indirectly. (I.T) ★★★★★

3. The author fails to prove that those discovered sites were inhabited by humans. (U.A) ★★★

4. The fact that no bones of large mammals were found at the sites does not sufficiently demonstrate that humans have not hunted

those mammals. (I.T) ★★★★

5. The author ignores other factors besides climate change and environmental ones that could also explain the extinctions. (F.D) ★

★★★

——————————————————————————————

203, (医疗卫生/论断/对比)The following appeared in a newspaper feature story.

"At the small, nonprofit hospital in the town of Saluda, the average length of a patient's stay is two days(2); at the large, for-profit hospital in the nearby city of Megaville, the average patient stay is six days(1). Also, the cure rate(2) among patients in the Saluda hospital is about twice(3) that of the Megaville hospital. The Saluda hospital has more employees per patient(5) than the hospital in Megaville, and there are few complaints(6) about service at the local hospital(4). Such data indicate that treatment in smaller, nonprofit hospitals(8) is more economical(7) and of better quality than treatment in larger, for-profit hospitals." ★★

1. The time length of a patient?s stay is not a good indication of the quality of treatment in the hospital. (U.C) ★★★★

2. Other factors might also explain the shorter length of patient stay and higher cure rate in the Saluda hospital. (I.C) ★★★★★

3. We do not know the severity of illness among patients in the Saluda hospital and in Megaville hospital, thus cannot evaluate the

cure rate in the two hospitals. (I.I) ★★★★

4. More employees per patient and few complaints do not necessarily indicate that treatment in Saluda hospital is better than that in

Megaville hospital. (U.C) ★★★★

5. A higher employee-patient ratio may just indicate organizational inefficiency at Saluda. (negative evidence) ★★★

6. Without the total number of patients going to each hospital, we cannot evaluate what percentage of their patients complained.

(V.D) ★★★

7. The author fails to provide the actual expenditures of patients in the two hospitals, thus the assumption that treatment in smaller,

nonprofit hospital is more economical is unwarranted. (I.I) ★★

8. The author hastily generalizes that treatment in other smaller, nonprofit hospitals will necessarily be more economical and of

better quality than that in larger, for-profit hospitals. (C.S) ★★★★

————————————————————————————-

204, (农业/建议/并列+递进)The following appeared as a letter to the editor of a farming publication.

"With continuing publicity about the need for healthful diets, and with new research about the harmful effects of eating too much sugar,

nationwide demand for sugar will no doubt decline(1). Therefore, farmers in our state should use the land on which they currently grow sugar cane to grow peanuts(2,3,4,9), a food that is rich in protein and low in sugar. Farmers in the neighboring country of Palin(8) greatly increased their production of peanuts last year(7), and their total revenues from that crop(6) were quite high(5)." ★★99

1. The facts offered by the author do not necessarily indicate that the demand for sugar will decline. (U.A/I.E) ★★★★

2. Granted that the demand for sugar will decline, we cannot ensure that growing sugar will no longer bring in profits for farmers in

our state. (U.A) ★★★

3. The author fails to illustrate that growing peanut will yield great profits for our farmers. (sufficiency of the solution) ★★★

4. The author fails to consider if the farms in our state are capable of growing peanuts, and if the demand for peanuts is significant.

(feasibility of the conclusion) ★★★★

5. The author fails to consider the possible cost of growing peanuts, thus the profits of farmers in Palin might be low although their

total revenues from peanuts were high. (adv:disadv) ★★★

6. Without data concerning the total number of farmers and amount of land used for growing peanuts in Palin, we could not 76

evaluate if the revenues are significant. (V.D) ★★★

7. The author unfairly assumes that the situation in Palin last year will continue in the future. (P→F) ★★

8. There may be many other differences between Palin and our state. (F.A) ★★★★★

9. The author ignores other profitable crops that we could grow. (F.D) ★★★

——————————————————————————————

205, (社会治安/决策/对比)The following appeared in a recommendation from the president of Amburg's Chamber of Commerce. "Last October the city of Belleville installed high intensity lighting in its central business district, and vandalism there declined almost immediately(1). The city of Amburg has recently begun police patrols on bicycles in its business district(3) but the rate of vandalism there remains constant. Since high intensity lighting is apparently the most effective way(4) to combat crime(7), we should install such lighting throughout(6) Amburg(2,5). By reducing crime in this way, we can revitalize the declining neighborhoods in our city(8)." ★★★239

1. The president unfairly assumes that it is the installation of high intensity lighting that resulted in the decline in vandalism in

Belleville. (post hoc, ergo propter hoc) ★★★★

2. Belleville and Amburg might not be comparable at many aspects. (F.A) ★★★★

3. The author fails to point out in what places do most vandalism at Amburg take place, thus we cannot evaluate if the police

patrols are ineffective. (I.I/U.A) ★★★

4. The president ignores many other effective ways to combat crime. (F.D) ★★★★

5. The president fails to consider the cost of installing high intensity lighting throughout Amburg and if we could afford it.

(adv:disadv/feasibility of the conclusion) ★★★

6. Granted that installing such lighting could reduce vandalism, the necessity of installing the lighting all over Amburg is open to

doubt. (C.S) ★★★★

7. Granted that installing such lighting could reduce vandalism, we cannot guarantee that it would deter other types of crimes since

vandalism is not the only type of crime in Amburg. (C.S) ★★★★

8. The president unfairly assumes that the proposed actions would sufficiently guarantee revitalization of city neighborhoods.

(sufficiency of the solution) ★★★

——————————————————————————————

206, (儿童,体育/提议/并列)The following appeared in a letter to the editor of the Parkville Daily Newspaper.

"Throughout the country(1) last year, as more and more children below the age of nine participated in youth-league softball and soccer(10), over 80,000 of these young players(2) suffered injuries(3,4,5). When interviewed for a recent study, youth-league softball players(6) in several major cities also reported psychological pressure(7) from coaches and parents to win games. Furthermore, education experts say that long practice sessions for these sports take away time that could be used for academic activities(8,9). Since the disadvantages apparently outweigh any advantages(11), we in Parkville should discontinue organized athletic competition(10) for children under nine." ★★

1. The fact that many young players suffered injuries throughout the country does not necessarily indicate that Parkville should

discontinue athletic competition. (C.S) ★★★

2. Without the total number of children who participated in the competitions we could not evaluate the situation. (V.D) ★★★

3. The author does not differentiate the severity of injuries. (I.I) ★★★

4. We need to know how about the injury-rate of those young players compared with children who do not participate in these

games. (lack of controlled experiment) ★★★

5. We do not know the rate of injuries before children participated in these games, thus could not evaluate if the situation are

getting worse. (confusing comparison and variation) ★★

6. The players in the recent study might not be representative of all young players. (are the respondents representative?) ★★★★

7. The young players at Parkville may not necessarily also suffer psychological pressure as the players do in those major cities.

(C.S) ★★★★

8. The author unfairly assumes that children would use the time for academic activities if they did not take part in those games.

(U.A) ★★★

9. No information about the academic performance of the children who participated in youth-leagues is provided, thus we could not

judge if we need to discontinue athletic competition to guarantee more academic activities for these students. (I.I/necessity of the solution) ★★

10. The fact that many children were injured in softball and soccer games does not indicate that we should discontinue all other 77

athletic competitions. (C.S) ★★★★★

11. The author fails to consider the positive effects of athletic competition. (adv:disadv) ★★★★

——————————————————————————————

207, (动物,环境/论断/先并列后递进)It is known that in recent years, industrial pollution has caused the Earth's ozone layer to thin, allowing an increase in the amount of ultraviolet radiation that reaches the Earth's surface. At the same time, scientists have discovered, the population of a species of salamander that lays its eggs in mountain lakes(1) has declined. Since ultraviolet radiation is known to be damaging to delicate tissues and since salamander eggs have no protective shells, it must be the case that the increase in ultraviolet radiation has damaged many salamander eggs and prevented them from hatching(2,3). This process will no doubt cause population declines in other species, just as it has in the salamander species(4). ★★★★150

1. The scientists only studied the population of salamanders at one certain cite, thus we could not assert that the overall salamander

population is declining. (selective sample) ★★★

2. The author fails to compare the populations of other species of salamander whose eggs are not affected by ultraviolet radiation.

(lack of controlled experiment) ★★★★

3. The author fails to rule out other possible factors that could result in the decline in the population of the salamander species.

(NCR) ★★★★★

4. We could not hastily generalize that other species will also be affected by the process merely based on the decline in the

population of certain salamander species. (C.S) ★★★★★

——————————————————————————————

208, (社会,能源/预言,论断/并列)The following appeared in a memorandum from the planning department of an electric power company. "Several recent surveys indicate that homeowners are increasingly eager to conserve energy(1) and manufacturers are now marketing many home appliances(2), such as refrigerators and air conditioners, that are almost twice as energy-efficient as those sold a decade ago. Also, new technologies for better home insulation and passive solar heating(2) are readily available to reduce the energy needed for home heating.

Therefore, we anticipate that the total demand for electricity in our area will not increase(3), and may decline slightly. Since our three electric generating plants in operation(5) for the past 20 years have always met our needs, construction of new generating plants should not be necessary(4)."

★★★

1. The author provides no evidence that homeowners who are eager to conserve energy would purchase and use those

energy-efficient home appliances. (U.A) ★★★

2. We are not informed about the price of those energy-efficient home appliances and the cost of employing the new technologies,

thus could not predict if residents could afford using them, and if they are willing to pay for these technologies. (adv:disadv/feasibility of the conclusion) ★★★

3. Many factors could lead to the increase in the demand for electricity in the future. (sufficiency of the conclusion) ★★★★★

4. The author fails to rule out many possibilities which might render one or more additional generating plants necessary. (C→F) ★

★★★★

5. The old plants might be less energy efficient than a new plant using new technology, or might have some negative effects on

environment and economic. (adv:disadv) ★★★

——————————————————————————————

209, (人事/提议/——)The following recommendation was made by the Human Resources Manager to the board of directors of the Fancy Toy Company.

"In the last three quarters of this year(1), under the leadership of our president, Pat Salvo, our profits have fallen considerably(2). Thus, we should ask for her resignation in return for a generous severance package. In Pat's place, we should appoint Rosa Winnings(6,7). Rosa is

currently president of Starlight Jewelry(4), a company whose profits have increased dramatically over the past several years(3). Although we will have to pay Rosa twice the salary that Pat has been receiving, it will be well worth it because we can soon expect our profits to increase considerably(5)." ★★26,85,140,225

1. The manager fails to illustrate that the decreased profits in the last three quarters are not temporary fluctuation but will continue

in the future. (P→F) ★★★

2. Many other factors could also contribute to the decline in the profits of Fancy Toy Company. (NCR) ★★★★

3. The manager unfairly attributes the increased profits of Starlight Jewelry to the leadership of Rosa Winnings. (NCR) ★★★★ 78

4. Even if Rosa is highly competent in her current position, we could not ensure that she can also do an excellent job as the

manager of the toy company. (F.A) ★★★★

5. The company?s profits will not necessarily increase merely because the replacement of the manager. (sufficiency of the solution)

★★★★

6. The manager fails to consider if Rosa is willing to accept the offer. (feasibility of the conclusion) ★★

7. The manager fails to inform us if there are any other competent candidates we can choose from. (necessity of the solution) ★★

——————————————————————————————

210, (工业,生产/论断/并列)The following is a letter to the editor of a news magazine.

"Clearly, the successful use of robots on missions to explore outer space in the past 20 years demonstrates that robots could be increasingly used to perform factory work(1,2,3) more effectively, efficiently, and profitably than human factory workers. The use of robots in factories would offer several advantages. First, robots never get sick, so absenteeism would be reduced(4). Second, robots do not make mistakes, so factories would increase their output(4). Finally, the use of robots would also improve the morale of factory workers(5), since factory work can be so boring that many workers would be glad to shift to more interesting kinds of tasks(6)." ★★★

1. The fact that robots are successfully used in exploring outer space does not indicate that they could also be effectively used to

perform factory work. (F.A) ★★★

2. The author fails to consider if robots which are capable of factory works are available currently. (feasibility of the conclusion)

★★★

3. The author fails to analyze the cost of using robots in factory works, and if factories could afford. (feasibility of the conclusion)

★★★★

4. The author ignores many disadvantages of using robots to perform factory work. (adv:disadv) ★★★★★

5. The morale of factory workers would actually be impaired if robots are widely used, because more human workers will be laid

off. (negative evidence) ★★★★

6. The assumption that workers will be assigned more interesting tasks once robots are used is unwarranted. (U.A) ★★

——————————————————————————————

211, (教育,就业/提议/类比)The following appeared in a memorandum to faculty from the academic vice president of Waymarsh University.

"So that we can better accomplish Waymarsh University's academic goals(1), we should adopt the job-opportunity (job-op) program(2) offered at Plateau Technical College(5) and strongly encourage all(4) students at Waymarsh to participate in it(3). The success of the job-op program at Plateau is evident: over the past two years, more than 75% of the freshmen at Plateau have enrolled in the optional job-op program(6).

Moreover, at Plateau, the grades of job-op students are consistently higher than those of other students, 90% of the job-op students receive job offers within a month after their graduation(7,8), and most former job-op students report much success in their careers(9)." ★★9, 238

1. The president fails to clarify what are Waymarsh?s academic goals, thus we cannot further evaluate the argument. (definition of

the term ?academic goals?) ★★

2. There might be alternative means of accomplishing Waymarsh?s goals, furthermore, adopting the program alone may not

sufficiently guarantee that these goals could be accomplished. (sufficiency/necessity of the solution) ★★★

3. The president fails to consider if students at Waymarsh are fit for and willing to participate in the program. (feasibility of the

conclusion) ★★★

4. The president fails to convince us that all students at Waymarsh need to participate in the program, and that all students will

benefit from it. (C.S) ★★★

5. Adopting the program at Waymarsh may not be equally effective as at Plateau, there may be many differences between the two

colleges. (F.A) ★★★★★

6. The fact that 75% of the freshmen at Plateau have enrolled in program is scant evidence to show that the program at Plateau is

successful. (do the statistics make any difference?/I.E) ★★★

7. Other differences between job-op students and other students may also account for the higher grades and job placement rates of

job-op students. (I.C/confusing the cause and the effect) ★★★★

8. We do not know the employment results of those non-job-op students, and if the job offers received by job-op students are ideal

for them, therefore cannot evaluate the effect of the program. (I.I/ex parte information) ★★★★

79

9. The report from former job-op students is misleadingly vague at many aspects. (V.D/I.I/definition of the term ?success?/are the

respondents being forthright when answering the questions) ★★★★

——————————————————————————————

212, (商业,生产/决策/——)The following appeared in a memorandum from the new president of the Patriot car manufacturing company. "In the past, the body styles of Patriot cars have been old-fashioned, and our cars have not sold as well as have our competitors' cars. But now, since many regions(3) in this country report rapid increases in the numbers of newly licensed drivers(4), we should be able to increase our

share(7) of the market by selling cars to this growing population(2,8). Thus, we should discontinue our oldest models(1) and concentrate instead on manufacturing sporty cars(5,6). We can also improve the success of our marketing campaigns by switching our advertising to the Youth Advertising agency(10,11), which has successfully promoted the country's leading soft drink(9)." ★★147

1. The president unfairly assumes that it is the body style of Patriot cars that was responsible for the decline in the sales of those

cars. (NCR) ★★★★★

2. The increase in the number of new drivers does not automatically lead to the increase in the demand for new cars. (U.C/NCR)

★★★★

3. The president unfairly assumes that those regions which report increases in the numbers of new drivers account for a statistically

significant portion of Patriot?s potential buyers. (are the respondents representative?) ★★

4. The president fails to inform us what proportion of the country experienced increases in the number of new drivers, the actual

number of these drivers, and the variation of new driver population in other regions. (V.D/I.I) ★★★

5. No evidence concerning the preference of new drivers in term of the style of cars is provided, thus we cannot evaluate if we

should manufacture sporty cars instead of old models. (I.I) ★★★★

6. Granted that the new drivers will tend to favor sporty cars, the president concludes hastily that these drivers will favor Patriot?s

sporty cars over other manufacturer?s sporty cars. (U.A) ★★★

7. We do not know what percentage of the whole car population do the newly licensed drivers make up. (V.D) ★★★

8. The president fails to consider if the new drivers could afford sporty cars. (feasibility of the conclusion) ★★★

9. The promotion of soft drink might not be comparable with that of cars. (F.A) ★★★

10. The president fails to illustrate that Patriot?s current advertising agency is partly responsible for Patriot?s poor sales, and that it is

necessary to replace the agency with another. (necessity of the solution) ★★★

11. The president fails to prove that Youth would be more effective than Patriot?s current ad agency. (I.C/ex parte information) ★★

——————————————————————————————

213, (经营/对策/类比)The following appeared in a memorandum from the owner of Armchair Video, a chain of video rental stores "Because of declining profits, we must reduce operating expenses at Armchair Video's ten video rental stores. Raising prices is not a good option(1), since we are famous for our special bargains. Instead, we should reduce our operating hours. Last month(3) our store in downtown Marston(4) reduced its hours by closing at 6:00 P.M. rather than 9:00 P.M. and reduced its overall inventory by no longer stocking any film released more than two years ago. Since we have received very few customer complaints about these new policies(2,5), we should now adopt them(6) at all other Armchair Video stores as our best strategies for improving profits(7)."

★★★182,185

1. The owner fails to sufficiently illustrate that why raising prices would not be a good solution for improving profits. (I.E) ★★★

2. The fact that few customer complaints were received does not indicate that most customers are satisfied with these policies. (F.D)

★★★

3. The new policies were implemented only one month ago, the situation of this month might not be representative of that of the

whole year, or the negative effects might not have emerged yet. (P→F /reliability of the survey) ★★★★

4. The success of the new policies at one store does not ensure that adopting them would necessarily be successful at all other

Armchair stores. (C.S) ★★★★

5. The owner fails to provide any information concerning the possible change in the profit of the store in Marston after the policies

were adopted, thus we cannot evaluate if we should adopt them at all other stores. (I.I) ★★★★

6. The owner fails to consider many negative effects of the new policies.(adv:disadv) ★★★

7. There are many other possible measures we can adopt to improve profits. (necessity of the solution) ★★★

——————————————————————————————

80

214, (教育,城市,社会/论断/对比)education. For example, Parson City typically budgets(1,2) twice as much money per year as Blue City does for its public schools—even though both cities have about the same number of residents(3,4). It seems clear, therefore, that Parson City residents care(5) more about public school education than do Blue City residents. ★★★★

1. The city?s budget for its public schools is not an accurate indication of the value it places on public education. (U.C) ★★★★

2. The author fails to inform us what fraction of the total budget are used for public schools in each city, thus we cannot evaluate

which city cares more about public school education. (V.D) ★★★★

3. Many other differences between Parson City and Blue City may also contribute to the differences in the amount of money they

budget for their public schools. (I.C) ★★★

4. We do not know the amount of students who go to public schools in the two cities, thus we could not evaluate the argument. (I.I)

★★★★

5. The amount of city budgets for public schools may not represent residents? attitude towards public school education. (U.C) ★★

★★★

——————————————————————————————

215, (工业生产,政治经济/提议/——)The following appeared in a letter to the Grandview City Council from a local business leader. "During last year's severe drought, when the water supply in the Grandview city reservoir fell to an extremely low level, the city council imposed much more rigid water-rationing rules(1). But just after these rules were imposed, industrial growth(4) in the area declined(2). This clearly shows that the new rationing rules have hurt industry in Grandview. Therefore, to promote the health of the local economy(5), the city council should now stop water rationing(3,6,7)." ★★★

1. The author fails to inform us the detailed content of the rules, thus we could not evaluate if local industry is subject to the rules.

(I.I) ★★

2. The author unfairly assumes that it is the new rationing rules that contributed to the decline in industrial growth. (post hoc, ergo

propter hoc) ★★★★★

3. The author ignores the necessity of water rationing and fails to consider the negative effects of stopping the rationing.

(adv:disadv) ★★★★★

4. The declined industrial growth does not indicate that the entire industry has declined. (U.C) ★★★

5. The author simply equates declined industrial growth with unhealthy local economy. (U.C) ★★★★

6. Merely canceling the rules may not suffice to promote the health of the local economy. (sufficiency of the solution) ★★★

7. The author ignores many other methods that could promote the health of the local economy. (necessity of the solution) ★★★ ——————————————————————————————

216, (生活/建议/并列)The following appeared in a magazine article about planning for retirement.

"Because of its spectacular natural beauty and consistent climate, Clearview should be a top choice for anyone seeking a place to retire(4). As a bonus, housing costs in Clearview have fallen(1) significantly during the past year(2), and real estate taxes remain lower than those in

neighboring towns(3). Nevertheless, Clearview's mayor promises(5) many new programs to improve schools, streets, and public services(6). Retirees in Clearview can also expect excellent health care as they grow older, since the number of physicians(7) in the area is far greater than the national average(8)." ★

1. The variation in the housing costs does not indicate that Clearview has advantage in housing costs. (confusing comparison and

variation) ★★★★

2. We cannot guarantee that housing costs in Clearview will not increase in the future. (P→F) ★★

3. We need to know the average real estate taxes of the whole nation, non-neighboring cities might enjoy even lower real estate

taxes. (I.C) ★★★★

4. We do not know other conditions in Clearview which would influence retirees? decisions, such as commodity price,

transportation, etc. (I.I) ★★★★★

5. We do not know if the mayor can keep his promise, and how long does it take to finish those improvements. (U.A) ★★★

6. The mayor?s promise might just indicate that those facilities are in bad condition now. (negative evidence) ★★★

7. The number of physicians does not necessarily indicate the quality of health care.(U.C) ★★★★

8. Although the number of physicians in Clearview is relatively high, the per capita number might be relatively low. (V.D) ★★★ 81

——————————————————————————————

217, (健身休闲,商业/建议/并列)The following appeared in a brochure promoting the purchase of local franchises for a national chain of gyms.

"Now is the time to invest in a Power-Lift Gym franchise so that you can profit from opening one of our gyms in your town. Consider the current trends: Power-Lift Gyms(1) are already popular among customers in 500 locations(2,5), and national surveys(2) indicate increasing concern with weight loss and physical fitness(3). Furthermore, last year's sales of books and magazines on personal health totaled more than $50 million, and purchases of home exercise equipment almost doubled(4,5). Investing now in a Power-Lift Gym franchise will guarantee a quick profit(6,7)." ★

1. The author fails to inform us the profitability of former Power-Lift Gyms. (I.I) ★★★★

2. The national trend and popularity of Power-Lift Gyms in other 500 locations do not necessarily ensure the gyms will be

profitable in every specific region. (C.S) ★★★★

3. The result of the national survey does not indicate that people who are concerned about weight loss and physical fitness will also

be interested in Power-Lift Gyms. (U.C) ★★★★

4. The sales of personal health books and magazines and the number of purchases of home exercise equipment could tell nothing

about the profitability of Power-Lift Gyms. (do the statistics make any difference?) ★★★★

5. The fact that Power-Lift Gyms are already popular in 500 locations, and that purchased of home exercise equipment doubled

may just indicate that the demand for such gyms will decline. (negative evidence) ★★★

6. We do not know the cost of a Power-Lift Gym, and the detailed process of installation and management of the gym, thus a quick

profit cannot be guaranteed. (adv:disadv/sufficiency of the solution) ★★★

7. Without knowing other requirements of opening such a gym we could not make the decision. (feasibility of the conclusion) ★

★★

——————————————————————————————

218, (商业/决策/大并列小类比)The following appeared in a memorandum from the president of Hyper-Go Toy Company. "Last year(1), sales of our Fierce Fighter toy airplane declined sharply(2), even though the toy had been a top seller for three years. Our customer surveys(3) show that parents are now more worried about youthful violence(4) and are concerned about better education for their children. Therefore, to maintain profits we should discontinue all our action toys(5) and focus exclusively on a new line of educational toys(6,7). Several other toy companies have already begun marketing educational toys and report sales increases(8) last year of 200 percent(9,10). And since the average family income is growing, sales of new Hyper-Go toys should also increase(11,12)." ★

1. The president fails to illustrate that the decline in the sales of Fierce Fighter last year was not an aberration and will continue in

the following years. (P→F) ★★★★

2. The president fails to analyze the profit generated by Fierce Fighter, the profit might still increase although its sales declined. (I.I)

★★

3. The results of Hyper-Go's customer survey are not necessarily representative of the overall population of toy-buying parents.

(are the respondents representative?) ★★★

4. The president fails to demonstrate that Fierce Fighter has anything to do with youthful violence. (U.C) ★★★★

5. Granted that Fierce Fighter is losing its popularity, we cannot hastily conclude that all action toys should be discontinued. (C.S)

★★★★

6. No sufficient evidence is provided to show why we should focus exclusively on educational toys. (F.D) ★★★★

7. The president fails to consider the cost of introducing a new line of educational toys and their profitability. (adv:disadv) ★★★

8. The president unfairly attributes the sales increases of those companies to their marketing of educational toys. (post hoc, ergo

propter hoc) ★★★★

9. The fact that other companies enjoyed sales increases through marketing educational toys does not guarantee that Hyper-Go

would also experience such increase by taking the same actions. (F.A) ★★★★

10. The fact that other companies enjoyed sales increases through marketing educational toys may just indicate that Hyper-Go might

encounter severe competition when marketing their toys. (negative evidence) ★★★

11. The president unfairly assumes that growing family income will necessarily lead to sales increase of toys. (NCR) ★★★★

12. The president unfairly assumes that parents who buy toys will automatically choose Hyper-Go products. (U.A) ★★★

——————————————————————————————

82

219, (商业/决策/递进)The following appeared in a memorandum from a vice president of the Megamart department store chain. "For the third year in a row, the average household income in our country has risen(1,2) significantly. That prosperity means that families are likely to be spending more time and money on leisure activities(3). Megamart stores should therefore concentrate on enlarging and promoting its line of products typically used in leisure activities(4): athletic and outdoor equipment, televisions, gourmet cooking equipment, and luggage and travel accessories(5)." ★★14,117

1. The president unfairly assumes that the average household income will continue to rise in the future. (P→F) ★★★

2. We need to know how much did average household income actually increase after adjustment for inflation, and how much did

the cost of living increase. (I.I) ★★★★

3. The increase in the average household income does not necessarily guarantee that families will spend more money on leisure

activities. (U.A) ★★★★★

4. Granted that people will be more interested in leisure activities, we cannot ensure that they will purchase those products, or that

they will purchase them at Megamart. (U.A/sufficiency of the solution) ★★★★

5. Without clear definition of ?leisure activities?, we cannot evaluate what sort of products should Megamart promote. (definition of

the term ?leisure activities?) ★★★★

——————————————————————————————

220, (传媒/建议/先对比后递进)The following appeared in an article in a magazine for writers.

"A recent study(1) showed that in describing a typical day's conversation, people make an average of 23 references to watching television and only 1 reference to reading fiction(4). This result suggests that, compared with the television industry, the publishing and bookselling(4) industries are likely to decline in profitability(2,3,5). Therefore, people who wish to have careers as writers should acquire training and experience in writing for television rather than for print media(6)." ★★★

1. Without further description of the procedure of the study, the result could lend little support to the argument. (what question was

asked in the survey?) ★★★

2. The number of references to the two activities could hardly indicate the profitability of the two industries. (U.C) ★★★★

3. The author ignores many differences between television and print media. (I.C) ★★★

4. The fact that people seldom refer to reading fiction does not imply that the entire publishing and bookselling industries will

decline in profitability. (C.S) ★★★★

5. Granted that the television industry is more profitable than the book-publishing industry, the assumption that television writers

enjoy more secure and lucrative careers than book writers is unwarranted. (U.C/NCR) ★★★★★

6. People who wish to have careers as writers could write for media besides television and print media. (F.D) ★★★★

——————————————————————————————

221, (教育/建议/递进)The following appeared in the editorial section of a student newspaper.

"In a recent survey, most students who were studying beginning Russian(2) gave higher course-evaluation ratings(1) to classes taught by

non-native Russian speakers than to classes taught by native Russian speakers. The reason that the non-native speakers were better teachers of Russian is easy to see: the non-native speakers learned Russian later in life themselves, and so they have a better understanding of how the language can be taught effectively(3). Therefore, in order to improve instruction for all languages(4) and also save money(5), our university should hire non-native speakers as language instructors instead of trying to find and recruit native speakers(6)." ★★★

1. The course-evaluating ratings given by students might not accurately represent the actual teaching quality of the teachers.

(U.C/NCR) ★★★★

2. The result of the survey was only based on students who were studying beginning Russian, thus could not be representative of all

students studying Russian. (C.S) ★★★★

3. The fact that non-native speakers learned Russian later in their lives does not necessarily guarantee that they would have a better

understanding of how the language should be taught. (U.A) ★★★★★

4. Granted that non-native speakers are more effective in teaching Russian, we cannot hastily conclude that all languages should

also be taught by non-native speakers. (C.S) ★★★★

5. The author does not inform us how much does the university pay for the two groups of teachers, thus the author?s purpose of

saving money is not necessarily ensured. (I.I/adv:disadv) ★★

6. The author ignores many positive effects of hiring native speakers as language instructors and negative effects of hiring

non-native speakers. (adv:disadv) ★★★★★

83

——————————————————————————————

222, (教育/提议/对比)The following article appeared in a recent issue of a college newspaper.

"Among all students who graduated from Hooper University over the past five years, more(1) physical science majors than social science

majors found permanent jobs within a year of graduation(2). In a survey of recent Hooper University graduates(3), most physical science majors said they believed that the prestige of Hooper University's physical science programs helped them significantly in finding a job. In contrast, social science majors who found permanent employment attributed their success to their own personal initiative(4). Therefore, to ensure that social science majors find permanent jobs(5), Hooper University should offer additional social science courses and hire several new faculty members who already have national reputations in the social sciences." ★★★

1. We cannot evaluate the employment result because we do not know the base amount of students of these two majors. (V.D) ★

★★

2. Without detailed conditions of the jobs found by students, e.g. the pay, working condition, their positions, etc., and the number of

students who found permanent jobs after one year, we cannot conclude that the job-placement record of social science majors is comparatively poor. (I.I/I.C) ★★★★★

3. The author unfairly assumes that the graduates? opinions could exactly reflect the university?s actual need. (U.A) ★★★

4. The author ignores many other factors that would contribute to the comparatively poor job-placement record of social science

majors. (NCR/I.C) ★★★★

5. The author?s proposal may be neither sufficient nor necessary for improving job-placement record of social science majors.

(sufficiency and necessity of the solution) ★★★

——————————————————————————————

223, (政治经济/论断/并列)The following appeared as an editorial in one of Coleville's city newspapers.

"Even though a high percentage of Coleville City's businesses failed last year, we who live in Coleville City should keep in mind the fact that the Coleville region(2) has attracted a great many new businesses(1) over the last three years(3). It is well known that new businesses are, on average, much more likely to fail than are long-established ones, so the business failures should not be considered a sign of poor economic health(4). Indeed, many analysts regard the presence of a significant number of new companies among a region's businesses as a sign of economic health(5). Thus Coleville City appears to be in good, not poor, economic shape." ★★★★

1. The author fails to inform us what percentage of long-established businesses failed last year, thus we could not evaluate if it is

the emergence of new businesses that resulted in the high percentage of business failures. (V.D) ★★★★

2. Without clarify the definition of the term ?Coleville region?, we cannot evaluate if there are many new businesses at Coleville

City merely based on the fact that the Coleville region has attracted many new businesses. (U.C/C.S/definition of the term ?Coleville region?) ★★★

3. The great many new businesses were emerged over the last three years, as the author cited, thus could not constitute a

convincing explanation for the failures last year. (NCR) ★★★★

4. The author fails to provide sufficient evidence to show why the business failures are not a sign of poor economic health. (I.E) ★

★★★

5. The analysts? opinion might not be true for all conditions. (U.A) ★★★

——————————————————————————————

224, (食品,商业/对策/并列)The following appeared in a corporate planning memo at ABC Cereal Company, the makers of Better Bran cereal.

"Sales of Better Bran have declined in recent years, for reasons that management has now identified. First, Better Bran is a cereal with high sugar content, and recent research studies(1) have found that most consumers say they are concerned about the amount of sugar added to their breakfast cereal. Second, the price of Better Bran has increased by 5 percent(3) in each of the last three years(2,4,5). Therefore, to increase our company's profits we need to reduce the amount of sugar in Better Bran and lower Better Bran's price(6,7)." ★★

1. We do not know if the consumers who responded to the study could represent the target customers of Better Bran. (are the

respondents representative) ★★★★

2. The decline in the sales of Better Bran may not necessarily result from its high sugar content and price. (NCR) ★★★★★

3. We need to know how much the price of Better Bran actually increased after adjustment for inflation. (I.I) ★★★

4. The author fails to consider if the cost of producing Better Bran has also increased during the same period. (adv:disadv) ★★

5. The author fails to inform us if the prices of cereals produced by other companies have also increased, and fails to make a 84

comparison between the price of Better Bran with prices of other cereals. (confusing comparison and variation/lack of comparison) ★★★

6. Adopting the author?s proposal alone might not sufficiently ensure increased profits. (sufficiency of the solution) ★★★

7. ABC can improve its profits by other means. (necessity of the solution) ★★★

——————————————————————————————

225, (人事,选举/预言,论断/并列)The following appeared as part of a letter to the editor of a local newspaper.

"During her three years in office, Governor Riedeburg has shown herself to be a worthy leader. Since she took office, crime has decreased, the number of jobs created per year has doubled(2), and the number of people choosing to live in our state has increased(1). These trends are likely to continue(3) if she is reelected. In addition, Ms. Riedeburg has promised(4) to take steps to keep big companies here(6), thereby providing jobs for any new residents(5). Anyone who looks at Ms. Riedeburg's record can tell that she is the best-qualified candidate(7) for governor." ★★26,85,140,209

1. The author unfairly attribute the decline in crime, increase in the number of new job opportunities and number of people who

choose to live here to the leadership of Riedeburg. (NCR) ★★★★★

2. The author fails to inform us how many workers have been laid off since Riedeburg took office. (I.I) ★★★

3. The author unfairly assumes that these trends will continue in the future. (P→F) ★★

4. We cannot guarantee that Riedeburg would definitely keep her promise. (U.A) ★

5. The author provides no evidence that these employers would be either willing or able to hire new residents. (I.I/U.A) ★★★

6. Reideburg?s promise may just suggest that some big companies are not willing to stay in our state. (negative evidence) ★★★

7. The author fails to consider other competent candidates. (necessity of the solution) ★★★

——————————————————————————————

226, (生产,管理运营/决策/并列+类比)The following appeared as part of a memo from the manager of an automobile manufacturing company.

"Because the demand for our automobiles is expected to increase dramatically, we need to open a new manufacturing plant(1) as soon as possible in order to continue to thrive. Our marketing projections(2) indicate that 80 million people will want to buy our automobiles(3,6), yet our existing plant can only produce 40 million automobiles. The new plant can be opened on a part-time basis, with workers from our existing site rotating responsibilities(4,5), until an operational staff can be trained. A major airplane manufacturer(7) was extremely successful(8) using this part-time rotating strategy when it opened its new plant five years ago." ★★24,100,240

1. The manager fails to consider the cost of opening a new plant, and if the company could afford it. Furthermore, the new plant

would not be necessarily if the trend of consumers? demand will not continue in the future. (adv:disadv/ C→F) ★★★★

2. Without any detailed information about the actual procedure and method used in the marketing projections, the reliability of the

projections is open to doubt. (credibility of the evidence) ★★★★

3. The manager unfairly assumes that there will be no significant change in some conditions which would influence the potential

consumers? choice and decision in the future. (C→F) ★★★★

4. The manager fails to consider if their workers are willing to accept this rotating responsibilities. (feasibility of the conclusion)

★★★

5. The manager ignores some negative effects of the part-time rotating strategy. (adv:disadv) ★★★

6. Opening a second plant might not ensure that the company could meet the projected demand. (sufficiency of the solution) ★★

7. The airplane manufacturer may not be comparable with the automobile manufacturing company. (F.A) ★★★★★

8. The manager fails to identify what constituted "success" for the airplane manufacturer, thus we cannot evaluate if the strategy is

effective. (I.I/definition of the term ?successful?) ★★★

——————————————————————————————

227, (城市,社会/提议/并列)The following appeared as an editorial in a local newspaper.

"In order to attract visitors to Central Plaza downtown and to return the plaza to its former glory(5), the city should prohibit skateboarding there and instead allow skateboarders to use an area in Monroe Park(2). At Central Plaza, skateboard users are about the only people one sees now, and litter and defaced property have made the plaza unattractive(1). In a recent survey of downtown merchants(3), the majority supported a prohibition on skateboarding in the plaza. Clearly, banning skateboarding in Central Plaza will make the area a place where people can congregate for fun or for relaxation(4,6)." ★★★

1. The author unfairly assumes that it is the presence of skateboarders that result in the current situation of the plaza. (confusing the 85

cause and the effect) ★★★★★

2. There may be some negative impacts of allowing skateboarders to use part of Monroe Park. (adv:disadv) ★★★

3. The survey was based only on data from downtown merchants, thus could not be representative of all citizens. (selective sample)

★★★★

4. The author unfairly assumes that banning skateboarding in Central Plaza would suffice to make the plaza an attractive place.

(sufficiency of the solution) ★★★★

5. The author fails to consider other possible means to regain the plaza?s prosperity. (necessity of the solution) ★★★

6. Skateboarding may actually be helpful for achieving the city's objective of providing a fun and relaxing place for people to

congregate. (adv:disadv) ★★★

——————————————————————————————

228, (城市建设/论断,提议/并列)The following appeared in a newsletter from a political organization.

"In order to promote economic growth(4) in the city, city residents should vote 'yes' on the state government's proposal to build a new

expressway linking the outlying suburbs directly to the city center(7). A direct link to the city center will enable downtown businesses to receive deliveries more frequently, so that downtown retailers will no longer run out of stock and city manufacturers will not be affected by shortages of materials(1,2,3). Booming businesses will attract qualified workers(5) from all over the state, workers who will be able to take advantage of the new expressway(6) to commute to work in our city. In additio(18,55,103,119)these advantages, hundreds of workers will be employed to build the expressway, further stimulating the local economy!" ★★★89

1. The author fails to illustrate that the stock and material problems, if any, are due to the absence of an expressway. (NCR) ★★

★★

2. The author fails to inform us to what extent downtown retailers and city manufacturers depend on the supply from the outlying

suburbs. (I.I) ★★★

3. The author fails to illustrate that the suburban areas linked by the expressway are regions from where the downtown retailers?

stock and city manufacturer?s materials come from. (I.I) ★★★

4. Merely building an expressway would not sufficiently ensure economic growth. (sufficiency of the solution) ★★★

5. The author unfairly assumes that qualified workers will be willing to and able to work in the city. (feasibility of the conclusion)

★★★

6. The author fails to consider how many such workers from the state could take advantage of the new expressway. (I.I/U.A) ★★

7. The author fails to consider the cost and negative impacts of building the expressway and if the city could afford the cost, nor

does the author illustrate that the expressway is indeed necessary. (adv:disadv/feasibility of the conclusion/necessity of the solution) ★★★★

——————————————————————————————

229, (管理运营/预言/——)The following appeared as part of a memo from the manager of a hazardous-waste disposal company. "Our new plan will help us better protect our staff against exposure to toxic chemicals(3): we are fitting each new(1) safety suit with an alarm that will sound in the main control booth when the suit is punctured(2). The control booth will notify the managers on duty, who will then take the necessary steps to ensure employees' safety. In addition to the obvious safety-related benefits, the fast reaction time will ensure minimal work stoppage(4) and thus result in increased worker productivity and company profitability(5,6)." ★★★

1. The manager does not inform us if all the staff will be equipped with this new suit. (I.I) ★★

2. We are not informed about the working mechanism of the alarming system, there is possibility of system failure, and the

manager fails to indicate if there is any emergency strategy in case of system failure. (I.I) ★★★★

3. Fitting the alarm on safety suits may not sufficiently protect staff against all chemical hazards. (sufficiency of the solution) ★★

★★

4. The manager fails to illustrate that most work stoppage are caused by problems which could be prevented by the new suits.

(sufficiency of the solution) ★★★★

5. The system in itself could not guarantee increased worker productivity and company profitability. (sufficiency of the solution)

★★★

6. We do not know the expenditure of installing the entire system, the company?s profits would probably decline due to the new

system. (adv:disadv) ★★★

86

——————————————————————————————

230, (城市,社会/提议/类比+并列)The following appeared in a recommendation from the planning department of the city of Transopolis. "Ten years ago(9), as part of a comprehensive urban renewal program, the city of Transopolis adapted for industrial use a large area of severely substandard housing near the freeway. Subsequently, several factories were constructed there, crime rates in the area declined, and property tax revenues for the entire city(2) increased(1). To further revitalize the city(6), we should now take similar action in a declining residential area on the opposite side of the city(3,4,5,7,8). Since some houses and apartments in existing nearby neighborhoods are currently unoccupied, alternate housing for those displaced by this action will be readily available(10)." ★★

1. The author unfairly assumes that the prosperity of the area results from the industrialization. (post hoc, ergo propter hoc) ★★★

★★

2. The increase in overall property-tax revenue does not necessarily indicate an increase in tax revenue from properties in the

freeway area. (C.S) ★★★

3. The author fails to demonstrate that the residential area on the opposite side of the city is suitable for industrial use. (feasibility

of the conclusion) ★★★★

4. The two areas may not be comparable at many aspects, thus taking similar action in the area may not generate the same good

effects. (F.A) ★★★★

5. The author fails to substantiate the necessity of adapt more area for industrial use. (necessity of the solution) ★★

6. Taking the same action in the area may not sufficiently revitalize the city. (sufficiency of the solution) ★★★

7. Other possible solutions could also be used to revitalize the city. (necessity of the solution) ★★★

8. The author fails to consider the cost of the proposed action, and if the city could afford it. (adv:disadv) ★★★

9. Many factors that would influence the effectiveness of the action would have changed during the past decade. (P→C) ★★★★

10. The author fails to consider if the residents of the area are willing to be allocated to those houses and apartments. (feasibility of

the conclusion) ★★★

——————————————————————————————

231, (教育/提议,论断/并列)The following appeared in a letter to the editor of a local newspaper.

"The members of the town school board should not be reelected(9) because they are not concerned about promoting high-quality education in the arts in our local schools. For example, student participation in the high school drama club(4) has been declining steadily(1), and this year the school board refused to renew the contract of the drama director(3), even though he had received many awards for his original plays(2). Meanwhile, over $300,000(5,6) of the high school budget goes to athletic programs(7), and the head football coach is now the highest-paid member of the teaching staff(8)." ★★

1. The decline in the students who participated in the drama club might be explained by other factors, but not the ineffectiveness of

the school board. (I.I) ★★★★★

2. The fact that the drama director received many awards for his original plays does not prove that he is also competent as a drama

director. (U.C) ★★★★

3. There may be some special reasons for the school board to discontinue the contract of the drama director, this piece of evidence

does not indicate that the school board are not concerned about education in the arts. (NCR) ★★★★

4. The author fails to inform us the situation of other arts programs, thus we could not evaluate if the school board are not

concerned about arts education. (I.I/selective sample) ★★★★

5. The author does not indicate the total amount of the school budget, thus we cannot evaluate the significance of the $300,000.

(V.D) ★★★

6. The author fails to demonstrate that the allocation of over $300,000 to athletic programs is unreasonable. (U.A) ★★

7. The author fails to inform us how much school budget was allocated to arts programs. (I.I/ex parte information) ★★★★

8. The fact that the head football coach is the highest-paid member of the teaching staff could tell nothing about the author?s

conclusion. (I.E) ★★

9. The author fails to consider other achievements of the school board, thus we cannot evaluate if we should not reelect the board

merely because they are not concerned about education in the arts. (adv:disadv) ★★★★★

——————————————————————————————

232, (商业,广告/决策/并列)The following appeared in a memo from the sales manager of Eco-Power, a company that manufactures tools and home appliances.

87

"Many popular radio and television commercials(1) use memorable tunes and song lyrics to call attention to the products being advertised(2). Indeed, a recent study of high school students(5) showed that 85 percent could easily recognize the tunes used to advertise leading soft drinks and fast-food restaurants(3,4). Despite our company's extensive advertising in magazines during the past year, sales of our home appliances declined(6). Therefore, to boost company profits(10), we should now switch to advertisements featuring a distinctive song(7,8,9,11)." ★

1. Radio and television commercials may use memorable tunes and song lyrics in the advertisement because of the special

characteristics of the media. (NCR) ★★

2. The manager fails to illustrate the effect of those advertisements which use memorable tunes and song lyrics. (I.I) ★★★★

3. The study fails to indicate if the students who could recognize the tunes used to advertise soft drinks and fast-food restaurant

could also memorize the product and services effectively. (I.I) ★★★★

4. The students may recognize the tunes used to advertise leading soft drinks and fast-food restaurants just because these products

and companies have been well established and are popular among students. (confusing the cause and the effect) ★★★

5. High school students may not be representative of potential customers of Eco-Power. (selective sample) ★★★★

6. The manager unfairly attributes the decline in the sales of their home appliance to the advertisement strategy. (NCR) ★★★★

7. The manager fails to consider if Eco-Power?s products are capable of being advertised with distinctive songs. (feasibility of the

conclusion) ★★★

8. The manager fails to demonstrate the comparability between Eco-Power?s products and soft drinks, and that between Eco-Power

and fast-food restaurants. (F.A) ★★★★

9. The manager fails to analyze the possible cost of advertising Eco-Power?s products through featuring a distinctive song.

(adv:disadv) ★★

10. Switching to new advertisement strategy may not suffice to boost company profits for Eco-Power. (sufficiency of the solution)

★★★

11. The manager ignores other solutions to boost company profits. (necessity of the solution) ★★★

——————————————————————————————

233, (工程建设/决策/对比+并列)The following appeared in a memo from the vice president of a company that builds shopping malls throughout the country.

"The surface of a section of Route 101(1), paved two years ago by McAdam Road Builders, is now badly cracked and marred by dangerous potholes. In another part of the state, a section of Route 66(1), paved by Appian Roadways more than four years ago, is still in good

condition(2). Appian Roadways has recently purchased state-of-the-art paving machinery, and it has hired a new quality-control manager(3,4,5). Because of its superior work and commitment to quality, we should contract with Appian Roadways rather than McAdam Road Builders(9) to construct the access roads(6,8) for all(7) our new shopping malls." ★17,41,93

1. Only one of all roads built by each company is compared, and only one section of each road is compared, thus we cannot

evaluate the overall quality of construction of the two companies. (I.C) ★★★★

2. Many other factors that would influence the condition of a road are ignored by the president. (I.C) ★★★★★

3. The arguer does not provide any information about the machinery used and the status of quality-control personnel at McAdam.

(ex parte information) ★★★

4. No information about the performance of the new quality-control manager at Appian is provided. (I.I) ★★★

5. Many factors besides equipment and on-site management would also affect the quality of a pavement job. (I.C) ★★★★

6. Granted that the quality of routes paved by Appian is superior, we cannot guarantee Appian could also do an excellent job in

constructing the access roads for shopping malls. (U.C) ★★★

7. The president hastily assumes that Appian is capable of constructing the access roads of all new shopping malls. (C.S) ★★★

8. The president fails to consider if Appian are willing to contract with the company to construct access roads. (feasibility of the

conclusion) ★★

9. Other competent companies might be available for the president to choose from. (F.D) ★★★

——————————————————————————————

234, (生活,保健/建议,论断/对比)The following appeared in a newspaper feature story.

"There is now evidence that the relaxed manner of living in small towns promotes better health and greater longevity than does the hectic pace of life in big cities. Businesses in the small town of Leeville(5) report fewer days of sick leave(1) taken by individual workers than do businesses 88

in the nearby large city of Mason City(5). Furthermore, Leeville has only one physician(2) for its one thousand residents(3), but in Mason City the proportion of physicians to residents is five times as high. And the average age of Leeville residents is significantly higher(4) than that of Mason City residents. These findings suggest that people seeking longer and healthier lives should consider moving to small communities(6,7)." ★★

1. The number of days of sick leave taken by individuals does not necessarily indicate their health status. (U.C) ★★★★

2. The proportion of physicians to residents is not a good indication of the health level of citizens. (U.C) ★★★★

3. The fact that Leeville has lower proportion of physicians to residents may just suggest that residents here may have difficulty in

finding adequate health care. (negative evidence) ★★★★

4. The author ignores many other differences that would contribute to the higher average age of Leeville residents. (I.C) ★★★★

5. Leeville and Mason City are not necessarily representative of all small towns and big cities. (C.S) ★★★★

6. Granted that living in small towns does promote better health and greater longevity, we cannot ensure that living at small

communities would have the same advantages. (U.C) ★★★★

7. The author fails to consider the disadvantages and inconveniences of living in small towns. (adv:disadv) ★★★★

——————————————————————————————

235, (传媒/决策/并列)The following appeared in a memorandum from the general manager of KNOW radio station.

"Several factors indicate that radio station KNOW should shift its programming from rock-and-roll music to a continuous news format. Consider, for example, the number of older people in our listening area has increased dramatically(1), while the total number of our listeners has recently declined(2). Also, music stores in our area report decreased sales of recorded music(3,4,5). Finally, continuous news stations in neighboring cities have been very successful(6), and a survey taken just before the recent election shows that local citizens are interested in becoming better informed about politics(7)." ★6,97

1. The arguer falsely assumes that older people do not prefer rock-and-roll but favor all-news programming. (U.A) ★★★

2. Several other factors may also contribute to the decline in the number of listeners. (NCR) ★★★★

3. The sales of recorded music are not a good indication of whether people prefer rock-and-roll. (U.C) ★★★★

4. We do not know what types of music recordings actually experienced sales decline in the music stores, thus cannot evaluate if

the decline results from decreasing popularity of rock-and-roll. (I.I) ★★★

5. The manager fails to illustrate that people who buy music recordings are basically the same group of people who listen to music

on the radio. (are the respondents representative?) ★★★

6. The fact that news stations in neighboring cities have been successful does not indicate that KNOW could also be successful

through shifting its programming to a continuous news format. (F.A) ★★★★

7. The fact that citizens are interested in becoming better informed about politics before the election does not necessarily imply that

they will also be interested in continuous news program all the time. (When was the survey conducted?) ★★★★

——————————————————————————————

236, (教育/提议/大并列小类比)The following appeared as a letter to the editor of a local Solano newspaper.

"The Solano school district should do away with its music education programs. After all, music programs are not especially popular in our schools: only 20 percent(1) of our high school students participated in music programs last year. Furthermore, very few of the Solano district's college-bound students(3) are interested in majoring in music at the university level(2,4). Also, when the school district of our neighboring town of Rutherford eliminated its music programs two years ago, the overall grade point average in the high school increased by 10 percent(4,5) the next year(6). Surely the money spent on Solano music programs could be better used to improve the quality of traditional academic

courses(7,8)." ★★133

1. 20 percent of our high school students might be a significant group, and the fact mentioned above does not prove that students

are uninterested in music. /We do not know the percent of Solano high school students who participated in other programs. (V.D/lack of comparison) ★★★★

2. The fact that few college-bound students are interested in majoring in music in the future does not necessarily indicate that music

education programs are unnecessary in our schools. (I.E) ★★★★

3. The author fails to consider the attitude of non-college-bound students toward music education. (selective sample) ★★★★

4. The author unfairly assumes that any education program is valuable only to the extent that it enhances overall grades and only if

students choose to pursue that course of study in college. (U.A/I.T) ★★★★

5. The author unfairly assumes that it is the elimination of music programs in Rutherford that resulted in the increase in the overall 89

grade point average. (NCR) ★★★★

6. There might be many other differences between Rutherford and Solano that render the two districts incomparable. (F.A) ★★★

7. The author fails to demonstrate that the quality of traditional academic courses in Solano district needs to be improved.

(necessity of the conclusion) ★★

8. The assumption that the money spent on music programs will automatically be used to improve the quality of traditional

academic courses once music programs are eliminated is unwarranted. (U.A) ★★

——————————————————————————————

237, (城市,经济/建议/——)The following appeared as part of an article in a local Beauville newspaper.

"According to a government report, last year(5) the city of Dillton reduced its corporate tax rate by 15 percent; at the same time, it began offering relocation grants and favorable rates on city utilities to any company that would relocate to Dillton. Within 18 months, two

manufacturing companies moved to Dillton, where they employ a total of 300 people(1,2,3). Therefore, the fastest way(6) for Beauville to stimulate economic development and hence reduce unemployment is to provide tax incentives and other financial inducements that encourage private companies(7) to relocate here(4)." ★★2,25

1. The author unfairly assumes that Dillton?s current prosperity, if any, results from the policies implemented last year. (post hoc,

ergo propter hoc) ★★★★★

2. The fact that two manufacturing companies moved to Dillton and employ 300 people is insignificant evidence to show that

Dillton are experiencing prosperity. (I.E) ★★★★

3. Without more information about Dillton's economic conditions and employment level both before and after the incentives were

established it is impossible to assess whether those incentives had a positive or negative impact—or any impact at all—on Dillton's overall economy. (adv:disadv/I.I/lack of comparison) ★★★

4. Granted that the policies are responsible for the prosperity of Dillton, we cannot ensure that Beauville could experience the same

prosperity through providing similar incentives. (F.A) ★★★★★

5. The author fails to substantiate the assumption that during the last 18 months all conditions upon which the effectiveness of the

policies depends have remained unchanged. (P→C) ★★★

6. The arguer ignores other possible methods that may improve Beauville?s economy more efficiently. (necessity of the solution)

★★★

7. Granted such incentives are indeed necessary, the author fails to illustrate why we should target on private companies. (I.E) ★

——————————————————————————————

238, (就业/提议/对比)The following appeared in a memorandum from the president of Mira Vista College to the college's board of trustees. "At nearby Green Mountain College, which has more business courses and more(1) job counselors than does Mira Vista College, 90 percent of last year's graduating seniors had job offers(4) from prospective employers(5). But at Mira Vista College last year, only 70 percent of the seniors who informed the placement office that they would be seeking employment(2) had found full-time jobs(4) within three months after graduation, and only half of these graduates were employed in their major field of study(3). To help Mira Vista's graduates find employment(8), we must offer more courses in business and computer technology(6) and hire additional job counselors(7) to help students with their resumés and interviewing skills." ★★9,211

1. The president fails to inform us the total amount of students at each college, thus we cannot compare the number of job

counselors per student. (V.D/I.C) ★★★

2. The survey does not include all the graduates in MVC, thus it is impossible to evaluate the employment result in MVC.

(selective sample) ★★★★

3. The comparison between GMC and MVC is incomplete, many other factors would contribute to the differences in the

employment results of the two colleges. (I.C) ★★★★★

4. Without further description of the job offers received by graduating seniors of the two colleges, we cannot evaluate if Mira

Vista's job-placement record is comparatively poor. (I.I) ★★★★

5. The president fails to establish the causal relationship between the fact that GMC has more business courses and job counselors

than MVC and the fact that 90% of last year?s graduating seniors from GMC got job offers.(confusing concurrence with causality) ★★★★★

90

6. The president fails to convince us that offering more courses in computer technology will necessarily contribute to the

employment of graduates in MVC. (I.E) ★

7. GMC and MVC might not be comparable in several aspects such as reputation, ranking, alumni, etc. (F.A) ★★★★

8. The proposed actions might be neither sufficient nor necessary for Mira Vista to improve its employment result.

(sufficiency/necessity of the solution) ★★★

——————————————————————————————

239, (社会/建议/并列+类比)The following appeared as an editorial in the local newspaper of Dalton.

"When the neighboring town of Williamsville adopted a curfew four months ago that made it illegal for persons under the age of 18 to loiter or idle in public places after 10 p.m., youth crime(2) in Williamsville dropped by 27 percent during curfew hours(1,2). In Williamsville's town square(5), the area where its citizens were once most outraged at the high crime rate, not a single crime has been reported(4) since the curfew was introduced. Therefore, to help reduce its own rising crime rate, the town of Dalton should adopt the same kind of curfew(6,7,8). A curfew that keeps young people at home late at night will surely control juvenile delinquency(9) and protect minors from becoming victims of crime(10)." ★★205

1. We are not informed about the variation in crime committed before curfew hours. (I.I) ★★★

2. We do not know if crime of other type also dropped since the curfew was introduced. (I.I) ★★★

3. The decline in Williamsville's overall crime rate is not necessarily attributable to the curfew. (post hoc, ergo propter hoc) ★★★

4. We do not know how many crimes were committed at the town square but not reported. (I.I) ★★★

5. The amelioration of the town square may not be representative of the situation of the entire town. (selective sample/quantity of

the sample) ★★★★

6. We do not know what fraction of all crime in Dalton is youth crime, thus could not evaluate if the same kind of curfew will also

be effective in Dalton. (I.I/sufficiency of the solution) ★★★

7. Dalton might differ from Williamsville in ways that would undermine the curfew's effectiveness in Dalton. (F.A) ★★★★★

8. There are many other ways to reduce Dalton?s crime rate. (sufficiency of the solution) ★★★

9. Granted that the proposed curfew would reduce Dalton's overall crime rate, the author unfairly infers that the curfew would also

curb juvenile delinquency. (U.C/definition of the term ?juvenile delinquency?) ★★★

10. Crimes against youths might occur during non-curfew hours, thus the inference that the curfew would protect minors from be-

coming crime victims is unwarranted. (I.T/U.A) ★★★

——————————————————————————————

240, (学校建设/提议/并列)The following appeared in a memo written by a dean at Buckingham College.

"To serve the housing needs of our students, Buckingham College should build a new dormitory(4). Buckingham's enrollment is growing and, based on current trends(1), should double over the next fifty years, thus making existing dormitories inadequate. Moreover, the average rent for an apartment in our town has increased(3) in recent years(2). Consequently, students will find it increasingly difficult to afford off-campus housing. Finally, an attractive new dormitory would make prospective students more likely to enroll at Buckingham(5,6)." ★★★24,100,226

1. The dean assumes without justification that the trends of increasing enrollment will continue in the future. (C→F) ★★★

2. The dean unfairly assumes that the rent for off-campus apartment will continue to increase in the following years. (C→F) ★★

3. We do not know if the average rent for off-campus apartment has increased to an extent that students cannot afford.

(I.I/confusing comparison with variation) ★★★

4. The dean fails to illustrate that current dormitories in Buckingham could not meet students? housing needs. (necessity of the

solution) ★★★

5. A new dormitory would be neither sufficient nor necessary for attracting prospective students. (sufficiency/necessity of the

solution) ★★★

6. The dean?s assumption that more prospective students will enroll at Buckingham conflicts the dean?s purpose of solving the

housing problem. (negative evidence) ★★★

——————————————————————————————

241, (失业,就业/论断/对比)The following appeared in a memo at the XYZ company.

91

found jobs within a year(2,3,4,5). Moreover, Delany is clearly superior, as evidenced by its bigger staff(7) and larger number of branch offices(6). After all, last year(10) Delany's clients took an average of six months to find jobs, whereas Walsh's clients took nine(8,9)." ★★

1. The author unfairly attributes the successful employment result of the laid-off employees who used Delany to Delany?s

assistance. (NCR) ★★★★

2. The fact that only half of the laid-off workers found jobs within a year may not result from ineffectiveness of Walsh. (NCR/I.C)

★★★★

3. Many factors could explain the differences between the employment results of using each company. (I.C) ★★★★

4. Without any comparative data, we cannot determine if the employment result of workers who use Walsh eight years ago are poor.

(lack of comparison) ★★★★

5. Many conditions would have changed during the past eight years. (P→C) ★★★

6. The size of staff and number of branch offices do not necessarily indicate the quality and effectiveness of a company?s service.

(U.C) ★★★★

7. A bigger staff may just indicate that the efficiency at Delany is lower. (negative evidence) ★★

8. There is no significant difference between finding job within six months and within nine months. (I.E) ★★

9. Without detailed description of jobs found by Delany?s clients and Walsh?s clients respectively, and without more information

about their clients, we cannot evaluate which company is more effective. (I.I/I.C) ★★★★

10. A single year's placement statistics hardly suffices to draw any firm conclusions. (selective sample/ P→F) ★★★

——————————————————————————————

242, (教育/建议/类比+并列)The following appeared as an editorial in the student newspaper of Groveton College.

"To combat the recently reported dramatic rise in cheating among college and university students, these institutions should adopt honor codes(8) similar to Groveton's(9,10), which calls for students to agree(1) not to cheat in their academic endeavors and to notify a faculty member if they suspect that others have cheated(2). Groveton's honor code replaced an old-fashioned system in which students were closely monitored by teachers and an average of thirty cases of cheating per year were reported. The honor code has proven far more successful: in the first year it was in place, students reported(5) twenty-one cases of cheating; five years later(6), this figure had dropped to fourteen(3,4).

Moreover, in a recent survey conducted by the Groveton honor council(7), a majority of students said that they would be less likely to cheat with an honor code in place than without." ★★

1. The assumption that students will keep their promise under the existence of the codes is unwarranted. (U.A) ★★★★

2. The author unfairly assumes that Groveton students are just as capable of detecting cheating as faculty monitors, and that these

students are just as likely to report cheating whenever they observe it. (U.A) ★★

3. The author fails to illustrate that the number of cases of cheating at Groveton would be higher if we did not adopt the honor code.

(lack of controlled experiment) ★★★★

4. Other factors could also explain the decreasing number of cases of cheating at Groveton. (post hoc, ergo propter hoc) ★★★★

5. The author fails to investigate the number of cases of cheating which were not reported. (I.I) ★★★★

6. We do not know the situation of cheating in other years. (I.I) ★★★

7. If the students will be forthright under the circumstances in which the survey was set is suspicious (are the respondents being

forthright when answering the questions?) ★★★★

8. The author fails to consider the negative effects of adopting such honor codes. (adv:disadv) ★★★

9. The honor code may not be as same effective at other colleges and universities as it was at Grovetown. (C.S) ★★★

Other measures could be taken to combat cheating. (necessity of the solution) ★★★

附:题号为黄皮写作教程第一章翻译的顺序,网络顺序将前20题放到最后20题就是黄皮书顺序,比如网络顺序的第21题就是黄皮顺序的第一题

92

1、教育类(35篇)

12,14,19,30,31,32,33,35,47,58,60,70,74,78,80,82,84,86,92,108,110,112,114,133,134,171,181,193,194,202,203,208,210,212,229

2、学习类(21篇)

225,228,8,22,37,39,53,72,97,102,118,144,157,159,161,163,180,184,197,200,209

3、行为类(61篇)

1,5,7,11,15,42,44,48,51,54,55,61,64,66,67,71,75,79,98,101,107,116,117,119,121,125,126,128,136,137,142,143,145,148,155,162,164,166,167,168,172,174,179,186,188,189,190,191,192,196,205,206,211,214,215,216,218,219,230,236,242

4、社会类(44篇)

2,3,9,13,16,17,20,21,26,27,38,41,45,50,52,56,57,69,73,93,100,106,113,120,122,129,132,140,150,151,165,173,178,183,231,232,224,226,227,233,239,240,243,244

5、文化类(9篇)

43,127,152,153,156,187,207,230,234

6、科技类(21篇)

10,46,49,68,87,94,95,99,104,109,115,130,131,135,139,146,176,199,213,217,231

7、传媒类(6篇)

18,40,89,91,141,195

8、历史类(8篇)

6,28,54,83,90,105,169,201

9、艺术类(10篇)

62,76,81,111,123,124,138,190,198,223

10、政治类(18篇)

4,23,24,25,36,59,63,65,77,85,88,103,147,149,175,182,204,232

93

11、国际类(5篇)

96,177,185,235,237

12、法律类(4篇) 154,158,160,241

13、文学类(2篇) 29,238

94

更多相关推荐:
GRE argument官方满分范文分析

argument全部官方范文分析写在前面的话1官方范文的重要性不言而喻论坛里对官方范文的态度也有褒有贬有人说这些6分的官方文章都是大牛写出来的我们学不来于是有的同学的重心就转向了北美范文但有一点是不可否认的官方...

GRE Argument 满分范文5篇

ArgumentSample1InthisargumenttheauthoremploysavarietyofevidenceaboutPaleansincludingtheirgeographicalisolationandth...

GRE 写作argument全部官方范文分析汇总

GRE写作argument全部官方范文分析汇总写在前面的话1官方范文的重要性不言而喻大家对官方范文的态度也有褒有贬有人说这些6分的官方文章都是大牛写出来的我们学不来于是有的同学的重心就转向了北美范文但有一点是不...

GRE写作Argument范文1

6开头1Inthisarticletheauthorconcludesthatthedeclineinarcticdeerpopulationmustbeduetoglobalwarmingwhichhasle...

GRE写作argument范文

GRE写作Argument高分范文题目ThefollowingappearedinamemorandumfromthegeneralmanagerofKNOWradiostationquotKNOWshould...

GRE argument 范文选

ThenotionprovidedbythearguerthatBargainBrandshouldexpandtheirmarketingseemsatfirstglancetobereasonableAfterallitiss...

GRE写作Argument高分范文

GRE写作Argument满分范文题目TheMozartSchoolofMusicshouldobviouslybethefirstchoiceofanymusicstudentFirstofalltheMoz...

新GRE argument 模板 原创

开头InthisargumentthearguerarrivesattheconclusionthatTheconclusionisbasedonInadditiontothisthearguerprovide...

英语_GRE作文Argument模板之2错误因果

3The15declinethattheauthorcitesisnotnecessarilyduetothevocationalpreferencesofnewlawschoolgraduatesItisen...

GRE写作规则(Argument部分)

GRE写作规则Argument部分一学会使用TS不仅全文要有TS每个段落中也要有自己的TS这样写起来思路清晰看起来也省力TS放在段首目标在一个段落中不能找到任何一句与TS无关或与TS冲突的句子但过渡到下一段的过...

GRE作文argument41、123、125

1出事故是否与戴头盔有关论者的论证不能足够说明首先论者对于戴头盔会使人更容易冒险的结论完全没有根据他没有提供任何资料证明戴头盔的人都觉得这样很安全完全有可能戴头盔的人会比不戴的人更加小心其次论者也没有提供有多少...

GRE作文Argument模板+错误分类_必备

弃我去者昨日之日不可留乱我心者今日之日多烦忧ArgumentExample调查1Selectivesample样本选择一部分被排除在外Onemajorproblemisaboutthesampletheauth...

gre argument (26篇)